Download as pdf or txt
Download as pdf or txt
You are on page 1of 94

Indian Economy by Aman Soni

SOME OF OUR INITIATIVES

3. Creditors
Choose the correct answer.
A. 1 only
B. 1 and 2 only
C. 1 and 3 only
D. 2 and 3 only

Ans - C
Salaried workers such as clerks, teachers, and other
white collar persons lose purchasing power when there
is inflation. The reason is that their salaries are slow to
adjust to rising prices.
New Edition Coming Soon ! During periods of rising prices, debtors gain and
creditors lose. When prices rise, the value of money
falls. Though debtors return the same amount of
money, they pay less in terms of value of money or
purchasing power of money. Thus, inflation brings
about a redistribution of real wealth in favour of
debtors at the cost of creditors.

Q2. Arrange the following in descending order of


weights under the National CPI :
1. Food and Beverages
2. Housing
3. Fuel and light
4. Clothing bedding footwear
Choose the correct answer.

A. 1, 2, 3 , 4
Q1. A high Inflation is NOT beneficial for the following B. 1, 2, 4, 3
category of people : C. 3, 4, 1, 2
1. Fixed Salaried workers D. 4, 3, 2, 1
2. Debtors

Telegram – Indian_Economy_by_Aman_Soni Page 1


Ans - A Core or underlying inflation measures the long run
trend in the general price level. Temporary
effects on inflation are factored out to
calculate core inflation.
For this purpose, certain items are usually
excluded from the computation of core
inflation. These items include: changes in the
price of fuel and food which are volatile or
subject to short-term fluctuations and or
seasonal in nature like food items. So
inflation calculated without food and fuel
component is called Core Inflation

Q3. Choose the correct statements. In other words, core or underlying inflation is an
1. WPI includes all distribution costs and taxes of alternative measure of inflation that eliminates
retailers. transitory effects. These price changes are not within
2. CPI does not include inter-mediate goods the control of monetary policy as much as these are
transacted in the economy under its supply shocks(price of food and fuel depend on supply
calculations. and not on demand).

A. 1 only
B. 2 only Q5. Choose the correct statements.
C. Both 1 and 2
D. Neither 1 nor 2 1. The responsibility of achieving the inflation
target is primarily based on the Finance
Ans – B Ministry.
2. The Government does not play any role in
WPI is based on wholesale prices for primary articles, inflation targeting in the economy.
administered prices for fuel items and ex-factory prices
for manufactured products. On the other hand, CPI is A. 1 only
based on retail prices, which include all distribution B. 2 only
costs and taxes. C. Both 1 and 2
D. Neither 1 nor 2
CPI covers only consumer goods and consumer services
while WPI covers all goods including inter-mediate Ans - D
goods transacted in the economy. Inflation targeting focuses mainly achieving price
stability as the ultimate objective of monetary policy.
Q4 . Choose the INcorrect statements. This approach entails the announcement of an inflation
target – either a number or a range, that the central
1. Under the Core inflation calculations, certain bank promises to achieve over a given time period.
volatile items are excluded. The targeted inflation rate will be set jointly by the RBI
2. Prices of food and fuel depend more on supply and the Government, although the responsibility of
and not on demand. achieving the target would rest primarily on the RBI.
This would reflect an active government participation in
A. 1 only achieving the goal of price stability with fiscal discipline
B. 2 only (not borrowing in excess).Monetary policy and fiscal
C. Both 1 and 2 policy have to converge for achievement of inflation
D. Neither 1 nor 2 targeting.

Ans – D

Telegram – Indian_Economy_by_Aman_Soni Page 2


Q1. Choose the correct statements. A CD is issued at a discount to the face value, the
discount rate being negotiated between the issuer and
1. A higher call money rate indicates a liquidity the investor.
stress in the economy. The maturity period of CDs issued by banks should be
2. As per a RBI notification, Payment Banks are not not less than 15 days and not more than one year. The
allowed to participate in the Call Money FIs can issue CDs for a period not less than 1 year and
Market. not exceeding 3 years from the date of issue. CDs can be
issued to individuals or firms.
A. 1 only
B. 2 only
C. Both 1 and 2 Q3. Choose the correct statements.
D. Neither 1 nor 2
1. Both the Central and state governments can
Ans – A issue Treasury bills.
Call/Notice money is money borrowed or lent for a very 2. Treasury bills pay a fixed coupon interest every
short period. If the period is more than one day and up month to the holders.
to 14 days. It is called ‘Notice money’ if the amount is
borrowed for 1 day or for overnight purposes it is A. 1 only
known as' Call money’. No collateral security is required B. 2 only
to cover these transactions. The call market enables the C. Both 1 and 2
banks and institutions to even out their day to day D. Neither 1 nor 2
deficits and surplus of money, meet CRR, SLR
requirements and sudden demand for money. Ans – D

As per the new regulations, Payment Banks are also In India, the Central Government issues both, treasury
allowed to participate in CMM as both lenders and bills and bonds or dated securities while the State
borrowers.Since the participants are banks, the call Governments issue only bonds or dated securities,
money rate tells about the overall liquidity position in which are called the State Development Loans (SDLs). G-
the economy. Higher call rate indicates liquidity stress in Secs carry practically no risk of default and, hence, are
the economy. In this case, the RBI may increase the called risk-free gilt-edged instruments.
liquidity by OMOs, reducing repo rate etc.
Treasury bills are short term money market
Q2 . Choose the INcorrect statements. instruments, which are issued by the RBI on behalf of
the Government of India. The Government of India uses
1. A Certificates of Deposit (COD) is redeemed at these funds to meet its short term financial
par. requirements. T-Bills are sovereign zero risk
2. Regional Rural Banks and Local Area Banks can instruments. They are available in primary and
issue COD to local farmers. secondary market & issued at a discount to face value.
Treasury bills are zero coupon securities and pay no
A. 1 only interest. Instead, they are issued at a discount and
B. 2 only redeemed at the face value at maturity.
C. Both 1 and 2
D. Neither 1 nor 2 Q4. Choose the INcorrect statements.

Ans - B 1. G-Secs can be purchased only by companies and


Certificate of deposit (CD) is issued by Scheduled financial institutions.
Commercial Banks and Fls. Regional Rural Bank and 2. NRI’s are not eligible to invest in G-secs of
Local Area Banks cannot issue CDs. Govt. of India.

Telegram – Indian_Economy_by_Aman_Soni Page 3


A. 1 only Holders of debt security receive interest and repayment
B. 2 only of the principal.
C. Both 1 and 2
D. Neither 1 nor 2 Q1. Choose the correct statements.

Ans - C 1. P-notes are Offshore Derivative Instruments


G-Secs as are securities issued by the RBI on behalf of (ODIs) issued by registered Foreign Portfolio
the government of India to meet the latter’s borrowing Investors (FPIs) with Indian stocks as their
programme for financing fiscal deficit. The G-Sec underlying assets.
instrument is in the nature of a bond.G-Secs have a 2. The P- Note holder investors have to be directly
maturity period ranging from 1 to 30 years and they registered under SEBI.
carry a coupon rate (interest rate) which is paid semi-
annually. They are issued both in demat and physical A. 1 only
form. B. 2 only
C. Both 1 and 2
Government of India Dated Security can be held by any D. Neither 1 nor 2
person, firm, company, corporate body, State
Governments, Provident Funds and Trusts. NRIs, Ans – A
overseas corporate bodies predominantly owned by Participatory notes are often referred to as PNs or P-
NRIs and Foreign Institutional Investors registered with Notes. These are financial instruments used by investors
SEBI and approved by Reserve Bank of India are also and hedge funds to invest in the Indian securities, and
eligible to invest in the Government Stock. no registration is required with the SEBI, the market
watchdog in India. Investments flowing in through PNs
Q5. Choose the correct statements. are considered as offshore derivative investments
(ODIs).
1. Bond holders of a company always have a
voting right in the Annual General Meeting This has enabled the foreign investors that are
(AGM) of the company. unregistered to trade in the Indian securities market
2. During liquidation, the bond holders get without having to register with any of the Indian
preference over the equity share holders. securities market regulators.

A. 1 only Q2. The term “Silver Line” recently seen in news is


B. 2 only related to :
C. Both 1 and 2
D. Neither 1 nor 2 A. The discovery of the image of the black hole.
B. The limiting layer of stratosphere beyond which
Ans - B the green house gases would not escape.
Equity Security (stock/shares): represents ownership C. A semi high-speed railway corridor.
held by shareholders (owners) in a company/ D. The migration path of elephants in the western
corporation. Holders of equity security receive ghats.
profit/dividend and capital gains (increase in prices of
shares). Equity shares entitle the holder to some control Ans – C
of the company on a proportionate basis i.e. the equity
holders get voting rights and thus some control of the SilverLine, a semi high-speed railway project that
business. During liquidation, the bond holders get envisages trains running at 200 km/h between the
preference over equity holders for payout. state’s northern and southern ends of Kerala.
The proposed 529.45-km line will link
Debt Security(bonds): represents money that is Thiruvananthapuram in the south to Kasaragod in the
borrowed and must be repaid with terms that define north, covering 11 districts through 11 stations.The
the amount borrowed,interest rate and maturity date. project entails building a semi high-speed railway

Telegram – Indian_Economy_by_Aman_Soni Page 4


corridor through the state linking its southern end and alternatively defined as a period of inflation combined
state capital Thiruvananthapuram with its northern end with a decline in the gross domestic product (GDP).
of Kasaragod.

Q3. Which among the following is/are included under


the definition of “currency in circulation” ?
Q5. Choose the INcorrect statements.
1. Currency notes with the public
2. Coins with the public 1. As per Engels Law, as household income
3. Cash in hand with banks. increases, the percentage of that income spent
4. Amount withdrawn by RBI on food declines on a relative basis.
2. A poor family would spend a relatively greater
A. 1 & 2 only proportion of expenditure on food consumption
B. 1, 2 & 3 only as compared to a rich family.
C. 2, 3 & 4 only
D. 1,2,3 & 4 only A. 1 only
B. 2 only
Ans - B C. Both 1 and 2
Currency in circulation is the total value of the currency D. Neither 1 nor 2
(coins and paper currency) that has ever been issued by
the Reserve Bank of India minus the amount that has Ans - D
been withdrawn by it. Engel's Law is a 19th century observation that as
household income increases, the percentage of that
Currency in circulation (currency with the public) income spent on food declines on a relative basis.
comprises of: This is because the amount and quality of food a family
currency notes and coins with the public can consume in a week or month is fairly limited in price
cash in hand with banks. and quantity.
It is a major liability component of a central bank’s As food consumption declines, luxury consumption and
balance sheet. savings increase in turn.

Q4. Which among the following is/are the Ernst Engel wrote, “The poorer a family, the greater the
characteristics of an economy experiencing stagflation : proportion of its total expenditure that must be
devoted to the provision of food.” This was then
1. Slow economic growth extended to whole countries by arguing the richer a
2. High Unemployment country, the smaller the food share.
3. Deflation
4. Inflation
Choose the correct option. Q1. Which among the following is/are covered under
A. 1 & 2 only the Agreement on Agriculture (AoA) of WTO :
B. 1, 2 & 3 only
C. 1, 2 & 4 only 1. Market Access
D. 1,2,3 & 4 only 2. Domestic Support
3. Export subsidies
Ans – C
A. 1 only
Stagflation is characterized by slow economic growth B. 1 & 2 only
and relatively high unemployment—or economic C. 2 & 3 only
stagnation—which is at the same time accompanied by D. 1, 2 & 3 only
rising prices (i.e. inflation). Stagflation can be
Ans - D

Telegram – Indian_Economy_by_Aman_Soni Page 5


The WTO Agriculture Agreement provides a framework Q3. Which among the following is/are the eligible
for the long-term reform of agricultural trade and purposes for receiving contribution under the Foreign
domestic policies, with the aim of leading to fairer Contribution (Regulation) Act (FCRA) :
competition and a less distorted sector.
1. Social purpose
The Agreement covers: 2. Educational purpose
 Market access — the use of trade restrictions, 3. Religious purpose
such as tariffs on imports 4. Economic purpose
 Domestic support — the use of subsidies and 5. Cultural purpose
other support programmes that directly
stimulate production and distort trade
 Export subsidies — the use of export subsidies A. 1 ,2 & 3 only
and other government support programmes B. 1, 2 & 4 only
that subsidize exports. C. 1, 2, 4 & 5 only
D. 1, 2, 3, 4 & 5 only
Q2. Choose the correct statements.
Ans - D
1. Appeals against the orders of the Dispute Foreign funding of persons in India is regulated under
Settlement Body (DSB) is taken to WTO FCRA Act and is implemented by the Ministry of Home
Appellate Body. Affairs. The Act ensures that the recipients of foreign
2. The reports adopted by the WTO’s dispute contributions adhere to the stated purpose for which
settlement body, are NOT binding on the such contribution has been obtained. Under the Act,
parties. organisations are required to register/renew
themselves every five years. Registered NGOs can
A. 1 only receive foreign contributions for five purposes — social,
B. 2 only educational, religious, economic and cultural.
C. Both 1 and 2
D. Neither 1 nor 2 Q4. Choose the correct statements wrt PM KISAN
scheme :
Ans – A
The Appellate Body, set up in 1995, is a standing 1. All land holding eligible farmer families can avail
committee of seven members that presides over the benefits under this scheme.
appeals against judgments passed in trade-related 2. The scheme also aims to supplement the
disputes brought by WTO members. Countries involved financial needs of the farmers in procuring
in a dispute over measures purported to break a WTO various inputs to ensure proper crop health and
agreement or obligation can approach the Appellate appropriate yields.
Body if they feel the report of the panel set up to
examine the issue needs to be reviewed on points of A. 1 only
law. However, existing evidence is not re-examined but B. 2 only
legal interpretations are reviewed. C. Both 1 and 2
D. Neither 1 nor 2
Appeals against the orders of DSB is taken to WTO
Appellate Body. The Appellate Body can uphold, modify, Ans – C
or reverse the legal findings of the panel that heard the
dispute. Countries on either or both sides of the dispute Under the Pradhan Mantri Kisan Samman Nidhi (PM-
can appeal. The Appellate Body has so far issued 152 KISAN) scheme, a financial benefit of ₹6,000 a year is
reports. The reports, once adopted by the WTO’s provided to the eligible farmer families, payable in three
dispute settlement body, are final and binding on the equal instalments of ₹2,000. The money is transferred
parties. directly to the bank account of the beneficiaries.

Telegram – Indian_Economy_by_Aman_Soni Page 6


The scheme aims to supplement the financial needs of as the currency issued by the central bank, as well as
the farmers in procuring various inputs to ensure proper the various bank reserves deposited with the central
crop health and appropriate yields, commensurate with bank by the government and other financial institutions.
the anticipated farm income. All land holding eligible The reserves are managed by the Reserve Bank of India
farmer families (subject to the prevalent exclusion for the Indian government and the main component is
criteria) are to avail of the benefits under this scheme, foreign currency assets. The other components are gold,
as per the cabinet decision taken during May 2019. SDR (Special Drawing Rights and RTP(Reserve Tranche
Position)
Q5. Choose the correct statements.
Q2. Choose the INcorrect statements.
1. Currently, India is the fourth largest economy in
nominal terms. 1. Devaluation is an official lowering of the value
2. India is the third-largest economy in the world of a country’s currency.
in terms of purchasing power parity (PPP). 2. Devaluation makes exports costlier and
unattractive.
A. 1 only
B. 2 only A. 1 only
C. Both 1 and 2 B. 2 only
D. Neither 1 nor 2 C. Both 1 and 2
D. Neither 1 nor 2
Ans - B
Ans - B
India is likely to overtake Japan as Asia’s second-largest Devaluation is an official lowering of the value of a
economy by 2030 when its GDP is also projected to country’s currency, by which the RBI formally sets a new
surpass that of Germany and the U.K. to rank as the fixed rate with respect to a foreign reference currency
world’s No.3, IHS Markit said. Currently, India is the .eg. 1$= 40rupees and then RBI decides that 1$=
sixth-largest economy, behind the U.S., China, Japan, 50rupees. So earlier 1$ was equal to 40 rupees but not
Germany and the U.K in nominal GDP. now 1$ is equal to 50rupees. Now 1$ can buy more
rupees as compared to earlier, so rupees have become
India is the third-largest economy in the world in terms cheaper or rupees have lost its value as compared to
of purchasing power parity (PPP), behind the US and dollar or rupee has been devalued.
China. Devaluation makes exports cheaper or makes them
more competitive.
Q1. Which among the following is/are part of Forex
Reserves maintained by the RBI : Q3. Choose the INcorrect statements.
1. Foreign Currency
2. Special Drawing Rights 1. India follows the Managed floating exchange
3. Reserve Tranche rate system.
4. Gold 2. Under this system, during extreme fluctuations,
the central bank will intervene in the foreign
exchange market to control the volatility.
A. 1 & 2 only
B. 1, 2 & 4 only A. 1 only
C. 2, 3 & 4 only B. 2 only
D. 1,2,3 & 4 only C. Both 1 and 2
D. Neither 1 nor 2
Ans - D
Forex/Foreign Exchange Reserve. It is money or other Ans – D
assets held by a central bank or other monetary Managed Floating Exchange Regime
authority so that it can pay its liabilities if needed, such

Telegram – Indian_Economy_by_Aman_Soni Page 7


India is having this type of exchange rate system. In this A. 1 only
hybrid exchange rate system, the exchange rate is B. 2 only
basically determined in the foreign exchange market C. Both 1 and 2
through the operation of market forces. D. Neither 1 nor 2
Market forces mean the selling and buying activities by
various individuals and institutions. So far, the managed Ans – C
floating exchange rate system is similar to the flexible
exchange rate system. Voluntary Retention Route(VRR) is a new channel of
But during extreme fluctuations, the central bank under investment available to FPIs to encourage them to
a managed floating exchange rate system (like the RBI) invest in debt markets in India over and above their
intervenes in the foreign exchange market. investments through the regular route. The objective is
Objective of this intervention is to minimise the to attract long-term and stable FPI investments into
fluctuation in the exchange rate of rupee RBI interferes debt markets while providing FPIs with operational
to manage volatility in the floating exchange regime. flexibility to manage their investments. FPIs investing
through this route will be eligible to participate in repos
Q4. Choose the correct statements. for their cash management, provided that the amount
borrowed or lent under repo were not to exceed 10
1. FDI is very speculative and highly volatile. percent of the investment under VRR. They will also be
2. As compared to FDI, FPI may lead to more eligible to participate in any currency or interest rate
volatility in capital flight and currency crisis. derivative instrument, OTC or exchange-traded
instrument to manage their interest rate risk or
currency risk.
A. 1 only
B. 2 only Q1. Which one of the following is/are included under
C. Both 1 and 2 the External commercial Borrowings (ECBs):
D. Neither 1 nor 2
1. Suppliers' credit
Ans – B 2. Commercial borrowings from the private sector
3. Fixed rate bonds
If FDI is certain, long term and less fluctuating, FPI is 4. Commercial bank loans
speculative, highly volatile and non predictive.
FPI is investment aimed at getting profits from shares,
interests from deposits etc. It is also known as hot A. 1, 2 & 3 only
money. The portfolio investors keep their money in the B. 1, 2 & 4 only
capital market only for a short period of time. Its C. 2, 3 & 4 only
destination period is so small and is empirically D. 1,2,3 & 4 only
considered as fluctuating (often short term) capital. It is
highly volatile, a fair weather friend, speculative, Ans - D
involves exchange risks and may lead to capital flight External commercial Borrowings (ECBs): loans in India
and currency crisis affecting real economic variables. made by non-resident lenders in foreign currency to
Indian borrowers. They are used widely in India to
Q5. Choose the correct statements wrt Voluntary facilitate access to foreign money by Indian
Retention Route(VRR). corporations and PSUs (public sector undertakings).

1. It is aimed at attracting long-term and stable FPI ECBs include commercial bank loans, buyers' credit,
investments into debt markets in India. suppliers' credit, securitized instruments such as
2. FPIs investing through this route will be eligible floating rate notes and fixed rate bonds etc., credit from
to participate in repos for their cash official export credit agencies and commercial
management with some conditions. borrowings from the private sector window of

Telegram – Indian_Economy_by_Aman_Soni Page 8


multilateral financial Institutions such as International • It mandates inspection of bank documents and
Finance Corporation (Washington), ADB, AFIC, CDC, etc. records by ECGC officials for losses exceeding Rs.10
ECBs cannot be used for investment in the stock market crore as against the present Rs 1crore.
or speculation in real estate. • The banks shall pay a premium to ECGC monthly on
the principal and interest as the cover is offered for
Q2. Which one of the following is/are Prohibited Sectors both outstandings.
for FDI in India :
1. Gambling or Betting businesses Q4. Choose the correct statements.
2. Railways
3. Atomic Energy 1. Foreign Sovereign bonds would make the nation
4. Tobacco industry reliant on foreign inflows and vulnerable to
global uncertainties.
A. 1 & 2 only 2. Foreign Sovereign bonds are vulnerable to
B. 1, 2 & 4 only Currency risks.
C. 1, 3 & 4 only
D. 1,2,3 & 4 only
A. 1 only
Ans - C B. 2 only
Sectors in the Indian economy where FDI is not allowed C. Both 1 and 2
are: D. Neither 1 nor 2
●Atomic Energy Genera on
●Cigars, Cigare es, or any related tobacco industry Ans - C
●Lo eries (online, private, government, etc) When sovereign bonds are denominated in foreign
●Investment in Chit Funds, Nidhi company. currency and can be settled in foreign markets, it is
●Any Gambling or Be ng businesses generally referred to as Foreign sovereign bonds.
It makes the nation reliant on foreign inflows and
Q3. Consider the following statements wrt ‘NIRVIK’ vulnerable to global uncertainties.
scheme. Moreover, if the government is unable to return the
dues in time, it won’t be able to print dollars and repay.
1. The scheme aims to enhance loan availability But in case of domestic borrowings, if the government is
for exporters. unable to return the dues, it can ask RBI to print fresh
2. The scheme has been introduced by RBI. currency (rupees).

Choose the correct statements. Q5. Arrange the following in the descending order of
A. 1 only recipient of remittances from foreign nations :
B. 2 only 1. India
C. Both 1 and 2 2. China
D. Neither 1 nor 2 3. Egypt
4. Philippines
Ans – A 5. Mexico

Export Credit Guarantee Corporation of India (ECGC) Choose the correct option.
has introduced ‘NIRVIK’ scheme to ease the A. 1, 2, 3, 4, 5
lending process and enhance loan availability for B. 2, 1, 4, 5, 3
exporters. C. 1, 2, 5, 3, 4
D. 1, 2, 5, 4, 3
Key features :
• Insurance cover guaranteed will cover up to 90 Ans - D
percent of the principal and interest. India, the world’s largest recipient of remittances,
received $87 billion in 2021 with the United States

Telegram – Indian_Economy_by_Aman_Soni Page 9


being the biggest source, accounting for over 20 per Q3. Choose the correct statements.
cent of these funds, according to the World Bank. India
is followed by China, Mexico, the Philippines, and Egypt. 1. WTO strictly does not allow any departures
from the Most Favored Nation (MFN) principle.
Q1. Choose the correct statements. 2. Under Generalized system of preferences (GSP),
1. The General Council of the WTO itself functions Imports from low income countries are allowed
as the Dispute Settlement Body (DSB). at lower or no tariffs.
2. All the decisions of Dispute Settlement
Appellate body(DSAB) are final and binding on A. 1 only
the member. B. 2 only
C. Both 1 and 2
A. 1 only D. Neither 1 nor 2
B. 2 only
C. Both 1 and 2 Ans - B
D. Neither 1 nor 2 WTO allows departures from the MFN principle
1. Imports from poor countries are allowed at lower /
Ans - A zero tariffs (generalized system of preferences (GSP)).
There is no separate Dispute Settlement Body (DSB) but 2. Preferential and free trade arrangement among
the General Council which is the second highest body in countries of a region and others are allowed at
the organization works as the DSB while giving verdict concessional and free rates respectively.
on the trade dispute. DSB conclusions can be challenged
in the Dispute Settlement Appellate body(DSAB). Q4. Which among the following is/are provisions
allowed under the WTO’s Special and Differential
After the ruling, the erring nation is directed to make Treatment :
changes in its laws to make them WTO compliant within
a reasonable time. If the ‘erring country’ does not take 1. Proportionately lower tariff reductions
measures to correct its laws, the complaining country is 2. Working out a tariff reduction formula
allowed to take cross retaliatory measures. The decision 3. Shorter implementation periods
of DSAB is not binding on the member countries. 4. Additional benefits for developed countries

Q2. The term “Most Favored Nation” recently seen in A. 1 & 2 only
news is related to : B. 1, 2 & 4 only
C. 2, 3 & 4 only
A. United Nations Security Council D. 1,2,3 & 4 only
B. World Trade Organisation
C. World Bank Ans - A
D. International Monetary Fund
The principle of special and differential treatment was
Ans – B bought for addressing the concerns of developing
countries. It is a flexibility given to developing countries
Most Favored Nation in implementing WTO commitments.
The principle of the MFN treatment means that the It is incorporated through a number of provisions in the
tariff policy that one country receives in an organization area of market access including.
should be extended to all others.According to the MFN (1)Proportionately lower tariff reductions
principle of the WTO’s General Agreement on Tariffs (2) Longer implementation periods
and Trade (GATT) — to which India is a (3) Working out a tariff reduction formula after taking
signatory/contracting party — each of the WTO into account the different tariff structures of developed
member countries should “treat all the other members and developing countries
equally as ‘most-favored’ trading partners.”
Q5. Choose the correct statements.

Telegram – Indian_Economy_by_Aman_Soni Page 10


These sanitary and phytosanitary measures can take
1. Countervailing Duty is imposed when the goods many forms, such as requiring products to come from a
are imported below the fair market value. disease-free area,setting of allowable maximum levels
2. Anti- Dumping Duty is imposed to negate the of pesticide residues or permitted use of only certain
effect of unfair subsidies provided by a foreign additives in food.
government, to its producers.
The basic aim of the SPS Agreement is to maintain the
A. 1 only sovereign right of any government to provide the level
B. 2 only of health protection it deems appropriate, but to ensure
C. Both 1 and 2 that these sovereign rights are not misused for
D. Neither 1 nor 2 protectionist purposes and do not result in unnecessary
barriers to international trade.
Ans - D
Anti- Dumping Duty is a special import duty imposed by Q2. Choose the correct statements.
the importing countries when a firm ( following an
enquiry) is found to have sold a product in the 1. Under the WTO norms, there is no limit on the
importing market at a price below the one it charges in Green box subsidies
the home market or below the cost of production or at 2. Green box subsidies are direct payments to the
less than fair value and it harms the producers in the farmers to limit their production and are trade
importing country. distorting

Countervailing Duty is imposed when on a detailed


enquiry it is found out that the exporting country is A. 1 only
giving subsidies to harm the industry of another B. 2 only
country. In this case the importing country can impose C. Both 1 and 2
higher than normal duties to protect its domestic D. Neither 1 nor 2
economy, which are called countervailing duties.
Ans - A
Q1. Choose the correct statements wrt Sanitary and Green box subsidies relate to research and
Phytosanitary Measures (SPS) development and infrastructure like universities, pest
control measures, roads in rural areas, direct payment
1. SPS apply only to Plant based products and to farmers with decoupling effect (i.e. payment to the
diseases farmers will be made but no condition will be attached
2. The SPS Agreement is aimed at maintaining the on increasing or decreasing the production) etc. Since
sovereign right of nation to protect the health they do not distort trade there is no limit on them.
of its Population
Blue Box contains subsidies that are direct payments to
A. 1 only the farmers to limit their production.
B. 2 only
C. Both 1 and 2 Amber box subsidies are also called Aggregate Measure
D. Neither 1 nor 2 of Support (AMS). The AMS means annual level of
support (subsidies) expressed in monetary terms.
Ans - B Example of product specific subsidies is MSP and for
Sanitary and Phytosanitary Measures (SPS) non-product specific subsidy is fertilizer,power etc.
Sanitary (human and animal health) and phytosanitary As per the WTO norms, the AMS can be given up to 10
(plant health) measures apply to domestically produced % of a country’s agricultural GDP in the case of
food and local animal and plant diseases, as well as to developing countries. On the other hand, the limit is 5%
products coming from other countries. for a developed economy. This limit is called de
minimus level of support. The Aggregate Measurement

Telegram – Indian_Economy_by_Aman_Soni Page 11


of Support consists of two parts—product-specific private rights and protection is enforced through court
subsidies and non-product specific subsidies. orders. Trademark is typically a name, word, phrase,
logo, symbol, design, image or a combination of these
Q3. The term “Singapore Issues” often seen in news is elements.
related to :

A. Arbitration in Singapore between Govt of India Q5. Which among the following are correctly matched
and Crain energy India Ltd
B. Working groups under WTO 1. Kandhamal Haladi - Odisha
C. Agreement on Eradication of Ebola 2. Idu Mishmi Textiles - Mizoram
D. White paper on Combating Money Laundering 3. Srivilliputtur Palkova - Kerala

Ans - B A. 1 only
Singapore Issues B. 1 and 3 only
The first ministerial conference was held in Singapore in C. 2 only
1996. Ministers from WTO member countries decided D. None of the above
at the 1996 Singapore Ministerial Conference to set up Ans – A
three new working groups: on trade and investment, on
competition policy, and on transparency in government Kandhamal Haladi - Odisha
procurement. They also instructed the WTO General Pawndum - Mizoram
Council to look at possible ways of simplifying trade Ngotekherh - Mizoram
procedures, an issue sometimes known as “trade Idu Mishmi Textiles - Arunachal Pradesh
facilitation”. Kandangi Saree - Tamil Nadu
Srivilliputtur Palkova - Tamil Nadu
Kaji Nemu - Assam
Q4. Choose the correct statements.
Q1. Which among the following is/are Benefits to the
1. A Trademark protects the ‘expression of ideas’ government from Asset Monetisation :
like an artistic work
2. Copyright is a distinctive sign/word/phrase/logo 1. Revenue Stream
which is used to distinguish the products or 2. Productive utilization of assets
services of different businesses. 3. Development of infrastructure
4. Encourage private investment
A. 1 only
B. 2 only
C. Both 1 and 2 A. 1 & 2 only
D. Neither 1 nor 2 B. 1, 2 & 4 only
C. 2, 3 & 4 only
Ans - D D. 1,2,3 & 4 only
Copyright: Copyrights protect the ‘expression of ideas’.
Artistic works are generally considered to be expression Ans - D
of ideas. Books, paintings, songs, movies, computer Benefits to government from Asset Monetisation :
programs are given copyright. Generally the term of  Revenue to government
copyright is 60 years from the death of author.  Enable productive utilisation of under-utilised
assets and unutilized assets
Trademark: It is a distinctive sign which is used to  Encourage private investment and participation
distinguish the products or services of different in the economic development
businesses. The duration of trademark registration can  Boost local economy
vary but is usually 10 years. It can be renewed  Employment generation
indefinitely on payment of fees. Trademark rights are  Development of infrastructure
Telegram – Indian_Economy_by_Aman_Soni Page 12
 Decline of forex reserve
Q2. Choose the correct statements.  Stagflation

1. Unlike the paper currency, the Central Bank Q4. It is a forex tool, whereby the central bank uses its
Digital Currency (CBDC) is not backed by the currency to buy another currency or vice versa. The
promise of the Central Bank. term is :
2. The maintenance cost of issue of the currency is
lower wrt CBDC, over paper notes A. Global Depositary Receipt
B. Foreign Exchange Conversion
A. 1 only C. Foreign Currency Swap
B. 2 only D. Masala Bonds
C. Both 1 and 2
D. Neither 1 nor 2 Ans - C
A foreign currency swap, also known as an FX swap, is
Ans - B an agreement to exchange currency between two
A Central Bank Digital Currency (CBDC), or national foreign parties. The agreement consists of swapping
digital currency, is simply the digital form of a country’s principal and interest payments on a loan made in one
fiat currency. Instead of printing paper currency or currency for principal and interest payments of a loan of
minting coins, the central bank issues electronic tokens. equal value in another currency. One party borrows
This token value is backed by the full faith and credit of currency from a second party as it simultaneously lends
the government. CBDC is a legal tender issued by a another currency to that party.
central bank in a digital form.
Benefits : It is a forex tool whereby the central bank uses its
 The cost of printing, transporting and storing currency to buy another currency or vice versa. In a
paper currency can be substantially reduced. Dollar–Rupee buy/sell swap, the central bank buys
 Financial inclusion. dollars (US dollars or USD) from banks in exchange for
 Better targeting of beneficiary of cash subsidy Indian Rupees (INR) and immediately gets into an
schemes. opposite deal with banks promising to sell dollars at a
later date. In a dollar–rupee sell/buy swap it sells USD in
Q3. The Global crude oil price volatility would possibly exchange for INR and promises to buy dollars from
affect the Indian economy in the following ways : banks after some years.

1. Imported inflation Q5. Which one of the following is/are identified as


2. Depreciation of rupee thematic areas under the NITI Aayog Aspirational
3. Outflow of Capital Districts’ Programme :
4. Stagflation
1. Financial Inclusion
2. Health & Nutrition
A. 1, 2 & 3 only 3. Law and Order
B. 1, 2 & 4 only 4. Agriculture
C. 2, 3 & 4 only
D. 1, 2, 3 & 4 only A. 1, 2 & 3 only
B. 1, 2 & 4 only
Ans - D C. 2, 3 & 4 only
Impacts : D. 1, 2, 3 & 4 only
 Imports become more expensive leading to
current account deficit Ans - B
 Imported inflation The Aspirational Districts’ Programme aims at overall
 Capital outflows improvement in human development across 115 most
 Depreciation of domestic currency backward districts of India. The 115 districts were

Telegram – Indian_Economy_by_Aman_Soni Page 13


identified from 28 states, at least one from each state. 2. Economic growth
Aspirational Districts are those districts in India, that are 3. Inflation
affected by poor socio-economic indicators. The 4. Recession in other countries
improvement in these districts can lead to the overall
improvement in human development in India. At the A. 1 & 2 only
Government of India level, the programme is anchored B. 1, 2 & 3 only
by NITI Aayog. In addition, individual Ministries have C. 2, 3 & 4 only
assumed responsibility to drive the progress of districts. D. 1, 2, 3 & 4 only
Ans - D
The program is based on 49 indicators from the 5 Factors which affect the current account deficit/surplus
identified thematic areas, which focuses closely on of a country :
improving people’s  Exchange rate
 Health & Nutrition,  Economic growth
 Education,  Saving rates
 Agriculture & Water Resources,  Inflation
 Financial Inclusion & Skill Development, and  Recession in other countries
 Basic Infrastructure.
Q3. Consider the following statements.
Q1. Which among the following is/ are included under 1. India is the second largest producer of coal in
the Current Ac (import and export) of a country : the world
2. India imported most of its coal from Australia
1. The balance of trade in goods
2. The balance of trade in services. Choose the INcorrect statements.
3. Net current income A. 1 only
4. Transfer payments B. 2 only
C. Both 1 and 2
D. Neither 1 nor 2
A. 1 & 2 only
B. 1, 2 & 4 only Ans - B
C. 2, 3 & 4 only India is the second largest producer of coal in the world,
D. 1, 2, 3 & 4 only after China. India has the fifth largest coal reserves in
the world.
Ans - D India does not have enough reserves of good quality
Current Ac shows export and import of visibles (also coal especially coking coal that is used as a raw material
called merchandise or goods – represent trade balance) in steel making and allied industries. Most of it is
and invisibles (also called non-merchandise). Invisibles imported from Indonesia, South Africa, Russia and
include services, transfers and income. Australia.
It includes : Coal can be freely imported (under Open General
 The balance of trade in goods License) by the consumers themselves considering their
 The balance of trade in services. needs based on their commercial prudence.
 Net current income
 Transfer payments Distribution of coal imported into India in 2019, by
The balance of exports and imports of goods is referred country of origin:
to as the trade balance. Trade Balance is a part of Indonesia – 49%
‘Current Account Balance’. Australia- 20%
South Africa- 16%
Q2. Which among the following is/ are factors which USA- 5%
affect the current account deficit/surplus of a country : Russia- 3%
1. Exchange rate Others- 8%

Telegram – Indian_Economy_by_Aman_Soni Page 14


Q4. Consider the following statements. function of UPI, available on smartphones, is not there
1. India is the largest consumer of fertilizer in the on UPI123Pay.
world. The users will be able to undertake a host of
2. Out of the fertilizers consumed, Urea transactions based on four technology alternatives
consumption occupies the major share. including-
Choose the correct statements.  App-based Functionality
A. 1 only  Missed Call
B. 2 only  Interactive Voice Response (IVR)
C. Both 1 and 2  Proximity Sound-based Payments
D. Neither 1 nor 2
Even though UPI can alternatively be accessed through
Ans - B the National Unified USSD Platform using the code
India is the 3rd largest producer of fertilizer after China *99#, the process has not seen an uptick in adoption or
& the US. India is the 2nd largest consumer of fertilizer popularity. However, with UPI123Pay, it would reach 40
after china. India also ranks 2nd in the production of crore feature phone users in the country, potentially
nitrogenous fertilizers and 3rd in phosphatic fertilizers. making a material improvement in the way they access
the popular payments platform.
Potash requirement is met through imports since we
have limited reserves of potash. India depends heavily
on imports for meeting its fertilizer raw materials Q1. Choose the INcorrect statements.
(natural gas, sulphur and rock phosphate),
intermediates (ammonia and sulphuric and phosphoric 1. A member can keep certain amount of its
acids), and finished products (diammonium phosphate, contribution to IMF with itself, in the form of
potash and complex fertilizers) requirements. Reserve Tranche Position
2. All the members of the IMF get equal voting
India’s fertilizer consumption in FY20 was about 61 rights
million tonnes — of which 55% was urea. Since non-
urea (MoP, DAP, complex) varieties cost higher, many A. 1 only
farmers prefer to use more urea than actually needed. B. 2 only
C. Both 1 and 2
Q5. Choose the correct statements. D. Neither 1 nor 2

1. RBI’s UPI123Pay is a scan-and-pay function of Ans - B


UPI, launched for the feature phones. RTP (Reserve Tranche Position) :Of all the money to be
2. Currently, there is no way the feature phone given to the IMF by a country, it can keep 25% of it with
users can access the UPI and carryon itself in its forex reserve. It is called RTP.
transactions.
The quota largely determines a member’s voting power
A. 1 only in IMF decisions. Each IMF member’s votes are
B. 2 only comprised of basic votes plus one additional vote for
C. Both 1 and 2 each 1lakh SDR of quota. The amount of financing a
D. Neither 1 nor 2 member can obtain from the IMF (its access limit) is
based on its quota.
Ans - D
The unified payments interface (UPI) service, which was Q2. Choose the correct statements.
limited to smartphones to date, will be now available 1. IMF is contractually bound to repay a member,
for feature phones without internet. Reserve Bank of if the member exercises its claim of SDRs
India’s (RBI) UPI123Pay is a three-step method to (Special Drawing Rights) on the IMF.
initiate and execute services for users. The scan-and-pay 2. Members of IMF can exchange SDR’s and
currencies among themselves.
Telegram – Indian_Economy_by_Aman_Soni Page 15
1. The International Finance Corporation (IFC)
A. 1 only promotes private sector investment in
B. 2 only developing countries as a way to reduce
C. Both 1 and 2 poverty and improve people’s life.
D. Neither 1 nor 2 2. MIGA promotes foreign direct investment into
developing countries by insuring investors
Ans - B against political risk, advising governments on
The SDRs (Special Drawing Rights) was created by the attracting investment
IMF in 1969 as a supplementary forex reserve asset. The
international supply of two key reserve assets—gold A. 1 only
and the U.S. dollar—proved inadequate for supporting B. 2 only
the expansion of world trade and financial flows that C. Both 1 and 2
was taking place.Therefore, the international D. Neither 1 nor 2
community decided to create a new international
reserve asset under the auspices of the IMF. Ans - D
The SDR is neither a currency, nor a claim on the IMF. The International Finance Corporation (IFC) promotes
Rather, it is a potential claim on the freely usable private sector investment in developing countries as a
currencies of IMF members. Holders of SDRs can obtain way to reduce poverty and improve people’s life.
these currencies in exchange for their SDRs in two ways: Established in 1956, IFC is the largest multilateral source
first, through the arrangement of voluntary exchanges of loan and equity financing for private sector projects
between members; and second, by the IMF designating in the developing world.
members with strong external positions to purchase
SDRs from members with weak external positions. The Multilateral Investment Guarantee Agency (MIGA)is
a member of the World Bank group. It was established
Q3. Which among the following is/are NOT a part of to promote foreign direct investment into developing
basket of currencies that determine the value of SDR countries. MIGA was founded in 1988 and is
under IMF : headquartered in Washington.
MIGA promotes foreign direct investment into
1. US Dollar developing countries by insuring investors against
2. Russian Ruble political risk, advising governments on attracting
3. South Korean Won investment etc
4. Japanese Yen
Q5. Choose the correct statements.

A. 2 only 1. The members of the Asian Infrastructure


B. 3 only Investment Bank (AIIB) are from BRICS nations
C. 1 and 4 only only.
D. 2 and 3 only 2. China is the largest shareholder of the Asian
Infrastructure Investment Bank (AIIB).
Ans - D
Baskets of currencies determine the value of SDR.The A. 1 only
value of the SDR was initially defined as equivalent to B. 2 only
0.88 grams of fine gold—which, at the time, was also C. Both 1 and 2
equivalent to one U.S. dollar. Therefore, SDR is also D. Neither 1 nor 2
called Paper Gold. The SDR was redefined as a basket of
currencies in 1973.The SDR basket consists of the U.S. Ans - B
dollar, euro, the Chinese renminbi (Yuan), Japanese yen, Asian Infrastructure Investment Bank (AIIB) is a
and pound sterling. multilateral development bank that aims to support the
building of infrastructure in the Asia-Pacific region. The
Q4. Choose the INcorrect statements. total number of countries approved for membership of

Telegram – Indian_Economy_by_Aman_Soni Page 16


AIIB is 103 (Regional Members: 45, Non-Regional After Independence in 1947, India launched the five
Members: 37, Prospective Members:21). The AIIB took years plan for rapid growth.Planning has the following
shape with 50 members, including Australia, India, long term goals:
Russia and the United Kingdom. 1.Economic Growth
2.Modernization
There are 57 founding-members. China is the largest 3.Self reliance and
shareholder (at 30.34 per cent), followed by India (8.52 4.Social justice
per cent) and Russia (6.66 per cent). Though one among
the Asian giants , Japan has chosen to stay out of the Q3. Which among the following are correctly matched :
Beijing-initiated AIIB.
1. First Plan (1951 - 56) – Heavy Industry
Q1. Which among the following are NOT correctly 2. Second Plan (1956 - 61) - Agriculture
matched : 3. Third Plan (1961 - 66) – Devaluation of Rupee

1. The People’s Plan - M.N.Roy A. 1 and 2 only


2. Gandhian Plan – M K Gandhi B. 3 only
3. Bombay Plan - JRD Tata, GD Birla & others C. All of the above
4. The Sarvodaya Plan – B R Ambedkar D. None of the above

A. 1 only Ans - B
B. 1 and 3 only First Plan (1951 - 56) – Agriculture
C. 2 and 4 only Second Plan (1956 - 61) - Heavy industry
D. None of the above Third Plan (1961 - 66) – Devaluation of Rupee

Ans – C Q4. Choose the correct statements.

The People’s Plan - M.N.Roy 1. Nirmala Sitharaman is the present Chairperson


Gandhian Plan - Shriman Narayan Agarwal of the Governing Council of the NITI Aayog
Bombay Plan - JRD Tata, GD Birla & others 2. The President of India appoints the CEO of the
The Sarvodaya Plan - Jayaprakash Narayan NITI Aayog.

Q2. India started the Five year plans to achieve the A. 1 only
following goals : B. 2 only
C. Both 1 and 2
1. Economic Growth D. Neither 1 nor 2
2. Modernization
3. Self reliance Ans - D
4. Social justice The NITI Aayog will comprise the following:
 Prime Minister of India is the Chairperson of the
Choose the correct option. Governing Council consisting of the Chief
Ministers of all the States and Lt. Governors of
A. 1, 2 & 3 only Union Territories in India.
B. 1, 2 & 4 only  Regional Councils will be created to address
C. 2, 3 & 4 only particular issues and possibilities affecting more
D. 1, 2, 3 & 4 only than one state. These will be formed for a fixed
term. It will be summoned by the Prime
Ans – D Minister. It will consist of the Chief Ministers of
States and Lt. Governors of Union Territories.
These will be chaired by the Chairperson of the
NITI Aayog or his nominee.

Telegram – Indian_Economy_by_Aman_Soni Page 17


 Special invitees:Eminent experts, specialists
with relevant domain knowledge, which will be A. 1 only
nominated by the Prime Minister. B. 2 only
 Ex Officio members: Maximum of 4 members of C. Both 1 and 2
the Council of Ministers which are to be D. Neither 1 nor 2
nominated by the Prime Minister.
 Chief Executive Officer: CEO will be appointed Ans – A
by the Prime Minister for a fixed tenure. He will
be of the rank of Secretary to the The difference between planned economy and
Governmentof India. command economy is that in the former there may be
mixed economy and while in the latter Government
Q5. Choose the correct statements. owns and regulates economy to near monopolistic limit.

1. As a statutory body, the suggestions of NITI Command economies were set up in China and USSR,
Aayog are binding on the government. mainly for economic growth and social and economic
2. NITI Aayog promotes cooperative federalism in justice but have been dismantled in the last two
India. decades as they do not create wealth sustainably and
are not conducive for innovation and efficiency. Cuba
A. 1 only and North Korea are still command economies.
B. 2 only
C. Both 1 and 2 Q2. Which among the following is/are NOT a
D. Neither 1 nor 2 component of the Sarvodaya Plan :

Ans - B 1. Cottage industries


National Institution for Transforming India, better 2. Self-reliance
known as NITI Aayog, was formed via a resolution of the 3. Centralized planning
Union Cabinet. 4. Rapid Industrialization
NITI Aayog is the premier policy think tank of the
Government of India, providing directional and policy A. 1 and 2 only
inputs. Apart from designing strategic and long-term B. 3 and 4 only
policies and programmes for the Government of India, C. 1, 2 and 3 only
NITI Aayog also provides relevant technical advice to the D. None of the above
Centre, States, and Union Territories.
Ans - B
NITI Aayog has been constituted to actualise the The Sarvodaya Plan was published in January 1950. The
important goal of cooperative federalism and to enable plan drew its major inspirations from the Gandhian
good governance in India. On the premise that strong techniques of constructive works by the community and
states make a strong nation, NITI Aayog acts as the trusteeship as well as the Sarvodaya concept of Acharya
quintessential platform for the Government of India by Vinoba Bhave, the eminent Gandhian constructive
bringing States together as ‘Team India’ to work worker.
towards the national development agenda.
Major ideas of the plan were highly similar to the
Q1. Choose the correct statements. Gandhian Plan like emphasis on agriculture, agri based
small and cottage industries, self-reliance and almost no
1. Under a Command Economy, the Government dependence on foreign capital and technology, land
completely owns and regulates the economy to reforms, self-dependent villages and decentralised
a near monopolistic limit. participatory form of planning and economic progress
2. Command economies act as a hub for etc.
innovation and efficiency than the Capitalist
Economies.
Telegram – Indian_Economy_by_Aman_Soni Page 18
Q3.The ‘Target group’ approach of Policy involve citizens in crowd sourcing ideas to address
implementation is related to : challenges facing India’s development.

A. Resolving NPA’s in the Banking Sector In Phase I of the Grand Innovation Challenge, NITI Aayog
B. Socio-economic development of a Vulnerable Rural will seek the views of the citizens on the key challenges
Community facing India, across areas significant for the country’s
C. Dealing with Left-wing Extremism in the North East development. The idea is to find out from the people
India. what are the critical issues which need to be addressed
D. Re-enrollment of school drop out students to develop the social sector and the challenges which
need to be tackled on a priority basis.
Ans - B
Sixth Plan(1980-85) was launched with the slogan of In Phase II, a shortlist of the urgent challenges as
‘GaribiHatao’ (alleviate poverty). Already, a programme suggested by citizens would be prepared and innovative
(the TPP) was tested and tried by the same government solutions would be sought from the people to address
in the Fifth Plan which tried to improve the standard of them using appropriate technology. The idea is to
living of the poor masses with the ‘direct approach’ (the encourage innovation, entrepreneurship & citizen-led
idea of poverty alleviation, but such a slogan of solutions to problems through this Grand Challenge.
‘GaribiHatao’ was not given to the programme).
Q5 . Choose the INcorrect statements.
Some of the major issues addressed by the Plan were—
emphasis on socio economic infrastructure in rural 1. Atal Innovation Mission (AIM) is aimed at
areas; eliminating rural poverty and reducing regional developing new programmes and policies for
disparities through the IRDP(Integrated Rural fostering innovation in different sectors of the
Development Program)(1979); ‘target group’ approach economy
initiated; a number of national level programmes and 2. Atal Innovation Mission (AIM) is launched by
schemes were launched during the plan, which tried to the DRDO.
attend to the specific areas and the specific concerns of
socio-economic development (this is the ‘target group’ A. 1 only
approach). B. 2 only
C. Both 1 and 2
Q4. The ‘Grand Innovation Challenge’ was launched by : D. Neither 1 nor 2

A. Planning Commission Ans – B

B. Ministry of Electronics and Information Atal Innovation Mission (AIM) is Government of India’s
Technology flagship initiative to promote a culture of innovation
C. ISRO and entrepreneurship in the country.
D. NITI Aayog Launched by NITI aayog.
Objectives:
Ans - D  To develop new programmes and policies for
NITI Aayog launched the first phase of the ‘Grand fostering innovation in different sectors of the
Innovation Challenge’ to seek citizens inputs on the key economy,
developmental challenges facing India. The ‘Grand  To provide platform and collaboration
Innovation Challenge’ is being launched on the MyGov opportunities for different stakeholders,
portal, to involve citizens at the very first stage in  To create awareness and create an umbrella
innovating for India’s development. The idea is to work structure to oversee the innovation ecosystem
together with the States and every citizen as Team India of the country.
to ensure progress. leaving no one behind. The focus is
on the social sector, the most vulnerable sections and to

Telegram – Indian_Economy_by_Aman_Soni Page 19


Q1. Which one of the following is/are the measures holding after acquisition of the land of the Zamindar
taken in India post Independence to bring reforms in couldn’t exceed the ceiling prescribed by each state.
the Land Ownership :
Q3. Which among the following is/are the effects of
1. Zamindari Abolition Land reforms in India :
2. Security of tenure to tenants
3. Absolute Inheritance rights to the zamindars on 1. Increase in agriculture production
all lands 2. Reduction in Poverty
4. Ceiling on size of land holding 3. Increase in investment in agriculture
4. Nudged rural India towards an egalitarian
A. 1, 2 & 3 only society
B. 1, 2 & 4 only
C. 2, 3 & 4 only A. 1, 2 & 3 only
D. 1, 2, 3 & 4 only B. 1, 2 & 4 only
C. 2, 3 & 4 only
Ans - B D. 1, 2, 3 & 4 only
The major land reforms introduced in India after
independence: Ans - D
(i) Abolition of intermediaries i.e. Zamindari Abolition Effects of Zamindari abolition/Land reforms
(ii) Tenancy reforms
(iii) Providing security of tenure to tenants (1) Agricultural production increased
(iv) Decrease in rents (2) Improvement in the conditions of the tenants
(v) Giving ownership rights to the tenants (3) No land revenue was to be paid
(vi) Ceiling on size of land holding (4)Changed rural power structure towards egalitarian
society.
(5) Rise of the middle class.
Q2. Which one of the following is NOT the Objective of (6)Increase in investment in agriculture
the Tenancy Reforms in India : (7) Decrease in poverty
A. Security of tenure to the tenants
B. Zamindar to be deprived of all the land holdings Q4. Which one of the following is/are the Socio-
C. Reduce the rents paid by tenants economic Impact of Green Revolution in India :
D. Ownership rights of land to the Tenant
1. Increased the inequality among farmers
Ans - B 2. Increased the income of the farmers
Objectives of tenancy reforms:- 3. Food production increased
4. Reduced the Inter state migration
(1)Guarantee of security of tenure to the tenants who
had cultivated a piece of land for a fixed number of
years (number of years varied from state to state). A. 1, 2 & 3 only
(2)Reduce the rents paid by tenants to a fair and just B. 1, 2 & 4 only
level. C. 2, 3 & 4 only
(3) The tenant should gain the right to acquire D. 1, 2, 3 & 4 only
ownership of land he had cultivated subject to certain
restrictions. Ans - A
Socio- economic Impact
The aim of zamindari abolition and tenancy reforms was  Food production increased.(wheat in 1960s and
same: land to the tiller. rice, by 1970s). India not only became self –
Again the tenants’ right to acquire the landowners land sufficient in food production but later turned
was restricted by the condition that the land owner into a food exporting country.
wasn’t to be deprived of all his land. Also the tenants

Telegram – Indian_Economy_by_Aman_Soni Page 20


 It increased the income of the farmers and The FRBM act provided for certain documents to be
increased their standard of living. tabled in the Parliament of India, along with Budget,
 It increased the inequality between the large annually with regards to the country’s fiscal policy. This
farmers (who benefitted more) and the small includes :
farm- ers. It also increased the inequality among
the states too  Medium-term Fiscal Policy Statement,
 It caused migration from these poor states to  Fiscal Policy Strategy Statement,
the north western states.  Macro-economic Framework Statement, and
 Medium-term Expenditure Framework
Q5. Which among the following is the favorable Statement.
environmental impact caused due to the green
Revolution : The FRBM rules mandate four fiscal indicators to be
projected in the medium-term fiscal policy statement.
1. Increase in Soil fertility These are:
2. Increase in fertilizer Leaching • revenue deficit as a percentage of GDP
3. Eutrophication • fiscal deficit as a percentage of GDP.
4. Increase in Water Table. • tax revenue as a percentage of GDP.
• total outstanding liabilities as a percentage
A. 1 and 4 only of GDP
B. 2 and 3 only
C. All of the above Q2. Choose the correct statements.
D. None of the above 1. The Negotiable Warehouse Receipt (NWR)
System in India is implemented by Food
Ans - D Corporation of India.
The most devastating negative impact of the Green
Revolution was the ecological one. Impacts : 2. Farmers can avail loans from banks against the
(a) Soil degradation and decrease in fertility warehouse receipts issued to them against their
(b) Water table falling down produce stored.
(c) Eutrophication
(d) Toxic Level in food chain A. 1 only
B. 2 only
Q1. Under the Fiscal Responsibility and Budget C. Both 1 and 2
Management (FRBM) Act, which of the following D. Neither 1 nor 2
documents have to be tabled in the Parliament :
Ans - B
1. Medium-term Fiscal Policy Statement The Warehousing Development and Regulatory
2. Fiscal Policy Strategy Statement Authority (WDRA): WDRA was set up by the
3. Macro-economic Framework Statement Government of India on 26.10.2010 to ensure
4. Medium-term Expenditure Framework implementation of the provisions of the Warehousing
Statement (Development & Regulation) Act, 2007.

A. 1, 2 & 3 only It is a statutory authority under the Department of Food


B. 1, 2 & 4 only and Public Distribution, Government of India.
C. 2, 3 & 4 only
D. 1, 2, 3 & 4 only The main objective of WDRA is to implement Negotiable
Warehouse Receipt (NWR) System in the country, which
Ans - D would help farmers to store their produce in scientific
The Fiscal Responsibility and Budget Management storage godowns near by their farms and to seek loan
(FRBM) Act from banks against their NWR. Thus, It implements and

Telegram – Indian_Economy_by_Aman_Soni Page 21


regulates the Negotiable Warehouse Receipt (NWR) International Energy Agency (IEA) country has an
/electronic-NWR (e-NWR) System in the country. obligation to hold emergency oil stocks equivalent to at
least 90 days of net oil imports.
Q3. It is a form of monetary action in which a central
bank seeks to limit the effect of inflows and outflows of In case of a severe oil supply disruption, IEA members
capital on the money supply. The explanation is related may decide to release these stocks to the market as
to : part of a collective action. India’s strategic crude oil
storages are currently located at Visakhapatnam
A. Currency Swap (Andhra Pradesh), Mangalore (Karnataka), and Padur
B. Moral Suasion (Karnataka). The government has also given approval
C. Direct action by RBI for setting up of two additional facilities at Chandikhol
D. Sterilization (Odisha) and Padur (Karnataka).

Ans - D The concept of dedicated strategic reserves was first


Sterilization is a form of monetary action in which a mooted in 1973 in the US, after the OPEC oil crisis.
central bank seeks to limit the effect of inflows and Underground storage is, by far the most economic
outflows of capital on the money supply. Sterilization method of storing petroleum products because the
most frequently involves the purchase or sale of underground facility rules out the requirement of large
financial assets by a central bank and is designed to swathes of land, ensures less evaporation and, since the
offset the effect of foreign exchange intervention. The caverns are built much below the sea level, it is easy to
sterilization process is used to manipulate the value of discharge crude into them from ships.
one domestic currency relative to another and is
initiated in the foreign exchange market. The construction of the Strategic Crude Oil Storage
facilities in India is being managed by Indian Strategic
Classical sterilization involves central banks conducting Petroleum Reserves Limited (ISPRL). ISPRL is a wholly
buy and sell operations in open markets. owned subsidiary of Oil Industry Development Board
Usually, central banks modify classical sterilization by (OIDB) under the Ministry of Petroleum & Natural Gas.
including fiscal policy measures in order to overcome
problems like inflation. Q5. Choose the correct statements.

Q4. Choose the INcorrect statements. 1. India is the largest importer of vegetable oil in
the world.
1. The construction of the Strategic Crude Oil 2. India imports a major share of its sunflower oil
Storage facilities in India is being managed by from the South East Asian Countries
Indian Oil Corporation Ltd.
2. Currently, India does not have any Strategic A. 1 only
petroleum reserves B. 2 only
C. Both 1 and 2
D. Neither 1 nor 2
A. 1 only
B. 2 only Ans - A
C. Both 1 and 2 Indonesia is the largest producer of vegetable oil
D. Neither 1 nor 2 followed by China, and India is the largest importer of
vegetable oil in the world. Ukraine and Russia are the
Ans - C leading producers and exporters of sunflower oil which
Strategic petroleum reserves are huge stockpiles of comprises a 9% production share and nearly a 2%
crude oil to deal with any crude oil-related crisis like the export share for the world vegetable oil market.
risk of supply disruption from natural disasters, war or
other calamities. According to the agreement on an Nearly 60% of world sunflower oil production occurs in
International Energy Programme (I.E.P.), each Ukraine and Russia, and the two countries account for

Telegram – Indian_Economy_by_Aman_Soni Page 22


over 75% of world exports. India imports around 90 D. Neither 1 nor 2
percent of its sunflower oil from the Ukraine-Russia
region. Ans - A
Agroforestry is defined as a land use system which
Q1. Choose the correct statements. integrates trees and shrubs on farmlands and rural
1. National Pharmaceutical Pricing Authority landscapes to enhance productivity, profitability,
(NPPA) is a statutory body under Essential diversity and ecosystem sustainability. It is a dynamic,
Commodities Act, 1955. ecologically based, natural resource management
2. NPPA is the implementing agency of the Drugs system that, through integration of woody perennials
Prices Control Order and regulates the prices of on farms and in the agricultural landscape, diversifies
drugs and sustains production and builds social institutions.

A. 1 only Agroforestry systems include both traditional and


B. 2 only modern land-use systems where trees are managed
C. Both 1 and 2 together with crops and or/ animal production systems
D. Neither 1 nor 2 in agricultural settings. Agroforestry is practiced in both
irrigated and rain fed conditions where it produces
Ans - B food, fuel, fodder, timber, fertilizer and fibre,
National Pharmaceutical Pricing Authority (NPPA) was contributes to food, nutritional and ecological security,
constituted as an attached office of the Department of sustains livelihoods, alleviates poverty and promotes
Pharmaceuticals (DoP) in 1997 under Ministry of productive and resilient cropping and farming
Chemicals & Fertilizers as an independent Regulator for environments.
pricing of drugs and to ensure availability and
accessibility of medicines at affordable prices. Q3. Choose the INcorrect statements.
It fixes ceiling prices of scheduled essential drugs and
monitors Maximum Retail Prices (MRPs) of remaining 1. An Integrity Pact helps in bringing transparency,
non-scheduled medical devices, which have been equity and competitiveness in public
regulated as drugs. procurement.
2. The concept of Integrity Pact is developed by
The Drugs Prices Control Order, 1995 is an order issued the World Bank
by the Government of India under Sec. 3 of Essential
Commodities Act, 1955 to regulate the prices of drugs.
The Order inter alia provides the list of price controlled A. 1 only
drugs, procedures for fixation of prices of drugs, B. 2 only
method of implementation of prices fixed by Govt., C. Both 1 and 2
penalties for contravention of provisions etc. For the D. Neither 1 nor 2
purpose of implementing the provisions of DPCO,
powers of Government has been vested in NPPA. Ans - B
The ‘Integrity Pact’ is an agreement between the
Q2. Choose the correct statements. prospective vendors/bidders and the buyer, committing
persons and officials of both sides to not resort to any
1. Agroforestry is a land use management system corrupt practices in any aspect or stage of the contract.
in which trees or shrubs are grown around or Any violation of the clause entails the disqualification of
among crops or pasturelands. bidders and exclusion from future business dealings.
2. Agroforestry is practiced only in areas with The pact also ensures transparency, equity and
scanty rainfall. competitiveness in public procurement. Integrity pacts
are promoted and supported by the German-based
A. 1 only anti-corruption organisation, Transparency
B. 2 only International, which first developed the concept in the
C. Both 1 and 2 1990s.

Telegram – Indian_Economy_by_Aman_Soni Page 23


A. 1, 2 & 3 only
The Integrity Pact envisages a panel of Independent B. 1, 2 & 4 only
External Monitors (IEMs) for each organisation. IEM C. 2, 3 & 4 only
reviews independently and objectively, whether and to D. 1, 2, 3 & 4 only
what extent parties have complied with their
obligations under the pact. They may submit a report to Ans - B
the chief executive of the organisation concerned or The subsection 4 (2) of the FRMB Act says about various
directly to the CVO and the CVC, if they find serious grounds on which the FRBM’s fiscal deficit target may
irregularities attracting the Prevention of Corruption Act be exempted during a year.
provisions.
(i) national security, act of war,
Q4. Choose the correct statements. (ii) national calamity,
(iii) collapse of agriculture severely affecting farm
1. Fiscal consolidation is the process and steps output and incomes,
taken by the government to increase its Fiscal (iv) structural reforms in the economy with
Deficit. unanticipated fiscal implications,
2. During Fiscal Consolidation, the government (v) decline in real output growth of a quarter by at least
tries to reduce the tax revenue collection three per cent points below its average of the previous
four quarters.
A. 1 only
B. 2 only
C. Both 1 and 2 Q1. Which among the following is/ are the Rabi Crops in
D. Neither 1 nor 2 India :

Ans - D 1. Sorghum
2. Wheat
Fiscal consolidation is defined as concrete policies 3. Rice
aimed at reducing government deficits and debt 4. Barley
accumulation. These consolidation plans and detailed
measures are given as a per cent of nominal GDP. A. 1 only
Taxes are a fiscal policy tool because changes in taxes B. 1 and 3 only
affect the average consumer’s income, and changes in C. 2 and 4 only
consumption lead to changes in real GDP. So, by D. 1, 2 and 4 only
adjusting taxes, the government can influence economic
output. Reduction in tax revenue would mean the Ans - C
government has to borrow more. This borrowing The Indian cropping season is classified into two main
increases the Fiscal Deficit, which is against the concept seasons :
of Fiscal Consolidation. 1. Kharif Season: It is from July to October during the
south–west/summer monsoon. The Kharif crops include
Q5. Which of the following is/are the situations under rice, maize, sorghum, pearl millet/ bajra, finger
which, the Government can use the Escape Clause millet/ragi (cereals, Arhar (pulses), soybean, groundnut
under the FRBM Act for not meeting its Fiscal Deficit (oilseeds) cotton, etc.
Targets.
2. Rabi Season: It is from October to march (north
1. War east/winter monsoon). The rabi crops include wheat,
2. Collapse of agriculture barley, oats (cereals), chickpea/ gram (pulses), linseed,
3. When CPI inflation crosses 9% limit mustard (oilseeds) etc.
4. Decline in real output growth by certain
percentage Q2. Which one of the following is/are the probable
factors which impact the Cropping pattern in India :

Telegram – Indian_Economy_by_Aman_Soni Page 24


6. Enhancement of recharging of aquifers
1. Rainfall 7. Repair and renovate water bodies
2. Minimum support Price 8. Construction of rain harvesting structures (Jal
3. Cost of Fertilizer Sanchay).
4. Market price of a crop in previous season 9. Use of Information Communication Technology (ICT)
in the field of water-use efficiency
A. 1, 2 & 3 only
B. 1, 2 & 4 only Q4. The Kisan Credit Card (KCC) can be used to meet the
C. 2, 3 & 4 only following financial needs of the farmers :
D. 1, 2, 3 & 4 only 1. Short-term credit requirements for crop
cultivation
Ans - D 2. Post-harvest expenses
Factors determining Cropping Pattern 3. Consumption requirements of farmer
1.Natural factors: climate, soil, rainfall etc household
2.Government Policies: MSP, export policy, subsidy 4. Working capital for maintenance of farm assets
3.Economic factors: input prices, income, size of land
holdings, demand of the crop A. 1, 2 & 3 only
4.Social factors: customs, traditions, social environment B. 1, 2 & 4 only
C. 2, 3 & 4 only
Q3. Which of the following is NOT an objective of D. 1, 2, 3 & 4 only
Pradhan Mantri Krishi Sinchai Yojna :
1. Har Khet Ko Pani Ans - D
2. Repair and renovate water bodies Kisan Credit Card (KCC) scheme meets the financial
3. Improve the efficiency in water usage requirements of farmers at various stages of farming.
4. Construction of rain harvesting structures The scheme aims at providing adequate and timely
credit support from the banking system under a single
A. 1 and 2 only window with flexible and simplified procedure to the
B. 3 and 4 only farmers for their cultivation and other needs as
C. 2, 3 and 4 only indicated below:
D. None of the above
 To meet the short-term credit requirements for
Ans - D cultivation of crops;
Pradhan Mantri Krishi Sinchai Yojana has been  Post-harvest expenses;
formulated by combining ongoing schemes such as  Produce marketing loan;
Accelerated Irrigation Benefit Programme (AIBP),  Consumption requirements of farmer
Integrated Watershed Management Programme household;
(IWMP), River Development and Ganga Rejuvenation  Working capital for maintenance of farm assets
(RD & GR) and on Farm Water Management (OFWM). and activities allied to agriculture;
 Investment credit requirement for agriculture
It focuses on the following: and allied activities.
1. Expand the coverage of irrigation (‘Har Khet Ko Pani’)
2. Improve the efficiency in water usage (‘More Crop Q5. Choose the correct statements.
per Drop’) 1. Gypsum which is used in the manufacturing of
3. Encourage expansion activities linked to water lime and in cement industry, causes
harvesting deterioration in soil quality and leads to
4. Attract greater private investments in irrigation to chemical leaching.
enhance agricultural production and productivity and 2. Natural Gas is used as a raw material in the
increase farmers’ income. Fertilizer Industry
5. Exploration of the feasibility of reusing treated
municipal waste water

Telegram – Indian_Economy_by_Aman_Soni Page 25


Urea imports continue are restricted and canalized (only
A. 1 only three STEs can import urea). There are around 30
B. 2 only companies involved in domestic urea production and 3
C. Both 1 and 2 companies are allowed to import urea into the country.
D. Neither 1 nor 2 These 3 companies are government owned and are
called State trading Enterprises (STEs).
Ans - B
Gypsum is available in a fine mesh powder form from Q2. Choose the correct statements.
Phosphoric Acid plant and is mainly used in agriculture
as soil amendment. It works as an agent to remove 1. Neem coated urea helps in reducing the soil and
Saline/Alkaline ingredients in the soil. It acts more or water pollution by reducing the leaching
less like manure. 2. Neem coated urea reduces the
Gypsum also improves the ability of soil to drain and not misappropriation of urea to non agricultural
become waterlogged due to a combination of high uses but it adversely impacts the crop yield.
sodium, swelling clay and excess water. When we apply
gypsum to soil it allows water to move into the soil and A. 1 only
allow the crop to grow well. B. 2 only
C. Both 1 and 2
Main raw material used by fertilizer industry is naphtha D. Neither 1 nor 2
(by product of petroleum), rock phosphate, sulfur
gypsum and natural gas. Hence most of the plants are Ans - A
near petroleum refineries, coal mines or sea coasts. Neem coated urea is the urea which is coated with
Natural gas is the most preferred feedstock for the neem seed oil. In 2015, Government allowed urea
production of fertilizers because it has the highest producers to produce neem coated urea up to 100% of
hydrogen to carbon ratio (hydrogen is used for production and made it mandatory to produce a
production of ammonia, and thereafter urea is minimum of 75 percent of domestic urea as neem
manufactured with the reaction of ammonia, coated, so that farmers are benefitted. Farmers will
withcarbon dioxide also produced during the process). have to pay an additional price of only Rs.14/- per bag
of neem coated urea. Neem coated urea is required less
Q1. Choose the correct statements. in quantity with same plot size and gives higher crop
yields.
1. In India, any importer can import Urea by
paying the necessary duties and sell it in the Underground water contamination due to leaching of
domestic market. urea also gets reduced with neem coating since
2. The price of urea is statutorily fixed by the nitrogen in the neem coated urea gets released to
Government of India plants very slowly, helping plants gain more nutrients.
Neem coated urea is not fit for industrial use, so
A. 1 only chances of its illegal diversion to industries will also be
B. 2 only lesser.
C. Both 1 and 2
D. Neither 1 nor 2 Q3. Which one of the following is/are the possible
benefits of increase in Soil testing by the Farmers :
Ans – B
Urea is a nitrogenous fertilizer and it is heavily 1. Reduction in input costs
subsidized by the Central Government. Today urea is 2. Rampant use of fertilizers
the only fertilizer whose price remains controlled. Urea 3. Leads to sustainable agriculture
Subsidy is a part of Central Sector Scheme of 4. Increases Monoculture
Department of Fertilizers and is wholly financed by the
Government of India through Budgetary Support. A. 1 and 3 only
B. 1, 3 and 4 only

Telegram – Indian_Economy_by_Aman_Soni Page 26


C. 1, 2 and 4 only 1. Laying down procedure and guidelines for
D. 1, 3 and 4 only accreditation of laboratories
2. Contribute to the development of international
Ans - A technical standards for food, sanitary and
Soil testing reduces cultivation cost by application of phyto-sanitary standards.
right quantity of fertilizer.
It ensures additional income to farmers by increasing A. 1 only
yields. In the long term, it will ensure food security. B. 2 only
It promotes crop diversification and sustainable C. Both 1 and 2
farming. D. Neither 1 nor 2
It is also creating jobs for the agrarian youth through
provision of subsidy for setting up labs. Ans - D
Recent addition is the pilot project of ‘Development of
Model Villages’ which encourages sampling and testing The Food Safety and Standards Authority of India
of cultivable soil in partnership with the farmers. Under (FSSAI) has been established under Food Safety and
the pilot, one village per block is adopted to collect soil Standards , 2006 which consolidates various acts &
samples at the level of individual farm holdings rather orders that have hitherto handled food related issues in
than at grid level. various Ministries and Departments. FSSAI has been
created for laying down science based standards for
Q4. Which one of the following is/are the techniques articles of food and to regulate their manufacture,
applied under the Organic farming : storage, distribution, sale and import to ensure
availability of safe and wholesome food for human
1. Crop rotation consumption.
2. Use of Bio-fertilizers
3. Application of mineral bearing rocks FSSAI has been mandated by the FSS Act, 2006 for
4. Use of organic wastes performing the following functions:

A. 1, 2 & 3 only  Framing of Regulations to lay down the


B. 1, 2 & 4 only Standards and guidelines in relation to articles
C. 2, 3 & 4 only of food and specifying appropriate system of
D. 1, 2, 3 & 4 only enforcing various standards thus notified.
 Laying down mechanisms and guidelines for
Ans - D accreditation of certification bodies engaged in
Organic farming is a production system which avoids or certification of food safety management system
largely excludes the use of synthetically compounded for food businesses.
fertilizers, pesticides, growth regulators, genetically  Laying down procedure and guidelines for
modified organisms and livestock food additives. accreditation of laboratories and notification of
the accredited laboratories.
To the maximum extent possible organic farming  To provide scientific advice and technical
system rely upon crop rotations, use of crop residues, support to Central Government and State
animal manures, legumes, green manures, off farm Governments in the matters of framing the
organic wastes, biofertilizers, mechanical cultivation, policy and rules in areas which have a direct or
mineral bearing rocks and aspects of biological control indirect bearing of food safety and nutrition.
to maintain soil productivity and tilth to supply plant  Collect and collate data regarding food
nutrients and to control insect, weeds and other pests. consumption, incidence and prevalence of
biological risk, contaminants in food, residues of
Q5. Which among the following is NOT a function of various, contaminants in foods products,
Food Safety and Standards Authority of India (FSSAI) : identification of emerging risks and introduction
of rapid alert system.

Telegram – Indian_Economy_by_Aman_Soni Page 27


 Creating an information network across the scattered form in forest and non-forest areas as well as
country so that the public, consumers, in waste land /deserts/hilly areas. These TBOs are also
Panchayats etc receive rapid, reliable and good source of vegetable oil and therefore need to be
objective information about food safety and supported for cultivation.
issues of concern.
 Provide training programmes for persons who Q2. The “Tilhan Mission” often seen in news is related
are involved or intend to get involved in food to :
businesses. A. Providing Farm equipment to Punjab farmers to
 Contribute to the development of international reduce the crop residue burning
technical standards for food, sanitary and B. Mission for exploration of Rare earth minerals
phyto-sanitary standards. in Indian Ocean
 Promote general awareness about food safety C. Mission to boost oilseeds production in the
and food standards. country
D. DRDO’s invention to improve the ambient air
Q1. Choose the INcorrect statements. quality
1. A substantial portion of our requirement of
edible oil is met through import of palm oil from Ans - C
Indonesia and Malaysia. This initiative is to boost oilseeds production and
2. Under the National Mission on Oilseeds and Oil country self reliant in oilseed production. This Tilhan
Palm (NMOOP) the govt has a target to increase Mission by Union govt. of India will boost Indian
production under the Tree Borne Oilseeds economy as it will reduce imports and raise exports.
(TBOs) component too. Thus the Gross Domestic Product (GDP) of the nation
will improve.
The main objective of National Tilhan Mission 2021 is to
A. 1 only encourage local farmers to produce oilseeds and
B. 2 only thereby reducing import of edible oils.
C. Both 1 and 2
D. Neither 1 nor 2 A large portion of edible oils is met from import of palm
oil from Indonesia and Malaysia. So there is need to
Ans - D exploit domestic resources to maximize production of
India’s vegetable oil economy is world’s fourth largest edible oil in India. Palm Oil is new crop with highest
after USA, China & Brazil. The oilseed accounts for 13% vegetable oil yielding capacity. So, govt. will now launch
of the Gross Cropped Area, 3% of the Gross National National Tilhan Mission to boost Oilseeds production in
Product and 10% value of all agricultural commodities. the country.
The diverse agro-ecological conditions in the country
are favourable for growing 9 annual oilseed crops, Q3. Choose the correct statements wrt Integrated
which include 7 edible oilseeds (groundnut, rapeseed & Scheme of Oil Seeds, Pulses, Oil palm and Maize
mustard, soybean, sunflower, sesame, safflower and (ISOPOM).
niger) and two non-edible oilseeds (castor and linseed).
Oilseeds cultivation is undertaken across the country in 1. Under this scheme, financial assistance is
about 27 million hectares mainly on marginal lands, of provided to farmers for purchase of breeder
which 72% is confined to rainfed farming. seed.
2. Apart from the primary sources of oil, the
A substantial portion of our requirement of edible oil is scheme also covers the two non edible sources
met through import of palm oil from Indonesia and of oil.
Malaysia.
Tree Borne Oilseeds (TBOs), like sal, mahua, simarouba, A. 1 only
kokum, olive, karanja, jatropha, neem, jojoba, cheura, B. 2 only
wild apricot, walnut, tung etc. are cultivated/grown in C. Both 1 and 2
the country under different agro-climatic conditions in a D. Neither 1 nor 2

Telegram – Indian_Economy_by_Aman_Soni Page 28


This app will help the individual farmers, willing to
Ans - C provide their agricultural machinery & equipments on
The first programme on Oilseeds was launched in 1986 rental basis to increase their farm income beside
as Technology Mission on Oilseeds (TMO). The core idea making the optimum utilization of the available
was to increase the production and productivity of Agricultural machineries available in CHCs/FMBs/Hi-
oilseeds to make the country self-reliant in this vital tech Machinery Hubs. This app will provide a platform
sector. Later Pulses, Oil Palm & Maize were brought in for sell and purchase of old agriculture machinery to
its ambit in the 1990s. The scheme was later farmers also.
restructured in 2004 as Integrated Scheme of OilSeeds,
Pulses,Oilpalm and Maize (ISOPOM). Q5. Choose the INcorrect statements.

To increase production of oilseeds including soybean in 1. Under the Accelerated Irrigation Benefits
the country, the Government is implementing a Programme (AIBP) the government provides
Centrally Sponsored Integrated Scheme of Oilseeds, Central Loan Assistance (CLA) to major/medium
Pulses, Oil Palm and Maize (ISOPOM) in 14 major irrigation projects in the country
oilseeds growing States. 2. In India, Irrigation falls under the concurrent
Under this scheme, financial assistance is provided to list.
farmers for purchase of breeder seed, and other inputs
including free distribution of seed mini kits of oilseeds A. 1 only
and maize to encourage all types of farmers including B. 2 only
small and marginal farmers to grow these crops. C. Both 1 and 2
The Government fixes Minimum Support Price (MSP) for D. Neither 1 nor 2
agricultural commodities including Soybean on the
recommendations of the Commission for Agricultural Ans - B
Costs and Prices (CACP) to ensure remunerative prices Irrigation is a state subject and irrigation projects are
to the growers for their produce. formulated, executed and funded by the State
Governments themselves from their own resources.
Q4. Choose the correct statements. Central assistance is released in the form of block loans
and grants not tied to any sector of development or
1. The FARMS App connects the farmers with project. A large number of major and medium irrigation
Custom Hiring Centers in their locality to rent projects in the country are languishing due to various
farm machinery reasons, the most important of them being inadequate
2. Only the Gram panchayat is empowered to rent provision of funds by the concerned State
machinery under this scheme. Governments. Since the irrigation projects are capital
intensive, and states with limited resources at their
A. 1 only disposal find themselves unable to meet the desired
B. 2 only fund demands of all the projects, the implementation of
C. Both 1 and 2 these projects get delayed.
D. Neither 1 nor 2
Keeping the above in view, Central Government, during
Ans - A 1996-97, launched an Accelerated Irrigation Benefits
FARMS (Farm Machinery Solutions) Mobile App Programme (AIBP) to provide Central Loan Assistance
This Multi-language Mobile App platform 'FARMS- Farm (CLA) to major/medium irrigation projects in the
Machinery Solutions' will facilitate local farmers and country, with the objective to accelerate the
citizens of the different States across the country with implementation of those projects which were beyond
the Custom hiring services of Farm Machinery Banks, resource capability of the states or were in advanced
Custom Hiring Centers and Hi-tech Hubs established stage of completion. While selecting the projects,
under the various Schemes of DAC&FW, MoA&FW special emphasis was to be given to Pre-fifth and Fifth
without any computer support system. Plan projects. Priorities were also given to those
projects which were benefiting Tribal and Drought

Telegram – Indian_Economy_by_Aman_Soni Page 29


Prone Areas. However, under the revised AIBP
Guidelines from the year 1999-2000 onwards, Central Q2. Choose the correct statements wrt Pradhan Mantri
Loan Assistance under AIBP could also be extended to Kisan Sampada Yojana PMKSY
minor surface irrigation projects of special category 1. It is an umbrella scheme incorporating ongoing
states (N.E. States & Hilly States of H. P., Sikkim, J&K, schemes of the Ministry like Mega Food Parks,
Uttaranchal and projects benefiting KBK districts of Integrated Cold Chain and Value Addition
Orissa). Infrastructure etc.
2. The objective of PMKSY is to bring about
Q1. Choose the correct statements. ecologically healthy, economically viable, and
socially inclusive development of the Fisheries
1. The Mega Food Park Scheme aims to establish sector of India.
modern food processing units in the industrial
plots with well-established supply chain. A. 1 only
2. It is based on “Cluster” approach and helps in B. 2 only
maximizing value addition, minimizing wastage, C. Both 1 and 2
increasing farmers income D. Neither 1 nor 2

Ans - A
A. 1 only The Central Sector Scheme - SAMPADA (Scheme for
B. 2 only Agro-Marine Processing and Development of Agro-
C. Both 1 and 2 Processing Clusters) was approved by the cabinet in
D. Neither 1 nor 2 May 2017 for the period of 2016-20 coterminous with
the 14th Finance Commission cycle. The scheme has
Ans - C now been renamed as the "Pradhan Mantri Kisan
The Scheme of Mega Food Park aims at providing a Sampada Yojana (PMKSY)".
mechanism to link agricultural production to the market
by bringing together farmers, processors and retailers It is an umbrella scheme incorporating ongoing schemes
so as to ensure maximizing value addition, minimizing of the Ministry like Mega Food Parks, Integrated Cold
wastage, increasing farmers income and creating Chain and Value Addition Infrastructure, Food Safety
employment opportunities particularly in rural sector. and Quality Assurance Infrastructure, etc. and also new
The Mega Food Park Scheme is based on “Cluster” schemes like Infrastructure for Agro-processing
approach and envisages creation of state of art support Clusters, Creation of Backward and Forward Linkages,
infrastructure in a well-defined agri / horticultural zone Creation / Expansion of Food Processing & Preservation
for setting up of modern food processing units in the Capacities. The objective of PMKSY is to supplement
industrial plots provided in the park with well- agriculture, modernize processing and decrease Agri-
established supply chain. Waste.

Mega food park typically consist of supply chain Q3. Which one of the following is/are the Objectives of
infrastructure including collection centers, primary Agriculture Export Policy 2018 :
processing centers, central processing centers, cold
chain and around 25-30 fully developed plots for 1. To Double agricultural exports
entrepreneurs to set up food processing units.The Mega 2. To diversify our export basket
Food Park project is implemented by a Special Purpose 3. To promote indigenous products exports
Vehicle (SPV) which is a Body Corporate registered 4. Increasing market access to farmers
under the Companies Act. State Government, State
Government entities and Cooperatives are not required A. 1, 2 & 3 only
to form a separate SPV for implementation of Mega B. 1, 2 & 4 only
Food Park project. Subject to fulfillment of the C. 2, 3 & 4 only
conditions of the Scheme Guidelines, the funds are D. 1, 2, 3 & 4 only
released to the SPVs.

Telegram – Indian_Economy_by_Aman_Soni Page 30


Ans - D  The first component relates to incentivising
manufacturing of four major food product
Agriculture Export Policy 2018 - Objectives segments viz. Ready to Cook/ Ready to Eat
 To double agricultural exports from present (RTC/ RTE) foods including Millets based
~US$ 30+ Billion to ~US$ 60+ Billion by 2022 products, Processed Fruits & Vegetables,
and reach US$ 100 Billion in the next few years Marine Products, Mozzarella Cheese.
thereafter, with a stable trade policy regime.  Innovative/ Organic products of SMEs including
 To diversify our export basket, destinations and Free Range - Eggs, Poultry Meat, Egg Products in
boost high value and value added agricultural these segments are also covered under above
exports including focus on perishables. component.
 To promote novel, indigenous, organic, ethnic,  The conditions of stipulated Minimum Sales and
traditional and non-traditional Agri products mandated investment will not be applicable for
exports. entities selected for making innovative/ organic
 To provide an institutional mechanism for products.
pursuing market access, tackling barriers and  The second component relates to support for
deal with sanitary and phytosanitary issues. branding and marketing abroad to incentivise
 To strive to double India’s share in world agri emergence of strong Indian brands.
exports by integrating with global value chain at  For promotion of Indian Brand abroad, the
the earliest. scheme envisages grant to the applicant entities
 Enable farmers to get benefit of export for - in store Branding, shelf space renting and
opportunities in overseas market. marketing.

Q4. Choose the INcorrect statements wrt Production Q5. Which among the following are correctly matched
Linked Incentive Scheme for Food Processing Industry
1. ISI Mark – Jewellery
1. The scheme incentivizes manufacturing of Ready to 2. BIS Hallmark – Electric Motor, Helmet
Cook, Millets based, Marine Products & other food 3. AGMARK - Agricultural produce
products
2. Organic products like Eggs, Poultry Meat, Egg A. 1 and 2 only
Products have been specifically excluded from the B. 1 and 3 only
scheme. C. 3 only
D. All of the above
A. 1 only
B. 2 only Ans - C
C. Both 1 and 2  ISI Mark - It is mandatory for products like
D. Neither 1 nor 2 switches, electric motors, wiring cables,
heaters, kitchen appliances, some varieties of
Ans - B cement, packaged mineral water, etc
Production Linked Incentive Scheme for Food  BIS Hallmark - It is generally for Jewellery
Processing Industry (PLISFPI)  Indian Organic certification - Mandatory for
 Production Linked Incentive Scheme for Food organically farmed food products manufactured
Processing Industry (PLISFPI)” to support in India
creation of global food manufacturing  AGMARK - certification mark for agricultural
champions commensurate with India’s natural produce
resource endowment and support Indian
brands of food products in the international
markets with an outlay of Rs.10900 crore. Q1. Choose the correct statements.
 Central Sector Scheme with an outlay of Rs.
10900 crore 1. China is the top export destination from India.

Telegram – Indian_Economy_by_Aman_Soni Page 31


2. Recently, for the first time ever, Indian exports have reserves, boosts manufacturing and also increases
crossed the $400 billion target government's revenue collection.

A. 1 only It is also a good means by which a country can bring


B. 2 only itself out of the recession phase. Exporting to countries
C. Both 1 and 2 with a favourable economic climate helps in increasing
D. Neither 1 nor 2 the GDP levels as well as helps in reducing
unemployment.Besides, it also plays a key role in
Ans - B strengthening the domestic manufacturing units by
India achieved its ambitious target of crossing $400 scaling up their quality to make India made products
billion exports. compete and stand out against global peers.
This is the first time ever that exports have crossed the
$400 billion-mark. The previous best was $331.02 billion Q3. Which of the following steps India has to take, in
that was achieved in 2018-19. order to increase its exports :
One of the major reasons for jump in exports is rise in
pent up demand, which had fallen as the Covid 1. Production-liked incentive (PLI) schemes
pandemic forced nations to remain under strict 2. Refund of Duties on Exported Products
lockdown, thereby impacting global trade.Beside, boost 3. Local content requirements
in domestic manufacturing due to production-liked 4. Facilitate trade and increase FTA utilisation by
incentive (PLI) schemes and implementation of some exporters
interim trade pacts have also led to surge in exports.
A. 1, 2 & 3 only
The USA remained the top export destination, followed B. 1, 2 & 4 only
by UAE and China. India has diversified its export C. 2, 3 & 4 only
destinations in the last 25 years, yet more than 40% D. 1, 2, 3 & 4 only
India’s exports are still accounted for by only 7
countries. Only eight products constitute more than 55 Ans - B
per cent of the country’s total exports, there is a critical One of the major reasons for jump in exports is rise in
need for product diversification. pent up demand, which had fallen as the Covid
pandemic forced nations to remain under strict
Q2. The Exports of a nation have impact on which of the lockdown, thereby impacting global trade. Beside, boost
following macro economic factors : in domestic manufacturing due to production-liked
incentive (PLI) schemes and implementation of some
1. Growth of the Economy interim trade pacts have also led to surge in exports.
2. Exchange Rate
3. Level of domestic inflation In its mission to promote locally made products in
4. Domestic Interest Rates global market, the Centre implemented a series of steps
to promote exports of both goods and services and that
A. 1, 2 & 3 only includes the introduction of Refund of Duties and Taxes
B. 1, 2 & 4 only on Exported Products (RoDTEP) and Rebate of State and
C. 2, 3 & 4 only Central Levies and Taxes (RoSCTL) Schemes, the launch
D. 1, 2, 3 & 4 only of Common Digital Platform for Certificate of Origin to
facilitate trade and increase FTA utilisation by exporters,
Ans - D promoting districts as export hubs by identifying
Exports are one of the fundamental drivers of growth products with export potential in each district and
for any economy. It can influence a country's GDP, addressing bottlenecks, and promoting ease of doing
exchange rate, level of inflation as well as interest rates. business. Local content requirements helps in reducing
A robust export data is beneficial as it leads to increase imports, not in increasing exports.
in job opportunities, enhances foreign currency

Telegram – Indian_Economy_by_Aman_Soni Page 32


Q4. Which among the following is NOT an advantage of online through electronic means, teleshopping, multi-
Custom Hiring Centre (CHC) of Farm Machinery : level marketing or direct selling.

A. Provides access to small and marginal farmers Consumer is having the following six consumer rights
to costly farm machinery under the Act
B. Leads to Reduction in cost of cultivation • Right to Safety
C. Promotes Inefficiency in the use of farm inputs • Right to be Informed
due to mechanization • Right to Choose
D. Provides Work opportunities to skilled labour • Right to be heard
• Right to seek Redressal
Ans - C • Right to Consumer Awareness
Custom Hiring Centre (CHC) are basically a unit
comprising a set of farm machinery, implements and Six consumer rights have been defined, including the
equipment meant for custom hiring by farmers. The right to: (i) be protected against marketing of goods and
main objective of CHC is to supply of farm implements services which are hazardous to life and property; (ii) be
to small, marginal and poor farmers at subsidized rates informed of the quality, quantity, potency, purity,
on hire. standard and price of goods or services; (iii) be assured
of access to a variety of goods or services at competitive
Advantages : prices; and (iv) seek redressal against unfair or
 Provides access to small and marginal farmers restrictive trade practices.
to costly farm machinery
 Facilitates timeliness in farm operations and Q1. Which of the following is NOT a member of the G-
efficient use of inputs 20 :
 Promotes adoption of climate resilient practices 1. Argentina
and technologies by farmers because of 2. Philippines
availability of appropriate machines at 3. Venezuela
reasonable hiring charges 4. Turkey
 Reduction in cost of cultivation
 Provides work opportunities to skilled labour A. 1 and 2 only
and small artisans B. 2 and 3 only
C. 1, 2 and 3 only
Q5. A consumer residing in Delhi, has which of the D. None of the above
following consumer rights :
1. Right to Choose Ans - B
2. Right to Safety The G20 is an informal group of 19 countries and the
3. Redressal against unfair or restrictive trade European Union, with representatives of the
practices. International Monetary Fund and the World Bank.
4. To be informed of the quality & quantity The G20 membership comprises a mix of the world’s
largest advanced and emerging economies,
A. 1, 2 and 3 only representing about two-thirds of the world’s
B. 1, 2 and 4 only population, 85% of global gross domestic product, 80%
C. 2, 3 and 4 only of global investment and over 75% of global trade.
D. All of the above
The members of the G20 are Argentina, Australia, Brazil,
Ans - D Canada, China, France, Germany, India, Indonesia, Italy,
A consumer is defined as a person who buys any good Japan, Republic of Korea, Mexico, Russia, Saudi Arabia,
or avails a service for a consideration. It does not South Africa, Turkey, the United Kingdom, the United
include a person who obtains a good for resale or a States, and the European Union.
good or service for commercial purpose. It covers
transactions through all modes including offline, and
Telegram – Indian_Economy_by_Aman_Soni Page 33
Q2. Choose the correct statement wrt NITI Aayog Export and growth orientation across states; and lack of R&D
Preparedness Index infrastructure to promote complex and unique exports.

1. EPI ranks on 4 main pillars—Policy; Business


Ecosystem; Export Ecosystem; Export Q3. The current benchmark rate used by Indian banks
Performance for calculating the cost of funds and in determining
2. Most of the Landlocked states are best lending interest rate is :
performers in terms of export preparedness
A. Repo Rate
B. Benchmark Prime Lending Rate
A. 1 only C. Base Rate
B. 2 only D. Marginal Cost of Lending Rate
C. Both 1 and 2
D. Neither 1 nor 2 Ans - D
Marginal Cost of Lending Rate (MCLR): It came into
Ans - A effect in April 2016. It is a benchmark lending rate for
NITI Aayog, in partnership with the Institute of floating-rate loans. This is the minimum interest rate at
Competitiveness, released the Export Preparedness which commercial banks can lend. This rate is based on
Index (EPI) 2021. The report is a comprehensive analysis four components—the marginal cost of funds, negative
of India’s export achievements. The index can be used carry on account of cash reserve ratio, operating costs
by states and union territories (UTs) to benchmark their and tenor premium. MCLR is linked to the actual deposit
performance against their peers and analyse potential rates. Hence, when deposit rates rise, it indicates the
challenges to develop better policy mechanisms to banks are likely to hike MCLR and lending rates are set
foster export-led growth at the subnational level. to go up.

The EPI ranks states and UTs on 4 main pillars—Policy; Q4. Choose the Incorrect statements wrt proposed
Business Ecosystem; Export Ecosystem; Export National Bank for Financing Infrastructure and
Performance—and 11 sub-pillars—Export Promotion Development (NBFID)
Policy; Institutional Framework; Business Environment;
Infrastructure; Transport Connectivity; Access to 1. NBFID will Extend loans and advances for
Finance; Export Infrastructure; Trade Support; R&D financing infrastructure projects
Infrastructure; Export Diversification; and Growth 2. It also helps in facilitating foreign participation
Orientation. in infrastructure projects

This edition has shown that most of the ‘Coastal States’ A. 1 only
are the best performers, with Gujarat as the top- B. 2 only
performer. The EPI’s primary goal is to instil competition C. Both 1 and 2
among all Indian states (‘Coastal’, ‘Landlocked’, D. Neither 1 nor 2
‘Himalayan’, and ‘UTs/City-States’) to bring about
favourable export-promotion policies, ease the Ans - D
regulatory framework to prompt subnational export The National Bank for Financing Infrastructure and
promotion, create the necessary infrastructure for Development Bill, 2021 was introduced in Lok Sabha on
exports, and assist in identifying strategic March 22, 2021. The Bill seeks to establish the National
recommendations for improving export Bank for Financing Infrastructure and Development
competitiveness. It promotes competitive federalism (NBFID) as the principal development financial
and a fair contest among States/UTs. institution (DFIs) for infrastructure financing.
EPI 2021 brings out three major challenges to India’s
export promotion. These are intra- and inter-regional NBFID will be set up as a corporate body with
differences in export infrastructure; weak trade support authorised share capital of one lakh crore rupees.

Telegram – Indian_Economy_by_Aman_Soni Page 34


Functions of NBFID: 1. Cross Subsidization is strategy where support
 Extending loans and advances for infrastructure for a product comes from the profits generated
projects. by another product.
 Taking over or refinancing such existing loans. 2. In Indian Railways, low freight rates are
 Attracting investment from private sector supported by high passenger fares
investors and institutional investors for
infrastructure projects. A. 1 only
 Organising and facilitating foreign participation B. 2 only
in infrastructure projects. C. Both 1 and 2
 Facilitating negotiations with various D. Neither 1 nor 2
government authorities for dispute resolution in
the field of infrastructure financing. Ans - A
 Providing consultancy services in infrastructure Cross Subsidization is strategy where support for a
financing. product comes from the profits generated by another
product. The low price of a product is sustained by the
Q5. Choose the correct statements wrt Unorganised earnings of another product sold by the same company.
Workers’ Social Security Act, 2008 In Indian Railway, low passenger fares are supported by
high freight (cargo) rates. That is why Indian railway
1. The act empowers Centre to provide welfare freights are among the costliest in the world making
schemes relating to life and disability cover of Indian industry uncompetitive.
Unorganised workers.
2. It also contains provisions regarding health but Q2. The ‘FAME India scheme’ often seen in news is
not ones relating to maternity benefits related to :

A. Encouraging electric vehicles by providing


A. 1 only subsidies.
B. 2 only B. Encouraging dedicated freight corridors In
C. Both 1 and 2 Railways
D. Neither 1 nor 2 C. Encouraging women into STEM courses
D. Encouraging startups by providing tax free
Ans - A holidays
Unorganised Workers’ Social Security Act, 2008
empowers the Central Government to provide Social Ans – A
Security benefits to unorganised sector workers by Faster Adoption and Manufacturing of Electric Vehicles
formulating suitable welfare schemes on matters in India Phase II (FAME India Phase II)’ for promotion of
relating to life and disability cover, health and maternity Electric Mobility in the country.
benefits, old age protection and any other benefit as FAME India is a part of the National Electric Mobility
may be determined by the Central Government. Mission Plan. Main thrust of FAME is to encourage
electric vehicles by providing subsidies.
The State Governments are also empowered to Vehicles in most segments – two wheelers, three
formulate suitable welfare schemes on the matters wheelers, electric and hybrid cars and electric buses will
regarding housing, provident funds, educational obtain the subsidy benefit of the scheme.
schemes, skill upgradation, old age homes etc. FAME focuses on 4 areas i.e. Technology development,
Demand Creation, Pilot Projects and Charging
Infrastructure.

Q3. Which of the following is a sub component pillar, on


Q1. Choose the correct statements. which the Sagarmala project is based :
1. Port modernization
2. Port connectivity

Telegram – Indian_Economy_by_Aman_Soni Page 35


3. Port led industrialization immense potential but are lying dormant. The civil
4. Coastal community development aviation ministry with the assistance of state
governments, wants to develop “no frills”
A. 1, 2 & 3 only airports,(airports with basis essential services only and
B. 1, 2 & 4 only no luxuries) at an indicative cost of Rs. 50-100 crore
C. 2, 3 & 4 only each.
D. 1, 2, 3 & 4 only The scheme seeks to reserve a minimum number of
UDAN seats i.e. seats at subsidized rates and also cap
Ans - D the fare for short distance flights.
The Sagarmala project has been proposed to connect all
coastal cities in the country through road, rail, ports and Q5. Choose the INcorrect statements.
airports through a special development package. It will 1. The objective of District Mineral Foundation is
include coastal economic zones, smart cities, islands to to work for the interest of the benefit of the
attract tourists etc. The project is designed to enhance persons and areas affected mining related
the unique identities of coastal cities in consonance operations
with their sea-side culture. 2. The state govts will bear the entire premium of
The Sagarmala project is based on four pillars:- total royalty payable to people under social
 Port modernization security schemes
 Port connectivity
 Port led industrialization A. 1 only
 Coastal community development B. 2 only
C. Both 1 and 2
Q4. Choose the correct statements wrt UDAN(Ude Desh D. Neither 1 nor 2
ka Aam nagrik) Scheme
Ans - B
1. Under the Scheme, the govt would develop “no
frills” airports The Mines and Minerals (Development & Regulation)
2. The scheme also seeks to reserve a minimum Amendment Act, 2015
number of UDAN seats i.e. seats at subsidized  It mandated the setting up of District Mineral
rates and also cap the fare for short distance Foundations (DMFs) in all districts in the
flights. country affected by mining related operations
and to protect the interests of tribal
communities who have borne the costs of
A. 1 only mining.
B. 2 only  The miners have to contribute a fraction to DMF
C. Both 1 and 2 of total royalty payable to people.
D. Neither 1 nor 2  It also mandated the establishment of the
National Mineral Exploration Trust (NMET). It is
Ans - C a non-profit body run by the Central
UDAN(Ude Desh ka Aam nagrik) Regional Connectivity government with the primary objective of
Scheme (RCS) promoting regional and detailed mineral
The objective of the RCS is to make flying affordable for exploration in the country
the masses, to promote tourism, increase employment  The objective of District Mineral Foundation is
and promote balanced regional growth. The scheme to work for the interest of the benefit of the
intends to put on the aviation map India's smaller cities persons and areas affected mining related
and towns, which were till now uneconomical for operations in such manner as may be
commercial airlines to fly. prescribed by the State Government.
 Jurisdiction: Its manner of operation comes
Integral to the scheme is the development of the 450 under the jurisdiction of the relevant State
odd airports/ airstrips in the country which has Government
Telegram – Indian_Economy_by_Aman_Soni Page 36
Ans - B
New Exploration licensing policy, 1997
Q1. Choose the correct statements wrt National Mineral Before NELP,oil exploration was open for public
Policy 2019 enterprises only, but after it, even private players could
also bid.
1. The Policy provides for Right of First Refusal for This aimed to facilitate the exploration sector by
those who discover minerals providing a level playing field to private players against
2. Under the policy, afforestation practices are public enterprises.
made integral part of the mine development
strategy Revenue sharing Contracts(RSC) as an alternative to
production sharing contracts (PSC): Under revenue
A. 1 only sharing model, government gets share in revenue from
B. 2 only the exploration from the very beginning. In contrast
C. Both 1 and 2 PSC, allows government to have revenue share only
D. Neither 1 nor 2 when capital costs are recovered by explorer.

Ans - C Q3. Which of the following are currently part of OPEC –


Right of First Refusal for those who discover minerals: Organization of Petroleum Exporting Countries:
Earlier, if a firm that has a reconnaissance permit (RP) 1. Iran
found evidence of minerals, it has to inform the 2. Qatar
government. The government then auctioned off the 3. Venezuela
area. This was not an exciting option for explorers. The 4. Saudi Arabia
new policy gives firms with reconnaissance permit (RP)
and prospecting license (PL) the right of first refusal to A. 1, 2 & 3 only
mine them if they discover minerals. B. 1, 2 & 4 only
C. 1, 3 & 4 only
Auctioning in virgin areas: Firms will be allowed to give D. 1, 2, 3 & 4 only
the government the coordinates of the area they would
like to mine in, and this will then be put up for auction Ans - C
in a short period of time. This is similar to the open Current OPEC members are Algeria, Angola, Equatorial
acreage policy (OALP) in the oil and gas sector. Guinea, Gabon, Iran, Iraq, Kuwait, Libya, Nigeria, the
Prevention and Mitigation Of Adverse Environmental Republic of the Congo, Saudi Arabia, the United Arab
Effects due to mining in accordance with the latest Emirates and Venezuela. Ecuador, Indonesia and Qatar
scientific norms and modern afforestation practices are former OPEC members.
shall form an integral part of mine development
strategy. Q4. Choose the correct statements wrt
HELP(Hydrocarbon Exploration & Licensing Policy)
Q2. Choose the correct statements. 1. HELP promotes the Open Acreage Policy and
option to select the exploration blocks
1. Under the New Exploration licensing policy throughout the year without waiting for the
(NELP), 1997 oil exploration was open for public formal bid round by Govt
enterprises only 2. Contractor will need a separate license to
2. NELP shifted from Production sharing contracts explore conventional unconventional oil and gas
to Revenue sharing Contracts resources

A. 1 only A. 1 only
B. 2 only B. 2 only
C. Both 1 and 2 C. Both 1 and 2
D. Neither 1 nor 2 D. Neither 1 nor 2

Telegram – Indian_Economy_by_Aman_Soni Page 37


Ans – A
A. 1 only
Features of HELP B. 2 only
1. Uniform License: It will enable the contractor to C. Both 1 and 2
explore conventional as well as unconventional oil and D. Neither 1 nor 2
gas resources including Coalbed methane (CBM), shale
gas/oil, tight gas and gas hydrates under a single
license, instead of issuing separate licenses for each Ans - C
kind of hydrocarbons. The Industrial Policy Statement of1973
2. Open Acreages: It gives the option to a hydrocarbon It introduced some new thinking into the economy with
company to select the exploration blocks throughout major ones being as follows:
the year without waiting for the formal bid round from (i) A new classificatory term i.e., core industries was
the Government. created. The industries which were of fundamental
3. Revenue Sharing Model importance for the development of industries were put
in this category such as iron and steel, cement, coal,
Q5. Choose the INcorrect statements. crude oil, oil refining and electricity.
(ii) Out of the six core industries defined by the policy,
1. Uranium Corporation of India Ltd is responsible the private sector could apply for licences for the
for mining and processing of uranium ore for industries which were not a part of schedule A of the
commercial purposes In India Industrial Policy, 1956.
2. Jharkhand state is the largest producer of (iii) Some industries were put under the reserved list in
uranium in India which only the small or medium industries could be set
up.
A. 1 only (iv) A limited permission to foreign investment was
B. 2 only given, with the multinational corporations (MNCs) being
C. Both 1 and 2 allowed to set up subsidiaries in the country. This
D. Neither 1 nor 2 limited permission was restricted to the areas where
there was a requirement of foreign capital. Such MNCs
Ans - B entered the Indian Economy with the help of a partner
In India, Uranium Corporation of India Ltd. (UCIL) under from India- the partner being the major one with 74%
the Department of Atomic Energy, is the only shares in the subsidiaries set up for by the MNCs.
organisation responsible for mining and processing of
uranium ore for commercial purposes. Uranium mined Q2. The term “Washington Consensus” is related to
by the UCIL is used for weapons and civil nuclear
programmes both. The imported uranium is used for A. Document about human rights in the areas of
civil nuclear energy purposes only. sexual orientation and gender identity
B. Statement concerning the restitution of art
The state of Andhra Pradesh is the largest producer of confiscated by Germany during WW II
uranium in India. Tummalapalle village located in the C. Document on the epidemiology of severe acute
Kadapa district of Andhra Pradesh is considered one of respiratory syndrome (SARS)
the largest uranium reserves in the world. D. Set of economic policy prescriptions to reform
economic crisis struck countries

Q1. Choose the correct statements. Ans - D


1. Under The Industrial Policy Statement of1973 a The Washington Consensus is a set of ten economic
new classificatory term i.e., core industries was policy prescriptions considered to constitute the
created "standard" reform package promoted for crisis-wracked
2. Under the same policy, MNCs were allowed to developing countries by Washington, D.C.-based
set up subsidiaries In India with an Indian joint institutions such as the International Monetary Fund
Partner holding the major share (IMF), World Bank and United States Department of the

Telegram – Indian_Economy_by_Aman_Soni Page 38


Treasury. The term was first used in 1989 by English
economist John Williamson. A. 1 only
The prescriptions encompassed free-market
market promoting B. 2 only
policies in such areas as macroeconomic stabilization, C. Both 1 and 2
economic opening with respect to both trade and D. Neither 1 nor 2
investment, and the expansion of market forces within
the domestic economy. Ans - D
A special economic zone (SEZ) is an area in which
10 Elements of Washington Consensus : business and trade laws are different from the rest of
1. Fiscal Adjustment: the country. SEZs are located within a country’s national
2. Tax Reforms: borders. They are established with the objective of
3. Deregulation: promoting exports.
4. Trade Liberalisation: The SEZ Act was passed in 2005. According to thet act,
5. Competitive Exchange Rate: SEZ units are given the following benefits:
6. Privatisation: Overseas banking units can open branches in SEZs.
7. Removal of Barriers to Foreign Investment: These branches will not be required to follow RBI
8. Financial Reforms: guidelines regarding rate of lending and will be free to
9. Protection of Property Rights: borrow from the international market. These banking
ban
10. Redirection of Public Sector Investment: units will lend at internationally competent rates to SEZ
units.
Q3. Which of the following are conditions imposed on Labour laws are not applicable to SEZ units.
India by the IMF in the bailout package during the BOP Further sale from domestic units to SEZ units will be
crisis of 1990’s : considered export and, thus, exempted from taxes. On
other hand, sales from SEZ units to t domestic units will
1. Revaluation of the rupee be considered import and import duties will be levied.
2. Reduction in imports tariff
3. Cut government expenditure
4. Increase Excise Duty Q5. Which of the following have been correctly
A. 1, 2 & 3 only matched :
B. 1, 2 & 4 only
C. 2, 3 & 4 only 1. Industry 1.0 – Mass Production & Assembly line
D. 1, 2, 3 & 4 only 2. Industry 2.0 - Mechanization
3. Industry 3.0 – Automated Production
Ans - C 4. Industry 4.0 – IOT, Machine Learning
The IMF conditions put forth for India were as under:
 Devaluation of the rupee by 22 percent A. 1 & 2 only
 Drastic reduction in the peak imports tariff B. 3 & 4 only
 Excise duties (cenvat now ) to be hiked by 20 C. 2, 3, 4 only
percent to neutralize the revenue shortfalls due D. All of the above
to the custom cut
 All government expenditure to be cut down by Ans - B
10 percent annually

Q4. Choose the INcorrect statements.

1. A special economic zone is established with the


objective of promoting exports
2. Labour laws are not applicable to SEZ units for a
certain Period
Telegram – Indian_Economy_by_Aman_Soni Page 39
Ans - A
The new guidelines explicitly state that an entity having
Q1. Choose the correct statements. equity participation by e-commerce marketplace entity
or its group companies will not be permitted to sell its
1. Under Disinvestment, the management of the products on the platform run by such a marketplace
company is always handed over to the Buyer entity. The wording of the clause is clearly meant to
2. Under Strategic Disinvestment, the plug the loopholes of the earlier policy.
management of the company is handed over to
the Buyer The new policy also specifies that an e-commerce
marketplace entity will not mandate any seller to sell
A. 1 only any product exclusively on its platform only. The
B. 2 only intention of this clause is to ensure that the e-
C. Both 1 and 2 commerce marketplace player doesn’t arm twist a
D. Neither 1 nor 2 vendor into exclusive arrangements.

Ans - B
Government may also sell off a unit to a strategic buyer- Q3. It is the act of a domestic market leader lowering its
entire equity. Strategic buyer is one who not only buys prices below its costs, to gain an unfair advantage in the
the chunk of entire equity- in one tranche or more but market. The above explanation is related to the
also takes over management. That is the ‘strategic’ part following term :
of the sale. It is unlike disinvestment where sale of
shares is unaccompanied by management control A. Dumping of goods
transfer. The strategic partner gives a higher price for B. Discounts
the shares as he gets management control along with it( C. Predatory Pricing
management premium). Also, running of the unit D. Subsidies
improves. Privatization and strategic sale are the same.
Ans - C
Selling minority shares of Public Enterprises, to another Predatory pricing is the act of a market leader lowering
entity be it public or private is disinvestment. In this the its prices below its costs to gain an unfair advantage.
government retains ownership of the enterprise.On the Though the predator incurs short-term losses, it hurts
other hand, when the government sells majority shares other players and drives them out of the market. Later,
in an enterprise, that is strategic disinvestment/sale. with fewer competitors, the predator can raise prices to
Here, the government gives up the ownership of the recoup losses. Eg. Jio
entity as well.
Q4. Choose the INcorrect statements.
Q2. Choose the correct statements.
1. National Pharmaceutical Pricing Agency is a
1. The group company controlled by an e- statutory body under the Ministry of Commerce
commerce marketplace entity, shall not sell its 2. NPPA undertakes monitoring and enforcement
products on the platform run by such a of prices of scheduled drugs through market
marketplace entity surveillance
2. An e-commerce marketplace entity can
mandate any seller to sell any product
exclusively on its platform only
A. 1 only
A. 1 only B. 2 only
B. 2 only C. Both 1 and 2
C. Both 1 and 2 D. Neither 1 nor 2
D. Neither 1 nor 2
Ans - A

Telegram – Indian_Economy_by_Aman_Soni Page 40


National Pharmaceutical Pricing Agency is an 1. Structural Unemployment –Absence of
independent, executive body of experts under the work/job
Ministry of Chemicals and Fertilizers formed in 1997.Its 2. Cyclical unemployment – During periods of
primary function is to fix & revise the prices of recession
scheduled formulations under the Drugs (Prices Control) 3. Open unemployment – Bank tellers replaced by
Order (DPCO). ATM’s

It also undertakes monitoring and enforcement of A. 1 and 2 only


prices of scheduled drugs through market surveillance, B. 2 only
purchase of samples by NPPA officers all over India to C. 1 and 3 only
ensure compliance, examine complaints by individuals / D. 1, 2 and 3 only
NGOs
NPPA also provides inputs to the Government on Ans - C
pharmaceutical policy and issues related to Structural Unemployment – Bank tellers replaced by
affordability, availability and accessibility of medicines. ATM’s
Cyclical unemployment – During periods of recession
Q5. Choose the correct statements. Open unemployment – Absence of work/job

1. Fair and Remunerative Price (FRP) of sugar is Structural Unemployment: This kind of unemployment
approved by the Commission for Agricultural happens when the structure of an economy changes. It
Costs and Prices (CACP) essentially occurs because there exists a mismatch of
2. States are free to fix State Advised Prices (SAP) skills between the skills of the unemployed and skills
for sugar cane, which are generally higher than needed for the job. E.g. when computers came, the
the FRP people who worked on typewriters became
unemployed.
A. 1 only
B. 2 only The causes of structural unemployment can include
C. Both 1 and 2 shifts in the economy, improvements in technology, and
D. Neither 1 nor 2 workers lacking job skills that are required for them to
find employment.
Ans - B
Fair and Remunerative Price (FRP) is the cane price Conversely, swings in companies’ business cycles and a
announced by the Central Government on the basis of period of negative economic growth—called a
the recommendations of the Commission for recession—can cause cyclical unemployment. In other
Agricultural Costs and Prices (CACP) after consulting the words, cyclical unemployment is usually the result of
State Governments and associations of sugar industry. insufficient demand for goods and services.

(NOTE – CACP just recommends the prices. It is Open unemployment: This refers to a situation when
approved by the Cabinet Committee on Economic there are some workers who have absolutely no work to
Affairs) do. They are willing to work at the prevailing wage rate,
but they are forced to remain unemployed in the
Citing differences in cost of production, productivity absence of work. These workers are completely idle.
levels and also as a result of pressure from farmers' Such unemployment is clearly visible as the number of
groups, some states declare state specific sugarcane such persons can be clearly counted and therefore it is
prices called State Advised Prices (SAP), usually higher known as open unemployment
than the FRP
Q2. Choose the correct statements wrt “Jobless
Q1. Which of the following are INcorrectly matched. Growth”

Telegram – Indian_Economy_by_Aman_Soni Page 41


1. In a scenario of jobless growth, workers Disguised Unemployment: It occurs when people are
possessing employable skills will be benefitted employed in a job where their presence or absence
over the unskilled workers does not make any difference to the output of the
2. In such a scenario, Economy recovery is economy. It is a special kind of case.
insufficient to absorb the unemployed & under-
employed into the workforce Here people are apparently employed but their
marginal product is zero (contribution to production is
A. 1 only nil). Marginal product here refers to the produce added
B. 2 only to the existing production due to addition of a new
C. Both 1 and 2 employee/ worker. e.g. if 4 persons are employed in a
D. Neither 1 nor 2 factory and they produce 17 units and a fifth person is
added and the production increases to 19 units, the
Ans - C additional 2 units is marginal product.
When growth is coupled with high unemployment, it Here, we can consider the fifth person as employed. If
means that the economy is experiencing structural there is no increase in production, the marginal product
changes. This structural shift offers opportunities for is zero and he is disguisedly unemployed. Even if he is
some, difficult choices for others. removed from the activity, there will be no decline in
production.
As the population of a country grows, people need to
work in order to support their families and themselves. Such type of unemployment is quite common in the
An expanding economy is necessary to employ all those agricultural sector in India. Because of the large families
who seek work. Without sufficient economic growth, in the rural areas several people work on farms and at
people looking for work will be unable to find it. In any times, the work of 2-3 people is done by 4-5 people
economic condition, it is the individual workers because otherwise it would result in unemployment.
possessing employable skills who will find work first. If But in reality this is nothing but a case of disguised
the supply of jobs is plentiful, then more opportunities unemployment.
open up for those with less attractive skill sets.
Q4. Choose the correct objectives of The Periodic
In a jobless growth economy, unemployment remains Labour Force Survey (PLFS) :
stubbornly high even as the economy grows. This tends
to happen when a relatively large number of people 1. Measurement of labour force participation and
have lost their jobs, and the ensuing recovery is employment status only in the urban areas in
insufficient to absorb the unemployed, under- the Current Weekly Status (CWS)
employed, and those first entering the workforce. 2. Measurement of unemployment for both rural
and urban areas on key parameters in both
usual status and Current Weekly Status.
Q3.Choose the correct statements wrt Disguised
unemployment
A. 1 only
1. Alternative employment is not available B. 2 only
2. Marginal productivity of labour is zero C. Both 1 and 2
D. Neither 1 nor 2

A. 1 only
B. 2 only Ans - C
C. Both 1 and 2 Periodic Labour Force Survey (PLFS) is released by the
D. Neither 1 nor 2 National Statistical Office (NSO).
The Periodic Labour Force Survey (PLFS) was designed
Ans – B with two major objectives for measurement of
employment and unemployment.

Telegram – Indian_Economy_by_Aman_Soni Page 42


The first was to measure the dynamics in labour force Q1. Choose the correct statements.
participation and employment status in the short time
interval of three months for only the urban areas in the 1. Foster index involves the combined
Current Weekly Status (CWS). Second one was, for both measurement of incidence of poverty and
rural and urban areas, to measure the labour force depth of poverty
estimates on key parameters in both usual status 2. The Poverty Gap is the percentage of the
(ps+ss) and Current Weekly Status. population who are below the poverty line

The Current Weekly Status (CWS) approach to A. 1 only


measuring unemployment uses seven days preceding B. 2 only
the date of survey as the reference period. C. Both 1 and 2
D. Neither 1 nor 2
A person is considered to be employed if he or she
pursues any one or more of the gainful activities for at Ans – A
least one-hour on any day of the reference week. On
the other hand, if a person does not pursue any gainful The Poverty Gap is a measure of the intensity of poverty
activity, but has been seeking or available for work, the among the poor. It is the difference between the mean
person is considered as unemployed. income of the poor and the poverty line. This indicator
measures the magnitude of poverty as well as its
Usual Principal Status and Subsidiary Status (PS+SS) - If a intensity i.e. number of poor and how poor they are.
person has engaged in any economic activity for a The Poverty Gap Index is the combined measurement of
period of 30 days or more during the preceding 365 incidence of poverty (How many poor) and depth of
days a person is considered as employed under this poverty (How much poor). The Poverty Gap Index is also
approach. called the Foster index.

Q5. The term “unemployment trap” is related to : Headcount ratio is described as the percentage of the
population who are below the poverty line, that is, the
A. Cyclical unemployment during the periods of population which is not able to buy a basic basket of
recession items. It is also called poverty rate or incidence of
B. Unemployment arising from the mismatch poverty or number of people who are below the
between the jobs available in the market and poverty line or BPL.
the skills
C. An incentive being provided to return to work Q2. Choose the INcorrect statements.
D. A situation when unemployment benefits
discourage the unemployed to go to work. 1. Dadabhai Naoroji formulated a poverty line in
Ans - D his book, Poverty and the un-British rule in
Unemployment trap is a situation when unemployment India.
benefits discourage the unemployed to go to work. 2. The calculation of poverty line basket of goods,
People find the opportunity cost of going to work too involve the cost of only vegetarian dishes and
high when one can simply enjoy the benefits by doing excludes meat
nothing.
A. 1 only
While the purpose of social security and welfare B. 2 only
systems is to provide relief to the unemployed, they end C. Both 1 and 2
up providing them with an incentive not to return to D. Neither 1 nor 2
work. An unemployment trap arises when opportunity Ans – B
cost of going to work is higher than the income
received, discouraging people from returning to work One of the earliest estimates of poverty was done by
and being productive. Dadabhai Naoroji in his book, poverty and the un-British

Telegram – Indian_Economy_by_Aman_Soni Page 43


rule in India. He formulated a poverty line ranging from Economist Simon Kuznets had hypothesized that as an
Rs 16 to Rs 35 per capita per year based on 1867-68 economy develops, inequality increases but later
prices. The poverty line proposed by him was based on reduces. In other words, the shape of the Kuznets curve
the cost of a subsistence diet consisting of rice or flour, is like an inverted U. Japan, Germany and some other
dhal, mutton, vegetables, ghee, vegetable oil and salt. social democracies in Europe support the hypothesis

The recent work of Thomas Piketty, ‘Capital in the


Q3. Choose the correct statements. Twenty-First Century’, questions the Kuznets Curve.
Piketty shows that since 1980, there has been a sharp
1. For a perfect equality, Gini Co-efficient will be 1 rise in inequality in the US, Japan and Europe. His data
2. For complete inequality, Gini Co-efficient has to shows a U-curve in the trends of inequality in the
be zero advanced nations - US, Japan, Germany, France and
Great Britain - the exact opposite of the Kuznets Curve.
A. 1 only Inequality grows sharply after having fallen initially for a
B. 2 only few years.
C. Both 1 and 2
D. Neither 1 nor 2 Q5. The term “Gig economy”, often seen in news is
related to :
Ans - D
The Lorenz curve is used to calculate the Gini Co- A. A labour market characterized by freelance
efficient which is the numerical indicator or inequality in work
a country B. Industry 4.0 and changing production processes
To compute the Gini Co-efficient, we first measure the C. Formalization of informal workforce
area between the Lorenz Curve and the 45 degree D. The seasonal workforce in agriculture during
equality line. This area is divided by the entire area harvest
below the 45 degree line (which is always exactly one
half). The quotient is the Gini Co-efficient, a measure of
inequality.

For a perfect equality, there would be no area between Ans - A


the 45 degree line and the Lorenz curve - a Gini Co- Gig economy is a labour market characterized by the
efficient of zero. For complete inequality, in which only prevalence of short-term contracts or freelance work as
one person has any income (if that were possible, the opposed to permanent jobs.
Gini Co-efficient will be 1). The Gini Co-efficient of most Gig workers are independent contractors, online
of the economies stays between zero and one platform workers, contract firm workers, on-call
workers and temporary workers. Gig workers enter into
Q4. Choose the correct statements. formal agreements with on-demand companies to
1. Kuznets Curve says that as an economy provide services to the company's clients
develops, inequality increases but later reduces.
2. Kuznets wrote the work ‘Capital in the Twenty- Q1. Under the Income approach of calculation of GDP,
First Century which of the following is included :

A. 1 only 1. Land Rent


B. 2 only 2. Interest on Capital
C. Both 1 and 2 3. Consumption Expenditure
D. Neither 1 nor 2 4. Profits

Ans – A A. 1, 2 & 3 only


B. 1, 2 & 4 only
C. 2, 3 & 4 only

Telegram – Indian_Economy_by_Aman_Soni Page 44


D. 1, 2, 3 & 4 only A. 1 only
B. 2 only
Ans - B C. Both 1 and 2
Under Income Approach, GDP is sum of all factor D. Neither 1 nor 2
incomes generated in the production of a good.
It includes: Ans – A
1. Wages Nominal GDP and Real GDP
2. Profit to the owners of the firm If the GDP or any other related aggregates is measured
3. Rent earned by owners of the land in terms of current market prices then it is called
4.Interest earned by the person providing capital nominal GDP.

Q2. Which of the following factors does NOT influence Nominal GDP includes the influence of inflation on the
the Incremental Capital Output Ratio (ICOR) of a prices. Nominal GDP will change when either the overall
country : price level changes or when actual volume of
1. Level of Technology production changes or when both change
2. Skills of the workforce simultaneously.
3. Quality of the education system
4. Ease of doing business Real GDP is calculated at constant prices i.e., prices of
the base year. In this method, the GDP value is
A. 1, 2 & 3 only expressed in terms of prices prevailing in a year chosen
B. 1, 2 & 4 only to be a base year. In the given example,2015 is taken as
C. All of the above the base year and prices of 2015 are called base prices
D. None of the above or constant prices.

Ans - D
ICOR (Incremental Capital Output Ratio) is defined as
incremental / additional capital required to produce one Q4. Which of the following is NOT included in the GDP
additional unit of output. ICOR is how much extra unit of country :
of capital is required to produce one additional unit of
output. It represents how efficiently capital is being 1. Social Cost of Pollution
used in a country to produce output. 2. Interest on debt Instruments
3. Agricultural Production
The higher the ICOR, the lower is the productivity of 4. Government transfer payments
capital.
ICOR is influenced by a number of factors such as A. 1 & 3 only
technology, skill of the labour force, which in turn B. 1 & 4 only
depends on the quality of the education system and C. 2 & 3 only
ease of doing business. Bureaucratic hurdles, which D. 2, 3 & 4 only
impede speedy execution of projects need to be
removed. Thus improving the productivity of capital Ans – B
requires steps at several fronts.
In financial markets, people buy and sell financial assets
such as shares. When someone one buys a share from
Q3. Choose the correct statements. another person, there’s only transfer of ownership.
There is no production activity but only exchange of
1. Real GDP is calculated at constant prices i.e., funds. Hence the value of shares and bonds is not
prices of the base year. included in GDP. But the interest earned on bonds and
2. Real GDP includes the influence of inflation on dividend on shares is included in GDP.
the prices.

Telegram – Indian_Economy_by_Aman_Soni Page 45


GDP estimation doesn’t account for environmental loss the economy. Diversifying exports is also considered
incurred in production of goods and services. important to escape the middle income trap.
Eg. River water degradation due to discharge of
chemical waste Q1. Which of the following would the govt undertake
during a period of Low Growth in the Economy :

Q5. Which of the following are the characteristics of a 1. Increase spending on infrastructure
country suffering from Middle income trap : 2. Reduce the rate of GST
3. Increase allocation for MGNREGA
1. Low investment 4. Reduction of subsidies
2. Highly Diversified Industry
3. Slow growth in manufacturing A. 1, 2 & 3 only
4. High productivity and innovation B. 1, 2 & 4 only
C. 2, 3 & 4 only
A. 1 & 3 only D. 1, 2, 3 & 4 only
B. 1 & 4 only
C. 2 & 3 only Ans – A
D. 2, 3 & 4 only
During Expansionary fiscal policy , Government
Ans - A increases spending on infrastructure creation, salaries,
Economists Indermit Gill and Homi Kharas coined the pensions etc. and reduces tax rates.
term Middle income trap at World Bank in 2006 when When an economy has slowed down, unemployment
they were working in the ground strategies for Eastern levels have increased, spending has reduced, and
Asian economics. businesses are not making substantial profits. In such a
According to the concept, a country in the middle- scenario, the government follows expansionary fiscal
income trap has lost its competitive edge in the export policy , which leads to more funds with people and,
of manufactured goods due to rising wages and is consequently, rise in demand, investment, employment
unable to keep up with more developed economies in and output. However any such increase in expenditure
the high-value-added market. and decrease in revenue collection would also depend
on the financial condition of the government.
They suffer from low investment, slow growth in the
secondary sector of the economy, limited industrial
diversification and poor labor market conditions. Q2. Choose the correct statements.

Avoiding the middle income trap requires identifying 1. Sovereign bonds are debt securities issued by
strategies to introduce new processes and find new the government of a country
markets to maintain export growth. It is also important 2. Credit ratings do not affect the value of
to increase domestic demand, because an expanding Sovereign bonds
middle class can use its increasing purchasing power to
buy high-quality, innovative products and help drive A. 1 only
growth. B. 2 only
C. Both 1 and 2
The biggest challenge is to move from resource-driven D. Neither 1 nor 2
growth based on cheap labor and cheap capital to
growth based on high productivity and innovation. This Ans - A
requires investments in infrastructure and education— Sovereign bond – It is debt security issued by a national
building a high-quality education system that government. It can be denominated in foreign currency
encourages creativity and supports breakthroughs in or domestic currency. Because of default risk, they are
science and technology that can be applied back into offered at a discount. Less developed countries have

Telegram – Indian_Economy_by_Aman_Soni Page 46


difficulty issuing sovereign bonds denominated in their 1. Real interest rates can never be negative
own currency & thus have to issue in foreign currency. 2. USA experienced the situation of negative Real
interest rates
A country facing an economic crisis has to pay higher
interest to the investors for the higher risk they are A. 1 only
incurring, to make the bonds attractive and investment B. 2 only
worthy. C. Both 1 and 2
D. Neither 1 nor 2
Q3. Choose the correct statements.
Ans - A
1. Disinvestment proceedings from a PSU is Too much of government debt can lead to inflation and
considered as a Revenue receipt for the reduction in real interest rates(may becomes negative
government also if inflation is too high)(The interest which bank pays
2. Debt servicing is Capital expenditure for the to us on demand deposits or time deposits is nominal
government interest rate, but if nominal interest rate is adjusted to
inflation, we get real interest rate. Real interest rate is
nominal interest rate minus inflation rate.
A. 1 only
B. 2 only Suppose the bank is giving us 6% interest on fixed
C. Both 1 and 2 deposits of one year, but if the inflation rate is 4% in
D. Neither 1 nor 2 that year then my actual benefit(real interest rate) by
keeping money in the bank is only 2%). People will
Ans - D withdraw deposits from the banks. It might prompt
Non debt Creating Capital Receipts include: Loan people to invest more in gold and real estate, thereby
recovered from states and other countries, accentuating the problem of poor economic liquidity
Disinvestment proceeds etc.(Both of them reduce and black money. Banks would be left with very less
assets) money to lend for investment. Investment would come
down and hence growth also. To counter inflation, RBI
Non Tax Revenue Receipts :It includes user charges, will have to increase the interest rates. Currently, In the
bills, penalties, profits and dividends from PSUs (the US, where nominal interest rates are near zero, real
government is the largest shareholder of PSUs) , rates stand at around -5.3 per cent.
Interest earned on loans given, grants which the
government receives (both external (centre receives Q5. Which of the following would lead to Fiscal
from World Bank and other countries) and internal consolidation :
(states receive from centre),service income from
railways, advertisements and CISF giving protection to 1. Rationalization of government subsidies
MNCs, etc. 2. Increase in tax compliance
3. Increasing tax concessions
Revenue Expenditure - This expenditure of the 4. Widening the tax base
government neither creates assets nor reduces A. 1, 2 & 3 only
liabilities. It is synonymous with maintenance, B. 1, 2 & 4 only
consumption and welfare. It includes Interest paid on C. 2, 3 & 4 only
loans taken(also called debt servicing), salaries and D. 1, 2, 3 & 4 only
pension & provident fund contribution, subsidies,
defense, account expenditure, Law and order Ans - B
expenditure, expenditure on social service. Fiscal consolidation is a reduction in the underlying
fiscal deficit. It refers to the ways and means of
narrowing the fiscal deficit. Fiscal Consolidation refers
Q4. Choose the INcorrect statements. to the policies undertaken by Governments (national

Telegram – Indian_Economy_by_Aman_Soni Page 47


and sub-national levels) to reduce their deficits and D. 2, 3 & 4 only
accumulation of debt stock.
Ans - C
Steps to be taken : Direct taxes of Union
 Better targeting of government subsidies
 Improving efficiency of tax administration by 1. Income tax
eliminating evasion of tax, increasing tax 2. Corporate tax/MAT
compliance, reducing tax avoidance 3. Capital Gains Tax.
 Enhancing tax GDP ratio by widening the tax 4. STT (Securities Transaction Tax)
base and minimizing tax concessions and 5. Dividend Distribution Tax
exemptions also improves tax revenues.
Q3. Choose the correct statements.
Q1. Choose the correct statements.
1. Minimum Alternative Tax is levied under the
1. Tax Buoyancy is the percentage change in tax Integrated GST (IGST) Act.
revenue with change in Tax Rates 2. Minimum Alternative Tax is imposed only on
2. Tax buoyancy is influenced by volume of the tax foreign companies which export services to
base and transparency of the tax rates etc India

A. 1 only A. 1 only
B. 2 only B. 2 only
C. Both 1 and 2 C. Both 1 and 2
D. Neither 1 nor 2 D. Neither 1 nor 2

Ans - B Ans - D
Tax Buoyancy refers to the percentage change in tax Minimum Alternative Tax (MAT) is a tax levied under
revenue with the growth of national income (GDP). That Income Tax Act of India, 1961.Under the provisions of
is, growth-based increase in tax collections. the Minimum Alternate Tax Act every company
Tax buoyancy is the relationship among variations in the domestic or foreign is required to pay MAT. The rule
government’s tax income change and the potential in was put to practice so as to ensure that no taxpayer
GDP. It has to do with the sensitivity of tax revenue with substantial economic income is able to avoid tax
growth to changes in GDP. liability by use of various exclusions, deductions and
credits.
When a tax collects greater revenue without changing
the rate of taxing, it is said to be buoyant. Tax buoyancy There are several “zero tax companies” that book high
is influenced by several factors, including the volume of profit but pay almost nil taxes by rolling out substantial
the tax base, the attractiveness of the tax authorities, dividends to their shareholders. This nil tax comes as a
and the rationale and transparency of the tax rates. result of various exemptions, deductions and incentives
provided to them due to several conditions that they
Q2. Which of the following is NOT a direct Tax for the meet. However, the aim of MAT is to ensure that no
Central Govt company which has the ability to pay taxes, gets to
avoid payment of income tax.
1. Income tax
2. GST Q4. Choose the correct statements.
3. Tax on Agriculture
4. Capital Gains Tax 1. A cess is levied by the government for a specific
purpose.
A. 1 & 3 only 2. The Centre has to share the surcharges with the
B. 1 & 4 only states but it is not mandated to share the cess
C. 2 & 3 only
Telegram – Indian_Economy_by_Aman_Soni Page 48
A. 1 only
B. 2 only Q1. Choose the correct statements.
C. Both 1 and 2
D. Neither 1 nor 2 1. Liquidity Adjustment Facility (LAF) is extended
to scheduled commercial banks only
Ans - A 2. The interest rate in LAF is determined by the RBI
A cess imposed by the central government is a tax on
tax. It is levied by the government for a specific
purpose. A. 1 only
Generally, cess is expected to be levied till the time the B. 2 only
government gets enough money for that purpose. C. Both 1 and 2
Amount collected by imposing a cess for a particular D. Neither 1 nor 2
purpose can be used for that purpose only and not for
any general purpose. Ans - B
Eg.SBC( Swachh Bharat Cess).KBC(Krishi Kalyan Cess). LAF is a facility which was introduced in 2000 and is
Proceeds of SBC will be used for Swachh Bharat Mission. extended by RBI to the scheduled commercial banks
Both these cesses have been subsumed in GST now. (excluding RRBs), NBFCs and primary dealers to avail of
We still have higher education cess and education cess liquidity in case of requirement or park excess funds
levied on Income Tax. with the RBI in case of excess liquidity against the
collateral of government securities including State
A common feature of both surcharge and cess is that Government securities. Basically LAF enables liquidity
the centre need not share them with the states and management on a day to day basis.
they don’t form part of divisible pool of taxes
The operations of LAF are conducted by way of
Q5. Choose the INcorrect statements. repurchase agreements (repos and reverse repos) with
RBI being the counterparty to all the transactions. The
1. The GST Council decides the initial rates of GST interest rate in LAF is fixed by the RBI from time to time.
tax slabs and any further changes have to be It is different from OMOs as OMOs involve outright
done through a legislature in the parliament purchase/sale of security while LAF is for
2. Under Reverse Charge, the liability to pay tax is lending/borrowing by keeping g-secs as collateral.
on the recipient of supply of goods
Q2. Choose the correct statements.

A. 1 only 1. Both Helicopter Money and Quantitative easing


B. 2 only will increase the liquidity in the economy
C. Both 1 and 2 2. Under Quantitative Easing money is directly
D. Neither 1 nor 2 given to the people

Ans - A A. 1 only
The Goods and Services Tax (GST) is governed by the B. 2 only
GST Council. The GST council is the key decision-making C. Both 1 and 2
body that will take all important decisions regarding the D. Neither 1 nor 2
GST. The GST Council decides GST rate, tax exemption,
the due date of forms, and tax deadlines, which items Ans - A
will be included or excluded from GST etc. Helicopter money
American economist Milton Friedman coined this term.
Under GST means, Reverse Charge means the liability to Under such a policy, a central bank "directly increases
pay tax is on the recipient of supply of goods or services the money supply and, via the government, distributes
instead of the supplier of such goods or services in the new cash to the population with the aim of boosting
respect of notified categories of supply. demand.

Telegram – Indian_Economy_by_Aman_Soni Page 49


This is an unconventional monetary policy tool aimed at (iii)competitive rates can be good for savers and
bringing a flagging economy back on track. investors
It involves printing large sums of money and distributing
it to the public. Q4. Choose the correct statements.

Quantitative Easing 1. For inflation targeting and for Setting Policy


It is an unconventional monetary policy in which the Rates Wholesale Price Index (WPI) is the
central bank purchases government securities or other parameter
securities from the market in order to lower interest 2. The Finance Minister is the Chairperson of the
rates and increase the money supply. Monetary Policy Committee

Both Helicopter Money and Quantitative easing will A. 1 only


increase the liquidity in the economy but Quantitative B. 2 only
Easing is different from the former since money is not C. Both 1 and 2
given directly to the people but by buying bonds from D. Neither 1 nor 2
the banks at lesser interest rates and once RBI sells
bonds it can always take the money back from the Ans - D
banks. For inflation targeting and for Setting Policy Rates
CPI(Consumer Price Index) is going to be the sole
Q3. Choose the INcorrect statements. parameter. Before CPI, Wholesale Price Index (WPI) was
the main index for measurement of inflation in India till
1. A higher inflation in the economy will lead to April 2014.
higher interest rates in the market
2. Currently RBI fixes the lending rates on bank Monetary Policy Committee - The Government will
loans which will help in competitiveness nominate three members and the RBI will also
nominate three members (RBI Governor, Deputy
A. 1 only Governor-Monetary Policy & One Executive Director).A
B. 2 only search committee will recommend three external
C. Both 1 and 2 members who will be experts in the field of economics,
D. Neither 1 nor 2 banking or finance, for the Government appointees. The
decision will be taken by the majority. The RBI
Ans - B Governor, chair of the committee, will have a casting
The determinants of interest rates are: vote.
 Inflation: the higher the inflation, the higher the
interest rates because the liquidity has to be Q5. Choose the correct statements.
decreased to counter inflation
 Need for growth: lower interest rates reduce 1. Currently, credit cards cannot be used to
cost of credit and facilitate investments for withdraw cash from the ATM’s
growth. 2. Currently, the facility of cardless cash
 Promotion of savings:If savings have to be withdrawal through ATMs is limited only to a
increased, deposit rates have to be increased. few banks.

As a part of banking sector reforms, interest rates have A. 1 only


been deregulated. It means that the banks can B. 2 only
determine their lending and borrowing rates and RBI C. Both 1 and 2
won’t interfere in it. The rationale is that D. Neither 1 nor 2
(i) banks can adjust rates quickly according to market
conditions; Ans - B
(ii) populism through regulation can be prevented, Credit card cash advance is the technical term for credit
card cash withdrawal facility. It allows credit

Telegram – Indian_Economy_by_Aman_Soni Page 50


cardholders to withdraw cash using their credit cards at o Nodal person for coordination with law
the bank’s ATM. Using the facility cardholders can enforcement agencies,
withdraw cash within permissible limits and shall repay  An e-commerce entity should not allow
the same along with interest and other charges. The misleading advertisements on its platform.
feature may not be available on all the credit cards and  The Draft Rules prohibit specific/ back to back
the permissible limits for cash withdrawal may also vary flash sales — defined as “sale organized by an e-
from card to card along with the finance charges. commerce entity at significantly reduced prices,
high discounts or any other such promotions”.
The Reserve Bank of India (RBI) has proposed to make
cardless cash withdrawal facility available at all ATMs, Q2. India’s first pure green hydrogen plant is
irrespective of banks, through the Unified Payment commissioned in the state of
Interface (UPI). At the moment, a few banks such as
ICICI Bank, Kotak Mahindra Bank, HDFC Bank and SBI, A. Uttarakhand
allow their users to withdraw cash from their ATMs B. Assam
without a card, a feature introduced in the wake of the C. Meghalaya
Covid-19 pandemic. D. Maharashtra

Q1. Choose the correct statements wrt Draft Consumer Ans – B


Protection (E-Commerce) Rules, 2021:
Oil India Limited (OIL) has taken the first significant step
1. The rules contain a “country of origin” clause towards Green Hydrogen Economy in India with the
2. Draft Rules prohibit flash sales & misleading commissioning of India’s First 99.999% pure Green
advertisements on e com platforms Hydrogen pilot plant, with an installed capacity of 10 kg
per day at its Jorhat Pump Station in Assam.
A. 1 only
B. 2 only The plant produces Green Hydrogen from the electricity
C. Both 1 and 2 generated by the existing 500kW Solar plant using a 100
D. Neither 1 nor 2 kW Anion Exchange Membrane (AEM) Electrolyser
array. The use of AEM technology is being used for the
Ans - C first time in India. This plant is expected to increase its
Key Features: production of green hydrogen from 10 kg per day to 30
 None of an e-commerce entity’s related parties kg per day in future. The company has initiated a
can be enlisted as a seller for sale to consumers detailed study in collaboration with IIT Guwahati on
directly. This “broad definition” of ‘related blending of Green Hydrogen with Natural Gas and its
party’ can potentially include all entities such as effect on the existing infrastructure of OIL.
those involved in logistics, any joint ventures,
etc. Q3. Which one of the following is NOT an external
 E-commerce entities offering imported goods/ benchmark mandated by RBI, which is to be followed by
services to ‘incorporate a filter mechanism at a the banks in fixing the interest rates of loans :
pre-purchase stage to identify goods based on
country of origin. Every time, they have to offer A. RBI repo rate
local alternatives to ensure a fair opportunity to B. 91-day T-bill yield
domestic goods C. 182-day T-bill yield
 In case of non-delivery of goods or services by a D. MCLR
seller on a marketplace platform, which causes
loss to the consumer, the marketplace will be Ans - D
subject to a fallback liability. To ensure complete transparency and standardization,
 All e-commerce entities are required to appoint: RBI mandated the banks to adopt a uniform external
o Grievance Officer to address consumer benchmark within a loan category, effective 1st
grievances, October, 2019. Unlike MCLR which was internal system

Telegram – Indian_Economy_by_Aman_Soni Page 51


for each bank, RBI has offered banks the options to Q5. Choose the correct statements wrt The
choose from 4 external benchmarking mechanisms: Organization of the Petroleum Exporting Countries
 The RBI repo rate (OPEC)
 The 91-day T-bill yield
 The 182-day T-bill yield
 Any other benchmark market interest rate as 1. Algeria, Angola, Saudi Arabia, Indonesia and
developed by the Financial Benchmarks India Qatar are few of the current OPEC members.
Pvt. Ltd. 2. OPEC is headquartered in Dubai, the United
Arab Emirates

The Internal Benchmark Lending Rates are a set of A. 1 only


reference lending rates which are calculated after B. 2 only
considering factors like the bank's current financial C. Both 1 and 2
overview, deposits and non performing assets (NPAs) D. Neither 1 nor 2
etc. BPLR, Base rate, MCLR are the examples of Internal
Benchmark Lending Rate. Ans - D
The Organization of the Petroleum Exporting Countries
Q4. Choose the correct statements. (OPEC) is an intergovernmental organization of 13
countries. Founded on 14 September 1960 in Baghdad
1. The govt fixes the ceiling price of drugs in the by the first five members (Iran, Iraq, Kuwait, Saudi
National List of Essential Medicines (NLEM) Arabia, and Venezuela), it has since 1965 been
2. Prices of scheduled drugs are allowed an headquartered in Vienna, Austria, although Austria is
increase each year by the drug regulator, in line not an OPEC member state.
with the annual Consumer Price Index.
Current OPEC members are Algeria, Angola, Equatorial
Guinea, Gabon, Iran, Iraq, Kuwait, Libya, Nigeria, the
A. 1 only Republic of the Congo, Saudi Arabia, the United Arab
B. 2 only Emirates and Venezuela. Ecuador, Indonesia and Qatar
C. Both 1 and 2 are former OPEC members.
D. Neither 1 nor 2
Q1. Choose the correct statements.
Ans - A
Drug Price Control Orders (DPCO) are issued by the 1. As per the RBI Act, Reserve Bank of India has
Government in exercise of the powers conferred under the sole right to issue banknotes in India with
the Essential Commodities Act, 1955,for enabling the some exceptions
Government to declare a ceiling price for lifesaving 2. The Indian one rupee note is issued by the
medicines to ensure that these medicines are available Reserve Bank
at a reasonable price to the general public.

Price controls are applicable to “Scheduled drugs” or A. 1 only


“Scheduled formulations” e. those medicines which are B. 2 only
listed out in the Schedule I of DPCO, also referred to as C. Both 1 and 2
National List of Essential Medicines (NLEM). As per the D. Neither 1 nor 2
price control order, the NPPA is empowered to fix the
ceiling price of essential medicines and medical devices. Ans - A
Prices of scheduled drugs are allowed an increase each Section 22 of the RBI Act, Reserve Bank of India has the
year by the drug regulator, in line with the annual WPI. sole right to issue banknotes in India. Section 25 states
that the design, form and material of bank notes shall
be such as may be approved by the Central Government

Telegram – Indian_Economy_by_Aman_Soni Page 52


after consideration of the recommendations made by debentures, they provide somewhat higher interest
the Central Board of RBI. rates.

The Reserve Bank in terms of its clean note policy,


provides good quality banknotes to the members of the Q3. Choose the INcorrect statements.
public. The Reserve Bank, in consultation with the
Central Government and other stakeholders, estimates 1. Any software or work that is in Creative
the quantity of banknotes that are likely to be needed Commons licence (CC) can be used and
denomination-wise in a year and places indents with distributed free of cost.
the various currency printing presses for supply of 2. Proprietary software is an example for Creative
banknotes. Commons licence (CC)

The Government of India is responsible for the A. 1 only


designing and minting of coins in various denominations B. 2 only
as per the Coinage Act, 2011.The Indian 1-rupee note C. Both 1 and 2
(Rs 1) is made up of hundred 100 paise as Rs 1 = 100 D. Neither 1 nor 2
paise. Currently, it is the smallest Indian banknote in
circulation and the only one being issued by the Ans - B
Government of India. Creative Commons licence (CC)
There is the Creative Commons licence (CC) which is
public domain: any software or work that is in CC can be
Q2. Choose the correct statements. used and distributed free of cost.
For example, Wikipedia is under CC and hence its
1. Convertible debentures are debentures that can contents can be used freely with the condition that
be converted into equity attribution is made to Wikipedia (this is called ‘Creative
2. Non-convertible debentures generally provide Commons – Attribution-ShareAlike).
higher interest rates than Convertible
debentures
Proprietary License
A. 1 only There is proprietary software which is to be purchased
B. 2 only as a one-time transaction or as yearly licences.
C. Both 1 and 2 A popular example is Microsoft Windows which is
D. Neither 1 nor 2 purchased along with the computer or Microsoft Office
which typically has a yearly licence that has to be
Ans - C renewed upon payment.
Debentures are a sort of long-term debt instrument that
does not have any collateral backing them up. In other
words, debentures are not secured or have no security. Q4. Which among the following are correctly matched

Convertible debentures can be defined as a form of 1. Call Money - borrowing or lending of funds for 1
long-term debt issued by a corporation that may be day
converted into a stock market after a set time. 2. Notice Money - borrowing or lending for more
Convertible debentures are often unsecured bonds or than 2 weeks
loans with no underlying security to back them up. 3. Term money - borrowing and lending for more
than 14 days
Non-convertible debentures as the fixed-income
instruments that are often issued as a public offering by A. 1 only
high-rated firms in order to accrue long-term capital B. 2 only
appreciation. When compared to convertible C. 1 and 2 only
D. 1 and 3 only

Telegram – Indian_Economy_by_Aman_Soni Page 53


Calamine, which it sells at a lower price in Calamine. If a
Ans - D second company buys the drug in Calamine and imports
Banks and Primary Dealers borrow and lend overnight it into Belladonna at a price that is lower than company
or for the short period to meet their short term A’s price, that would be a parallel or grey import.
mismatches in fund positions. This borrowing and
lending is on unsecured basis Q1. Choose the correct statements wrt Domestic
 Call Money refers to the borrowing or lending Systemically Important Banks (D-SIBs)
of funds for 1 day. 1. The system of D-SIBs was adopted in the
 Notice Money refers to the borrowing and aftermath of the 2008 financial crisis
lending of funds for 2-14 days. 2. Every year, the Central Govt releases the list of
 Term money refers to borrowing and lending of all D-SIBs
funds for a period of more than 14 days. Notice
Money is also known as Short Notice Money. A. 1 only
B. 2 only
Q5. Choose the correct statements. C. Both 1 and 2
D. Neither 1 nor 2
1. The TRIPS Agreement contains the term
“compulsory licensing”. Ans - A
2. Parallel imports are imports of counterfeit Domestic Systemically Important Banks (D-SIBs) are
products or illegal copies. financial institutions that are large enough where they
cannot be allowed to fall.Due to the way the D-SIBs
A. 1 only become completely enmeshed in cross-jurisdictional
B. 2 only activities, their complex financial structures, and the
C. Both 1 and 2 lack of other alternatives, they are considered
D. Neither 1 nor 2 systematically important.

Ans - D A failure of any of these banks can lead to systemic and


Compulsory licensing is when a government allows significant disruption to essential economic services
someone else to produce the patented product or across the country and can cause an economic panic. As
process without the consent of the patent owner. a result of their importance, the government is
expected to bail out these banks in times of economic
The agreement allows compulsory licensing as part of distress to prevent widespread harm. Additionally, D-
the agreement’s overall attempt to strike a balance SIBs follow a different set of regulations in relation to
between promoting access to existing drugs and systemic risks and moral hazard issues. The system of D-
promoting research and development into new drugs. SIBs was adopted in the aftermath of the 2008 financial
But the term “compulsory licensing” does not appear in crisis where the collapse of many systematically
the TRIPS Agreement. Instead, the phrase “other use important banks across various regions further fueled
without authorization of the right holder” appears in the financial downturn.
the title of Article 31. Compulsory licensing is only part
of this since “other use” includes use by governments Since 2015, the RBI has been releasing the list of all D-
for their own purposes. SIBs. They are classified into five buckets, according to
their importance to the national economy. In order to
Parallel or grey-market imports are not imports of be listed as a D-SIB, a bank needs to have assets that
counterfeit products or illegal copies. These are exceed 2 percent of the national GDP. The banks are
products marketed by the patent owner (or trademark- then further classified on the level of their importance
or copyright-owner, etc) or with the patent owner’s across the five buckets. The banks need to maintain a
permission in one country and imported into another higher share of risk-weighted assets as tier-I equity.
country without the approval of the patent owner.
For example, suppose company A has a drug patented Q2. Choose the correct statements.
in the Republic of Belladonna and the Kingdom of
Telegram – Indian_Economy_by_Aman_Soni Page 54
1. The main target for payment banks will be RBI has announced the final guidelines for on-tap
MSME with turnover more than 10 crores and licencing of private sector SFBs. These guidelines
high income households. include:
2. Payment banks cannot indulge in lending loans 1.Payments banks can apply for conversion into small
directly finance banks (SFBs) after five years of operation.
2.The promoter of a payments bank is eligible to set up
A. 1 only an SFB, provided that both banks come under the non-
B. 2 only operating financial holding company *(NOFHC)
C. Both 1 and 2 structure.
D. Neither 1 nor 2 *Non-Operative Financial Holding Company (NOFHC)
means a non-deposit taking NBFC referred to in the
Ans - B “Guidelines for Licensing of New Banks in the Private
Payments banks were introduced by the RBI for Sector”1 issued by Reserve Bank, which holds the
promoting financial inclusion and facilitating payments shares of a banking company and the shares of all other
and remittance flows. financial services companies in its group.
They are different from the conventional universal 3.The minimum paid-up capital requirement for SFBs
banks as the Payments banks can concentrate in only has been raised from Rs. 100 crore to Rs. 200 crore.
two types of activities – accepting demand deposits and 4. Primary urban cooperative banks can convert into
facilitating payments. SFBs, provided they comply with the on-tap licencing
The main target for payment banks will be migrant guidelines.
labourers, self employed, low income households etc.
as they will offer low cost savings accounts and Resident individuals/professionals with 10 years of
remittance services so that those who now transact experience in banking and finance; and Companies and
only in cash can take their first step into the formal Societies owned and controlled by residents will be
banking system. eligible as promoters to set up small finance banks.

They can act as Banking Correspondent of another bank Q4. Which among the following is/are the reasons for
and can offer internet banking, sell mutual funds, Bank Nationalization In India :
insurance facilities. They cannot indulge in lending. CRR
is applicable. 1. Failure of many Private Banks
2. Priority sector lending ignored by banks
Q3. Choose the correct statements. 3. To help in Planned Growth of the Economy
4. To increase Rural bank branches
1. Payments banks can apply for conversion
into small finance banks (SFBs) after 10 A. 1, 2 & 3 only
years of operation. B. 1, 2 & 4 only
2. Only Corporate Companies and Societies C. 2, 3 & 4 only
owned and controlled by Indian residents, D. 1, 2, 3 & 4 only
will be eligible as promoters to set up small
finance banks. Ans - D
Reasons for Bank Nationalisation
A. 1 only 1.Banks were unreliable: There were 361 private banks
B. 2 only which "failed" across the country in the period from
C. Both 1 and 2 1947 to 1955, translating to an average of over 40 banks
D. Neither 1 nor 2 per year. This resulted in depositors losing all their
money as they were not offered any guarantee by their
Ans - D respective banks.
RBI Guidelines for Payment Banks’ Small Finance Banks
(SFB) License 2.Conformity with national policy and objectives:
Nationalisation of banks was to sync the banking sector

Telegram – Indian_Economy_by_Aman_Soni Page 55


with the goals of socialism adopted by the Indian At present, 39 Debts Recovery Tribunals (DRTs) and 5
government after independence. Debts Recovery Appellate Tribunals (DRATs) are
functioning across the country. Each DRT and DRAT are
3.Balanced flow of credit: it was aimed at ensuring the headed by a Presiding Officer and a Chairperson
balanced flow of credit to all the productive sectors, respectively. The post of Presiding Officer is equivalent
across various regions and social groups of the country. to a District Judge and the post of Chairperson is
Agriculture and other priority sectors were largely equivalent to that of a High Court Judge.
ignored by these banks.

4.Planned development of the economy: It was Q1. Which of the following class of persons get
envisaged that with nationalisation the proportion of adversely affected during the times of inflation :
bank investments in Government securities would go 1. Debtors
up. This would ensure enough development funds for 2. Creditors
the planned growth of the country 3. Exporters
4. Recipients of social security allowance
Q5. Choose the correct statements
A. 1, 2 & 3 only
1. Debt Recovery Tribunals were established B. 1, 2 & 4 only
under the Banking Regulation Act C. 2, 3 & 4 only
2. The post of Chairperson of Debts Recovery D. 1, 2, 3 & 4 only
Appellate Tribunals (DRATs) is equivalent to
that of a High Court Judge. Ans - C
During periods of rising prices, debtors gain and
A. 1 only creditors lose. When prices rise, the value of money
B. 2 only falls. Though debtors return the same amount of
C. Both 1 and 2 money, they pay less in terms of value of money or
D. Neither 1 nor 2 purchasing power of money. Thus, inflation brings
about a redistribution of real wealth in favour of
Ans - B debtors at the cost of creditors.
Debt Recovery Tribunals were established in 1993 to
facilitate the debt recovery of banks and other financial Inflation discourages exports as domestic sales are
institutions from their customers. DRTs were set up attractive and BOP problems can be caused. Inflation
after the passing of Recovery of Debts due to Banks and may erode the external competitiveness of domestic
Financial Institutions Act (RDBBFI), 1993. Section 3 of products if it leads to higher production costs such as
the RDDBFI Act empowers the Central government to wage increase, higher interest rate.
establish DRTs. Appeals against orders passed by DRTs
lie before Debts Recovery Appellate Tribunal (DRAT). The recipients of transfer payments such as pensions,
unemployment insurance, social security, etc. and
Functions of DRT recipients of interest and rent, live on fixed incomes. All
●DRT enforces provisions of the RDDBFI Act, 1993. such persons lose because they receive fixed payments,
●They also enforce the Securitization and while the value of money continues to fall with rising
Reconstruction of Financial Assets and Enforcement of prices.
Security Interests (SARFAESI) Act, 2002.
●The Tribunal has the powers bestowed with the
District Court. Q2. During the times of high Inflation, the Govt and RBI
●DRTs are fully empowered to pass comprehensive would take the following steps :
orders and can travel beyond the Civil procedure Code
to render complete justice. 1. Decrease in direct taxes
2. Increase in SLR
3. Decrease in CRR

Telegram – Indian_Economy_by_Aman_Soni Page 56


4. Implementation of dishoarding
A. 1 & 3 only Q4. Choose the correct statements.
B. 1 & 4 only
C. 2 & 4 only 1. WPI does not cover services
D. 2, 3 & 4 only 2. WPI does not include indirect taxes
3. CPI includes all distribution costs
Ans – C
A. 1 & 2 only
There are fiscal, monetary, supply side and B. 1 & 3 only
administrative measures to control inflation to ideal / C. 2 & 3 only
optimal rates. D. 1, 2 & 3 only
Fiscal measures include increase in direct taxes(To
reduce disposable income) Ans - D
In the new series of WPI, prices used for compilation do
Monetary measures include increase in interest not include indirect taxes in order to remove impact of
rate(Repo rate, reverse repo rate) and increase in fiscal policy. This is in consonance with international
reserve requirements(SLR,CRR). Open market practices and will make the new WPI conceptually
operations(RBI can sell government securities and suck closer to ‘Producer Price Index’.
out excess liquidity) can stabilize prices under normal WPI is based on wholesale prices for primary articles,
conditions also .Sterilization through government bond administered prices for fuel items and ex-factory prices
transactions as in the case of Market Stabilization Bonds for manufactured products. CPI is based on retail prices,
can also help. which include all distribution costs and taxes.
CPI covers only consumer goods and consumer services
Administrative measures include implementation of while WPI covers all goods including inter-mediate
dishoarding, import of goods causing inflation and anti- goods transacted in the economy.
black marketing measures. Wage and price controls can WPI doesn’t include services while CPI includes services.
also be used.
Q5. Choose the correct statements.
Q3. Choose the correct statements.
1. In an economy, deflation can lead to further
1. RBI adopted CPI as the key measure of inflation. deflation, leading to what is known as a
2. Base year of CPI is 2011-12 inflationary spiral.
3. CPI is released monthly 2. The Philips curve identifies an inverse
relationship between inflation and
A. 1 only unemployment
B. 1 & 2 only
C. 1 & 3 only
D. 2 & 3 only
A. 1 only
Ans - C B. 2 only
In India, CPI and WPI are two major indices for C. Both 1 and 2
measuring inflation. The WPI was the main index for D. Neither 1 nor 2
measurement of inflation in India till April 2014 when
the RBI adopted CPI as the key measure of inflation. Ans - B
The WPI is computed by the office of the Economic Deflation occurs when an economy’s annual headline
Adviser in the Ministry of Commerce and Industry,with inflation indicator – typically the consumer price index –
a two weeks lag period. enters negative territory; De- flation is hard to deal with
Both WPI and CPI are released monthly. because it is self reinforcing, put simply, unless it is
Base year of WPI is 2011-12 And Base year of CPI is stopped early, deflation can breed deflation, leading to
2012-13 what is known as a deflationary spiral.

Telegram – Indian_Economy_by_Aman_Soni Page 57


Ans - B
The Phillips curve is an economic concept developed by In India, the Central Government issues both, treasury
A. W. Phillips stating that inflation and unemployment bills and bonds or dated securities while the State
have a stable and inverse relationship. The theory Governments issue only bonds or dated securities,
claims that with economic growth comes inflation, which are called the State Development Loans (SDLs). G-
which in turn should lead to more jobs and less Secs carry practically no risk of default and, hence, are
unemployment. called risk-free gilt-edged instruments.

Q1. Choose the correct statements. Treasury bills are short term money market
instruments, which are issued by the RBI on behalf of
1. A Certificates of Deposit is generally issued at a the Government of India. The Government of India uses
discount and redeemed at par these funds to meet its short term financial
2. Local Area Banks can issue Certificates of requirements. T-Bills are sovereign zero risk
Deposits only in the district, in which they instruments. They are available in primary and
operate. secondary market & issued at a discount to face value.
At present, the Government of India issues four types of
A. 1 only treasury bills: namely, 14-day, 91-day, 182-day and 364-
B. 2 only day.
C. Both 1 and 2
D. Neither 1 nor 2 Q3. Which of the following is NOT a function of SEBI :

Ans - A 1. Regulating the stock brokers and merchant


After treasury bills, the next lowest risk category bankers.
investment option is the certificate of deposit (CD) 2. Imposing penalties on erring market
which is issued by Scheduled Commercial Banks and intermediaries
Fls. Regional Rural Bank and Local Area Banks cannot 3. Regulating mergers and acquisitions
issue CDs. 4. Enforcing a ban on insider trading

A CD is issued at a discount to the face value, the A. 1, 2 & 3 only


discount rate being negotiated between the issuer and B. 1, 2 & 4 only
the investor. C. All of the above
The maturity period of CDs issued by banks should be D. None of the above
not less than 15 days and not more than one year. The
FIs can issue CDs for a period not less than 1 year and Ans - D
not exceeding 3 years from the date of issue. CDs can be The capital markets in India are regulated by SEBI. It was
issued to individuals or firms. established in 1988 and got statutory status in 1992.

Functions of SEBI
Q2. Choose the INcorrect statements. 1.It regulates the working of stock exchanges and
intermediaries such as stock brokers and merchant
1. Treasury Bills (T-Bills) are issued only by the bankers.
Central Govt. 2.It provides approval for mutual funds, registers FIIs
2. Treasury bills are traded only in the Primary 3. It protects the interest of investors in securities.
market 4.It promotes investor’s education and training of
intermediaries of securities market
A. 1 only 5.It prohibits fraudulent and unfair trade practices.
B. 2 only 6.It imposes monetary penalties on erring market
C. Both 1 and 2 intermediaries and also the erring companies.
D. Neither 1 nor 2 7.It regulates mergers and acquisitions.
8. It conducts enquiries and audit of stock exchanges.

Telegram – Indian_Economy_by_Aman_Soni Page 58


9.It enforces corporate disclosures. Ans - A
10. It enforces a ban on insider trading. Masala Bond is a term used to refer to a financial
11. It introduces a code of conduct for all credit rating instrument through which Indian entities can raise
agencies operating in India. money from overseas markets in the rupee, not foreign
currency.These are Indian rupee denominated bonds
Q4. Choose the correct statements risk-return tradeoff issued in offshore capital markets.

1. It states that the potential return rises with a he rupee denominated bond is an attempt to protect
decrease in risk. issuers from currency risk and instead transfer the risk
2. It implies that, invested money can render to investors buying these bonds. As of now, it is being
higher profits only if the investor will accept a traded only at the London Stock Exchange. Masala
higher possibility of losses. Bonds have been given this name by IFC (International
Finance Corporation).
A. 1 only
B. 2 only
C. Both 1 and 2 Q1. Which of the following are included under External
D. Neither 1 nor 2 commercial Borrowings (ECBs) :
1. Commercial bank loans
Ans - B 2. Buyers' credit
The risk-return tradeoff states that the potential return 3. Suppliers' credit
rises with an increase in risk. Using this principle, 4. Fixed rate bonds
individuals associate low levels of uncertainty with low
potential returns, and high levels of uncertainty or risk
with high potential returns. According to the risk-return A. 1 & 4 only
tradeoff, invested money can render higher profits only B. 2 & 3 only
if the investor will accept a higher possibility of losses. C. 1, 2 & 3 only
D. 1, 2, 3 & 4 only
The risk-return tradeoff is an investment principle that
indicates that the higher the risk, the higher the Ans - D
potential reward. External commercial Borrowings (ECBs): loans in India
To calculate an appropriate risk-return tradeoff, made by non-resident lenders in foreign currency to
investors must consider many factors, including overall Indian borrowers. They are used widely in India to
risk tolerance, the potential to replace lost funds and facilitate access to foreign money by Indian
more. corporations and PSUs (public sector undertakings).
Investors consider the risk-return tradeoff on individual
investments and across portfolios when making ECBs include commercial bank loans, buyers' credit,
investment decisions. suppliers' credit, securitized instruments such as
floating rate notes and fixed rate bonds etc., credit from
official export credit agencies and commercial
Q5. Choose the correct statements. borrowings from the private sector window of
1. Masala Bonds are Indian rupee denominated multilateral financial Institutions such as International
bonds issued in offshore capital markets. Finance Corporation (Washington), ADB, AFIC, CDC, etc.
2. These bonds transfer the currency risk from the ECBs cannot be used for investment in the stock market
investors to the issuers of the bonds or speculation in real estate.

A. 1 only Q2. Choose the correct statements.


B. 2 only
C. Both 1 and 2 1. Depreciation of a currency takes place under
D. Neither 1 nor 2 fixed exchange rate system.
2. Devaluation makes exports more competitive.

Telegram – Indian_Economy_by_Aman_Soni Page 59


Q4. Choose the correct statements.
A. 1 only 1. A high Capital account convertibility makes it
B. 2 only easier for domestic companies to raise funds
C. Both 1 and 2 from foreign markets.
D. Neither 1 nor 2 2. India has full current account and capital
account convertibility
Ans - B
Fixed Exchange Rate: It was followed up to March 1992 A. 1 only
in India & 1$ was approx. equal to Rs. 40 The exchange B. 2 only
rate was fixed by RBI. C. Both 1 and 2
Devaluation: Devaluation is an official lowering of the D. Neither 1 nor 2
value of a country’s currency, by which the RBI formally
sets a new fixed rate with respect to a foreign reference Ans - A
currency Currency convertibility means the freedom to convert
one currency into other internationally accepted
Devaluation makes exports cheaper or makes them currencies. India has full current account convertibility
more competitive. but has partial capital account convertibility.
Revaluation makes exports costly or makes them less
competitive. Capital account convertibility means the freedom to
convert local financial assets into foreign financial assets
Q3. Choose the correct statements. and vice versa at the prevailing rates of exchange.
Capital account convertibility means the freedom to
1. NEER and REER are determined for each foreign convert rupees into foreign currency and vice versa for
currency separately. capital transactions. Capital account convertibility is
2. If NEER has increased over the base year, it considered to be one of the major features of a
shows that it is favorable for the exports developed economy. It helps attract foreign investment.
At the same time, capital account convertibility makes it
A. 1 only easier for domestic companies to tap foreign markets.
B. 2 only
C. Both 1 and 2
D. Neither 1 nor 2 Q5. Choose the INcorrect statements.

Ans - D 1. RBI has signed the currency swap agreement


NEER is the Nominal Effective Exchange Rate, and REER with the Central Bank of Sri Lanka under the
is the Real Effective Exchange Rate. Unlike nominal and SAARC Currency Swap Framework 2019-22
real exchange rates, NEER and REER are not determined 2. Currency Swaps can be used to hedge against
for each foreign currency separately. Rather, each is a exchange-rate risk
single number (usually ex- pressed as an index) that
expresses what is happening to the value of the
domestic currency against a whole basket of currencies. A. 1 only
B. 2 only
Both these measures are useful as benchmarks that give C. Both 1 and 2
an idea of the general movement of the domestic D. Neither 1 nor 2
currency against the rest of the world. If NEER has
become 110 from 100 of the base year, then the value Ans - D
of the domestic currency has appreciated over the Currency swaps are an essential financial instrument
period with respect to that basket of currency. It is not utilized by banks, multinational corporations, and
good for exports. institutional investors. Although these type of swaps
function in a similar fashion to interest rate swaps and
equity swaps, there are some major fundamental

Telegram – Indian_Economy_by_Aman_Soni Page 60


qualities that make currency swaps unique and thus  Ensuring effective use of resources through
slightly more complicated. convergence of various Government Schemes
with Gram Panchayats as the basic unit of
A currency swap involves two parties that exchange a planning.
notional principal with one another in order to gain  Work with a focused micro plan for sustainable
exposure to a desired currency. Following the initial livelihood for every deprived households.
notional exchange, periodic cash flows are exchanged in  Conduct an annual survey on measurable
the appropriate currency. outcomes at Gram Panchayat level to monitor
Swaps can be used to hedge against exchange-rate risk, the progress in the development process across
speculate on currency moves, and borrow foreign rural areas.
exchange at lower interest rates.  Supporting the process of participatory planning
for Gram Panchayat Development Plan (GPDP),
The Reserve Bank of India (RBI) signed the currency which will improve service delivery, enhance
swap agreement with the Central Bank of Sri Lanka citizenship, create pace for an alliance of
under the SAARC Currency Swap Framework 2019-22. people's institutions and groups and improve
Under the agreement, the Central Bank of Sri Lanka can governance at the local level.
make drawals of US Dollar, Euro or Indian Rupee in  Encourages partnerships with network of
multiple tranches up to a maximum of $400 million or professionals, institutions and enterprises to
its equivalent. further accelerate the transformation of rural
livelihoods.

Q1. Choose the correct statements wrt Mission


Antyodaya : Q2. Choose the correct statements.

1. Mission Antyodaya strives to realise the vision 1. NASSCOM is a joint venture of the Central govt
of Poverty-free India by 2022. with Private entities to invest in the
2. It aims at convergence of various Government Transportation sector
Schemes with Gram Panchayats as the basic 2. NASSCOM is focused on development and
unit of planning providing support to the IT BPM sector through
policy advocacy
A. 1 only
B. 2 only
C. Both 1 and 2 A. 1 only
D. Neither 1 nor 2 B. 2 only
C. Both 1 and 2
Ans - C D. Neither 1 nor 2
Adopted in Union Budget 2017-18, Mission Antyodaya is
a convergence and accountability framework aiming to Ans - B
bring optimum use and management of resources NASSCOM, a not-for-profit industry association, is the
allocated by 27 Ministries/ Department of the apex body for the $227 billion dollar IT BPM industry in
Government of India under various programmes for the India, an industry that had made a phenomenal
development of rural areas. It is envisaged as state-led contribution to India’s GDP, exports, employment,
initiative with Gram Panchayats as focal points of infrastructure and global visibility. In India, this industry
convergence efforts. provides the highest employment in the private sector.

The Ministry of Panchayati Raj and the Ministry of Rural Established in 1988 and ever since, NASSCOM’s
Development act as the nodal agents to take the relentless pursuit has been to constantly support the IT
mission forward. Mission Antyodaya strives to realise BPM industry, in the latter’s continued journey towards
the vision of Poverty-free India by 2022. seeking trust and respect from varied stakeholders,
Objectives : even as it reorients itself time and again to remain
Telegram – Indian_Economy_by_Aman_Soni Page 61
innovative, without ever losing its humane and friendly 2. The All-India Household Consumer Expenditure
touch. Survey, is usually conducted by the National
Statistical Office (NSO) every 10 years
NASSCOM is focused on building the architecture
integral to the development of the IT BPM sector A. 1 only
through policy advocacy, and help in setting up the B. 2 only
strategic direction for the sector to unleash its potential C. Both 1 and 2
and dominate newer frontiers. D. Neither 1 nor 2

NASSCOM’s members, 3000+, constitute 90% of the Ans - A


industry’s revenue and have enabled the association to The All-India Household Consumer Expenditure Survey,
spearhead initiatives at local, national and global levels. usually conducted by the National Statistical Office
In turn, the IT BPM industry has gained recognition as a (NSO) every five years, is set to resume this year after a
global powerhouse. prolonged break.
NSO is one of two wings of the Ministry of Statistics
Q3. Choose the INcorrect statements. and Programme Implementation. NSO, also called the
1. Ammonia is used in the manufacturing of Statistics Wing, consists of the Central Statistical Office
nitrogenous fertilizers like urea (CSO), the Computer centre and the National Sample
2. In Green ammonia production, the process of Survey Office (NSSO).
making ammonia is 100% renewable and
carbon-free. NSO is mandated with the following responsibilities:

 It acts as the nodal agency for planned


A. 1 only development of the statistical system in the
B. 2 only country
C. Both 1 and 2  Prepares national accounts as well as publishes
D. Neither 1 nor 2 annual estimates of national product,
government and private consumption
Ans - D expenditure, capital formation, savings,
Ammonia is a chemical which is used mainly in the estimates of capital stock and consumption of
manufacture of nitrogenous fertilizers, like urea and fixed capital
ammonium nitrate, but can be put to other uses too,  Maintains liaison with international statistical
such as to run engines. organizations, such as, the United Nations
Green ammonia production is where the process of Statistical Division (UNSD)
making ammonia is 100% renewable and carbon-free.  It compiles and releases the Index of Industrial
One way of making green ammonia is by using Production (IIP) every month in the form of
hydrogen from water electrolysis and nitrogen ‘quick estimates’; conducts the Annual Survey
separated from the air. These are then fed into of Industries (ASI)
theHaber process (also known as Haber-Bosch), all
powered by sustainable electricity. The Index of Industrial Production (IIP) is an index which
In the Haber process,hydrogen and nitrogen are reacted shows the growth rates in different industry groups of
together at high temperatures and pressures to the economy in a stipulated period of time. The IIP
produce ammonia, NH3. index is computed and published by the Central
Statistical Organisation (CSO) on a monthly basis.
Q4. Choose the correct statements.
Q5. Choose the correct statements.
1. The Index of Industrial Production (IIP) index is
published by the Central Statistical Organisation 1. The Trade Receivables Discounting System
(CSO) (TreDs) is a bills discounting platform for
MSMEs vendors.
Telegram – Indian_Economy_by_Aman_Soni Page 62
2. Only those receivables which are due from the grains, especially wheat during the lean season and
PSU’s, will be allowed to be collected on the thereby moderate the open market prices especially in
platform the deficit regions.

A. 1 only The FCI conducts a weekly auction to conduct this


B. 2 only scheme in the open market using the platform of
C. Both 1 and 2 commodity exchange NCDEX (National Commodity and
D. Neither 1 nor 2 Derivatives Exchange Limited).For transparency in
operations, the Corporation has switched over to e-
Ans - A auction for sale under Open Market Sale Scheme
The Trade Receivables Discounting System (TreDs) is an (Domestic).
electronic platform for facilitating the financing /
discounting of trade receivables of Micro, Small and The State Governments/ Union Territory
Medium Enterprises (MSMEs) through multiple Administrations are also allowed to participate in the e-
financiers. These receivables can be due from auction, if they require wheat and rice outside PDS.
corporates and other buyers, including Government
Departments and Public Sector Undertakings (PSUs). Q2. Choose the correct statements.
Only MSMEs can participate as sellers in TReDS.
1. The NITI Aayog has released a draft battery-
TReDS is a payment system authorised under the swapping policy which promotes battery-as-a-
Payment and Settlement Systems (PSS) Act, 2007. It is a service business model
platform for uploading, accepting, discounting, trading 2. The policy also proposes to assign a UIN to
and settling invoices / bills of MSMEs and facilitating swappable batteries at the manufacturing stage
both receivables as well as payables factoring (reverse to help track and monitor them
factoring). MSME sellers, corporate and other buyers,
including Government Departments and PSUs, and A. 1 only
financiers (banks, NBFC-Factors and other financial B. 2 only
institutions, as permitted) are direct participants in the C. Both 1 and 2
TReDS and all transactions processed under this system D. Neither 1 nor 2
are ‘”without recourse” to MSMEs.
Ans - C
The NITI Aayog has released a draft battery-swapping
Q1. Choose the correct statements. policy targeted at electric two- and three-wheelers as
1. Food Corporation of India sells surplus stocks of the government think tank aims to expedite large-scale
wheat and rice under Open Market Sale Scheme adoption of EVs.
(Domestic) at predetermined prices through e-
auction Under this, it has proposed offering incentives to
2. Only bulk consumers/private traders are electric vehicles (EVs) with swappable batteries,
allowed to participate in the auction. subsidies to companies manufacturing swappable
batteries, a new battery-as-a-service business model,
A. 1 only and standards for interoperable batteries, among other
B. 2 only measures.
C. Both 1 and 2
D. Neither 1 nor 2 Draft Battery Swapping Policy - Key Proposals :
 Rationalizing taxes on battery
Ans - A  Incentivization for swapping enabled vehicles
Food Corporation of India sells surplus stocks of wheat  Terms of contracts for battery providers
and rice under Open Market Sale Scheme (Domestic) at  Unique identification number (UIN)
predetermined prices through e-auction in the open
market from time to time to enhance the supply of food Q3. Choose the INcorrect statements.
Telegram – Indian_Economy_by_Aman_Soni Page 63
however it does not go by individual cases. The FATF
1. IMF Quotas determine a member’s share in Plenary is the decision making body of the FATF. It
general allocation of SDR’s. meets three times per year.
2. Quotas determine the maximum amount of
financing a member can obtain from IMF under Lists under FATF:
normal access
Grey List: Countries that are considered safe haven for
A. 1 only supporting terror funding and money laundering are put
B. 2 only in the FATF grey list. This inclusion serves as a warning
C. Both 1 and 2 to the country that it may enter the blacklist.
D. Neither 1 nor 2
Black List: Countries known as Non-Cooperative
Ans - D Countries or Territories (NCCTs) are put in the blacklist.
Each IMF member country is assigned a quota, or These countries support terror funding and money
contribution, that reflects the country’s relative size in laundering activities. The FATF revises the blacklist
the global economy. Each member’s quota also regularly, adding or deleting entries.
determines its relative voting power as well as its
borrowing capacity. Quotas are denominated in Special Q5. In which of the following transactions, can an
Drawing Rights (SDRs), escrow account be used, to protect the interest of both
The IMF’s Board of Governors conducts general reviews the parties :
of quotas at regular intervals (no more than five years
apart). Any changes in quotas must be approved by an 1. Transfer of shares
85 percent majority of the total voting power, and a 2. Transfer of Money
member’s own quota cannot be changed without its 3. Real Estate
consent. The two main issues addressed in a general 4. Project financing transactions
review of quotas are
(i) the size of an overall quota increase and A. 1 & 3 only
(ii) the distribution of the increase among the members. B. 1 & 4 only
Presently, India holds 2.75% of SDR quota, and 2.63% of C. 1, 2 & 3 only
votes in the IMF D. All of the above

Q4. Choose the correct statements. Ans - D


Escrow is the use of a third party capable of holding
1. The FATF assesses the strength of a country’s assets on behalf of two parties who are in the process
anti-money laundering and anti-terror financing of completing a transaction. The asset could be money,
frameworks funds, stocks etc. The third-party holds these, often
2. Countries which are known as Non-Cooperative called the escrow agent, until instructions regarding
Countries or Territories (NCCTs) are put in the disbursement are received, or as per predefined
Grey List. timeline. Thus, an escrow account is the third party
account which holds the asset until the conclusion of a
A. 1 only specific event or time.
B. 2 only
C. Both 1 and 2 Thus in a transaction where escrow account is used, the
D. Neither 1 nor 2 following activities take place,
 The buyer and the seller agrees on the terms
Ans - A and conditions
Financial Action Task Force is an inter-governmental  The buyer pays the amount into the escrow
body established in 1989 during the G7 Summit in Paris. account
The FATF assesses the strength of a country’s anti-  The seller performs the service/ships the goods
money laundering and anti-terror financing frameworks,  The buyer receives the same
Telegram – Indian_Economy_by_Aman_Soni Page 64
 The escrow releases the amount in favour of on increasing or decreasing the production) etc. Since
the buyer they do not distort trade there is no limit on them.

Q1. When the Chinese govt is subsidizing its Amber box subsidies are also called Aggregate Measure
smartphone exporters to export heavily to the Indian of Support (AMS). The AMS means annual level of
market, the Indian govt imposes a higher than normal support (subsidies) expressed in monetary terms.
duty called as : Example of product specific subsidies is MSP and for
non-product specific subsidy is fertilizer,power etc
A. Integrated GST
B. Customs Duty
C. Countervailing Duty Q3. Choose the INcorrect statements.
D. Anti dumping Duty
1. TFA (Trade Facilitation Agreements) aims to
Ans - C simplify customs rules across all international
Countervailing Duties are imposed when on a detailed borders for faster movement of goods and
enquiry it is found out that the exporting country is services.
giving subsidies to harm the industry of another 2. India has signed TFA of WTO but not yet ratified
country. In this case the importing country can impose it.
higher than normal duties to protect its domestic
economy, which are called countervailing duties. A. 1 only
B. 2 only
C. Both 1 and 2
Q2. Choose the correct statements. D. Neither 1 nor 2

1. At 2015, Nairobi Round both developed and Ans - B


developing countries have committed to TFA (Trade Facilitation Agreement)
remove export subsidies. This agreement aims to simplify customs rules across all
2. Pest control measures, direct payment to international borders for faster movement of goods and
farmers without conditions, are all examples of services.
Amber box subsidies Under TFA, each member shall carry out the following
reforms
A. 1 only 1. Publish its laws, rules and regulations related to
B. 2 only import and export procedures, tax rates, penalties in
C. Both 1 and 2 easily accessible manner
D. Neither 1 nor 2 2. Make available on internet the forms and documents
required for import and export and contact information
Ans - A on enquiry points
At 2015, Nairobi Round both developed and developing 3. Adopt procedures allowing the option of electronic
countries have committed to remove export subsidies. payment of duties, taxes, fees and charges.
Developed countries will remove some export subsidies The TFA which aims at simplifying customs procedure,
by 2015 and remaining by 2018, while developing increasing transparency and reducing transactions cost
countries will remove few by 2018 and remaining by has been pushed by developed countries to bolster
2023. their sagging economies through unhindered
international trade. TFA will reduce red tapism in
Green box subsidies relate to research and international trade and will ease trade relations
development and infrastructure like universities, pest between countries. India ratified TFA in 2016 and it
control measures, roads in rural areas, direct payment entered into force in 2017.
to farmers with decoupling effect (i.e. payment to the
farmers will be made but no condition will be attached
Q4. Choose the correct statements.

Telegram – Indian_Economy_by_Aman_Soni Page 65


(equivalent to about US$943 billion) have been
1. The membership of the IMF is a necessary allocated. This includes the largest-ever allocation of
condition precedent to the membership of the about SDR 456 billion approved on August 2, 2021
World Bank. (effective on August 23, 2021). This most recent
2. The current managing director (MD) and allocation was to address the long-term global need for
chairperson of IMF is David R. Malpass reserves, and help countries cope with the impact of the
COVID-19 pandemic. The value of the SDR is based on a
A. 1 only basket of five currencies—the U.S. dollar, the euro, the
B. 2 only Chinese renminbi, the Japanese yen, and the British
C. Both 1 and 2 pound sterling.
D. Neither 1 nor 2
The SDR is neither a currency nor a claim on the IMF.
Rather, it is a potential claim on the freely usable
Ans - A currencies of IMF members. SDRs can be exchanged for
The membership of the IMF is a necessary condition these currencies. Baskets of currencies determine the
precedent to the membership of the World Bank. Thus, value of SDR.The value of the SDR was initially defined
India’s membership of the IMF has entitled her to be a as equivalent to 0.88 grams of fine gold—which, at the
member of the World Bank and its affiliates viz., time, was also equivalent to one U.S. dollar. Therefore,
International Finance Corporation (IFC) and SDR is also called Paper Gold.
International Development Association (IDA).
Currencies included in the SDR basket have to meet two
criteria: the export criterion and the freely usable
David R. Malpass was selected as 13th President of the criterion. A currency meets the export criterion if its
World Bank Group by its Board of Executive Directors issuer is an IMF member or a monetary union that
on April 5, 2019. includes IMF members, and is also one of the top five
The current managing director (MD) and Chairwoman of world exporters. For a currency to be determined
the IMF is Bulgarian economist Kristalina Georgieva, “freely usable” by the IMF, it has to be widely used to
who has held the post since October 1, 2019. Gita make payments for international transactions and
Gopinath, who previously served as Chief Economist widely traded in the principal exchange markets.
was appointed as First Deputy Managing Director,
effective January 21, 2022. Q1. Choose the correct statements.

Q5. Choose the correct statements. 1. India is the largest producer and exporter of
jute in the world.
1. The SDR is neither a currency nor a claim on the 2. India lags behind Bangladesh in producing
IMF superior quality jute fibre
2. SDR is also called Paper Gold.
3. A currency meets the export criterion if its A. 1 only
issuer is an IMF member and is also one of the B. 2 only
top five world exporters. C. Both 1 and 2
D. Neither 1 nor 2
A. 1 & 2 only
B. 1 & 3 only Ans - B
C. 2 & 3 only As per the Food and Agriculture Organisation (FAO),
D. 1, 2 & 3 only India is the largest producer of jute followed by
Bangladesh and China. However, in terms of acreage
Ans - D and trade, Bangladesh takes the lead accounting for
The SDR is an international reserve asset, created by the three-fourth of the global jute exports in comparison to
IMF in 1969 to supplement its member countries’ India’s 7%.
official reserves. To date, a total of SDR 660.7 billion

Telegram – Indian_Economy_by_Aman_Soni Page 66


This can be attributed to the fact that India lags behind 2. In Agro ecology, we can adopt mass production
Bangladesh in producing superior quality jute fibre due & produce large quantities of food with the help
to infrastructural constraints related to retting, farm of fertilizers.
mechanisation, lack of availability of certified seeds and
varieties suitable for the country’s agro-climate. What A. 1 only
also does not bode well for India is that jute acreage B. 2 only
competes with crops as paddy, maize, groundnut, and C. Both 1 and 2
sesame. The increased availability of synthetic D. Neither 1 nor 2
substitutes is further bothering the demand for jute
domestically. Ans - A
Agroecology is farming that “centers on food
production that makes the best use of nature’s goods
Q2. Choose the correct statements. and services while not damaging these resources.”
1. The government can fix the upper ceiling for the Farming thrives when it works with local ecosystems,
selling price of raw jute for example, improving soil and plant quality through
2. The Jute Packaging Material Act, 1987, available biomass and biodiversity, rather than battling
mandate that 100% production of foodgrains nature with chemical inputs. Agroecological farmers
must be packaged in jute bags. seeks to improve food yields for balanced nutrition,
strengthen fair markets for their produce, enhance
A. 1 only healthy ecosystems, and build on ancestral knowledge
B. 2 only and customs.*
C. Both 1 and 2
D. Neither 1 nor 2 Promoters of agroecology strive to nurture a healthy
landscape in which to grow the world’s food and fiber.
Ans - C They are guided by an ethos of bio and cultural diversity
A September 30, 2021, notification mandated that no featuring small farmer-centered applied research and
entity would be allowed to purchase or sell raw jute at a policies that protect their livelihoods. Worldwide,
price exceeding ₹6,500 per quintal scientists, grassroots organizations, NGOs, consumers,
universities, and public agencies are working with
Bulk of the final jute produced is used for packaging farmers to construct sustainable and nutritious food
purposes. The provisions of the Jute Packaging Material systems based in agroecology.
(Compulsory use in Packing Commodities) Act, 1987 or
the JPM Act mandate that 100% production of The main disadvantage is the limited use of land, which
foodgrains and 20% sugar production must be packaged makes it difficult to produce large quantities of food.
in jute bags. The share of jute used for sacks, therefore, Therefore mass production is not possible. Since the use
increased from 67.9% for the TE (TE: Triennium Ending of machines is minimal or eliminated, it takes more time
or three years ending) 2010-11 to 78.3% in TE 2020-21. and people to successfully produce plants, which slows
On the other hand, jute used for manufacturing other down the production.
products (such as furnishing materials, fashion
accessories, floor coverings or varied applications in Q4. Choose the correct statements.
paper and textile industries) has declined from 15.5% to
9.7% during the same period. 1. Rice fortification is the practice of increasing the
content of essential micronutrients and to
Q3. Choose the correct statements. improve the nutritional quality of the rice.
2. Currently in India, the Fortified rice contains
1. Agro-ecology is a holistic approach, which uses only Vitamin A & Vitamin B12
ecological and social concepts for sustainable
agriculture A. 1 only
B. 2 only
C. Both 1 and 2

Telegram – Indian_Economy_by_Aman_Soni Page 67


D. Neither 1 nor 2
India imports about a third of its urea requirements.
Ans - A During April 2021-January 2022, the first 10 months of
Fortification is the addition of key vitamins and minerals this fiscal year, India produced 21 million tonnes of urea
such as iron, iodine, zinc, Vitamin A & D to staple foods and imports were at about 7.2 million tones.
such as rice, milk and salt to improve their nutritional
content. Q1. Choose the correct statements.
Rice fortification is done using extrusion technology.
Fortified rice kernels are produced from a mixture using 1. The Central Road and Infrastructure Fund falls
an extruder machine. These kernels are then blended under the Ministry of Road Transport
with regular rice to produce fortified rice. The 2. The fund comprises of a cess imposed along
micronutrients that are used to blend included iron, with excise duty on petrol and diesel.
folic acid and Vitamins D and B-12.
A. 1 only
According to the FSSAI norms, 1 kg fortified rice shall B. 2 only
contain iron (28mg-42.5mg), folic acid (75-125 C. Both 1 and 2
microgram) and Vitamin B-12 (0.75-1.25 microgram). In D. Neither 1 nor 2
addition, rice may also be fortified with micronutrients,
singly or in combination, at the level– zinc(10mg-15mg), Ans - B
Vitamin A (500-750 microgram RE), Vitamin B1 (1mg- The Central Road and Infrastructure Fund (earlier
1.5mg), Vitamin B2 (1.25mg-1.75mg), Vitamin B3 known as Central Road Fund) was established in 2000
(12.5mg-20mg) and Vitamin B6 (1.5mg-2.5mg) per Kg. under Central Road Fund Act, 2000. The fund comprises
of a cess imposed along with excise duty on petrol and
diesel. The administrative control of Central Road and
Infrastructure Fund (CRIF) falls under the Ministry of
Q5. Choose the correct statements. Finance. Earlier it was under the domain of Ministry of
1. Urea, Phosphate and Potash fertilizer prices are Road Transport and Highways.
controlled and MRP is fixed by the govt
2. India imports three-fourths of its urea This Act allows to, Levy and collect a cess for
requirements to meet the domestic demand. development and maintenance of NHs and
improvement of safety at railway crossings, and Levy
A. 1 only and collect a duty of excise and duty of customs on
B. 2 only motor spirit commonly known as petrol, high speed
C. Both 1 and 2 diesel oil. Fund shall be utilised for Development and
D. Neither 1 nor 2 maintenance of national highways; other State roads
including roads of inter-State and economic importance;
Ans - D Development of the rural roads; Construction of roads
In India, Urea is the only controlled fertilizer, which is either under or over the railways; Erection of safety
sold at statutory notified uniform sale price. The works at unmanned railroad crossings, new lines,
Phosphate and Potash fertilizes are under a conversion of existing standard lines into gauge lines
decontrolled regime and are sold at indicative and electrification of rail lines; and Undertaking other
maximum retail prices (MRPs). infrastructure projects.
The statutorily notified sale price and indicative MRP
are generally less than the cost of production of the Q2. Choose the INcorrect statements.
irrespective manufacturing unit. The difference 1. Bharat series of “BH” series of registration of
between the cost of production and the selling vehicles, is currently applicable only for
price/MRP is paid as subsidy/concession to commercial vehicles
manufacturers under the New Pricing Scheme (NPS) for 2. BH-series will be available on voluntary basis to
Urea units and the Concession Scheme for decontrolled Defense personnel, Government employees etc.
Phosphatic and Potassic fertilizers.

Telegram – Indian_Economy_by_Aman_Soni Page 68


A. 1 only Hence, whatever is not declared as a “minor mineral”
B. 2 only may be treated as the major mineral.
C. Both 1 and 2
D. Neither 1 nor 2 In addition to the minor minerals specified in Section
3(e) of the MMDR Act, the Central Government has
Ans - A declared the following minerals as minor minerals:
Ministry of Road Transport Highways(MoRTH) has  boulder,
notified Bharat series of “BH” series of registration to  shingle,
facilitate free movement of personal vehicles across  chalcedony pebbles used for ball mill purposes
States/UTs of India upon relocation to a new State/UT only,
 lime shell, kankar and limestone used in kilns
Under section 47 of Motor Vehicles Act, 1988, a person for manufacture of lime used as building
is allowed to keep vehicle for not more than 12 months material,
in any state other than the state where the vehicle is  murrum,
registered. BH-series will be available on voluntary basis  brick-earth,
to Defense personnel, Government employees and  fuller’s earth,
private sector companies/organizations, which have  bentonite,
their offices in four or more States/UTs.  road metal,
 reh-matti,
 slate and shale when used for building material,
Q3. Which of the following are classified as Minor
 marble,
Minerals in India :
 stone used for making household utensils,
1. Lignite
2. Marble  quartzite and sandstone when used for
3. Ordinary sand purposes of building or for making road metal
4. Saltpeter and household utensils,
 saltpeter and
A. 1, 2 & 3 only  ordinary earth (used or filling or leveling
B. 1, 2 & 4 only purposes in construction or embankments,
C. 2, 3 & 4 only roads, railways, building)
D. 1, 2, 3 & 4 only
Q4. Choose the correct statements.
Ans - C
According to section 3(e) of the Mines and Minerals 1. Integrated Land Information Management
(Development and Regulation) Act, 1957 “Minor System (ILIMS) is developed under The Digital
Minerals” means building stones, gravel, ordinary clay, India Land Record Modernization Programme
ordinary sand other than sand used for prescribed 2. ILIMS system contains information on parcel
purposes, and any other mineral which the Central ownership, land use, taxation, land value etc.
Government may, by notification in the Official Gazette,
declare to be a minor mineral. (For the purposes of this A. 1 only
Act, the word "minerals” includes all minerals except B. 2 only
mineral oils- natural gas and petroleum) C. Both 1 and 2
D. Neither 1 nor 2
Major minerals are those specified in the first schedule
appended in the Mines and Minerals (Development and Ans - C
Regulation) Act, 1957 (MMDR Act 1957) and the The Digital India Land Record Modernisation
common major minerals are Lignite, Coal, Uranium, iron Programme was approved by the Cabinet on 21st
ore, gold etc. It may be noted that there is no official August, 2008. For modernization of land records system
definition for “major minerals” in the MMDR Act. in the country, a modified programme, viz., the National
Land Records Modernization Programme (NLRMP), now

Telegram – Indian_Economy_by_Aman_Soni Page 69


renamed as Digital India Land Records Modernization multimedia, multi-ways connectivity to the farmers with
Programme (DILRMP), has been formulated. It is being the latest agricultural technologies, knowledge base and
implemented by the Department of Land Resources the pool of large number the subject matter experts.
(Ministry of Rural Development).

It attempts to build upon the commonalities that exist Q1. Choose the correct statements.
in the arena of land records in various States to develop
an appropriate Integrated Land Information 1. The Labour Force only consists of the people
Management System (ILIMS) across the country, on aged 21 to 59 years, who are currently
which different States can also add State-specific needs employed
as they may deem relevant and appropriate. 2. The main reason for India’s Labour Force
Participation Rate being low is the low level of
Integrated Land Information Management System female LFPR.
(ILIMS): The system contains information on parcel
ownership, land use, taxation, location boundaries, land A. 1 only
value, encumbrances and many more. B. 2 only
C. Both 1 and 2
D. Neither 1 nor 2
Q5. Choose the INcorrect statements.
Ans - B
1. Kisan Sarathi is a digital platform to facilitate Labour Force: It consists of the people aged 15 and up
farmers to get right information at right time in to 59 years (working age population), who are
their desired language employed and unemployed (people who donot have
2. It helps farmers with the latest agricultural jobs but are looking for work) and those who are in this
technologies, knowledge base and the pool of age group but not looking for work (like students,
large number the subject matter experts. housewives or even people who do not want to work).
People below 15 years and above 59 years are
dependents and are not part of labour force or working
A. 1 only age population.
B. 2 only The main reason for India’s Labour Force Participation
C. Both 1 and 2 Rate being low is the abysmally low level of female
D. Neither 1 nor 2 LFPR.

Ans - D Q2. Choose the correct statements.


Kisan Sarathi - System of Agri-information Resources
Auto-transmission and Technology Hub Interface, ICAR. 1. Tourism sector in the hill stations of Himachal
It is jointly launched by Ministry for Agriculture and Pradesh experience Seasonal Unemployment
Farmers' Welfare and Union Ministry of Electronics & 2. A high number of post graduate applications,
Information Technology. for the job which needs 12th pass as the
Powered by: Interactive Information Dissemination qualifying criteria, is an example of Disguised
System (IIDS), Digital India Corporation (DIC), Ministry of Unemployment
Electronics and Information Technology (MeitY), Govt.
of India A. 1 only
B. 2 only
C. Both 1 and 2
“Kisan Sarathi” an Information Communication and D. Neither 1 nor 2
Technology (ICT) based interface solution with an
ultimate goal of: An intelligent online platform for Ans - A
supporting agriculture at local niche with national Seasonal Unemployment: This kind of unemployment is
perspective. Which is intended to provide a seamless, expected to occur at certain parts of the year. For

Telegram – Indian_Economy_by_Aman_Soni Page 70


example, the people at hill stations may experience
seasonal unemployment during the winter months A. Seasonal Unemployment
because less people will visit these areas during this B. Structural Unemployment
time. Another case could be the seasonal C. Underemployment
unemployment in agriculture D. Frictional Unemployment

Underemployment: This term can be used in multiple Ans - D


connotations but one of the major usages is to Frictional/Transitional Unemployment: This kind of
showcase a situation where a person with high skills unemployment occurs when a person leaves/loses a job
works in a low wage and low skills job.eg a Post and starts looking for another one. This search for a job
graduate scholar working as a peon. A high number of may take a considerable amount of time resulting in
post graduate applications, for the job which needs frictional unemployment.
12th pass as the qualifying criteria, is an example of
Underemployment. Q5. Choose the correct statements.

1. The Ministry of Labour & Employment has


Q3. Choose the correct statements wrt The Quarterly developed eSHRAM portal
Employment Survey (QES) 2. Current members of ESIC or EPFO are NOT
eligible to get registered under the portal.
1. The survey covers firms with 10 or more
workers in nine selected sectors. A. 1 only
2. As per latest QES, the Education sector B. 2 only
employed the highest number of workers C. Both 1 and 2
D. Neither 1 nor 2
A. 1 only
B. 2 only Ans - C
C. Both 1 and 2 The Ministry of Labour & Employment has developed
D. Neither 1 nor 2 eSHRAM portal for creating a National Database of
Unorganized Workers (NDUW) for optimum realization
Ans - A of their employability and extend the benefits of the
The Quarterly Employment Survey (QES) is part of the social security schemes to them. Candidates who
All-India Quarterly Establishment-based Employment register for E Shram will get a Unique Identification
Survey (AQEES). It covers establishments employing 10 Number (UAN) Card.
or more workers in the organised segment in 9 sectors.
The survey covered manufacturing, construction, trade,
transport, education, health, accommodation and
restaurants, IT/BPOs and financial services that
accounted for 85% of the employment in units with 10
or more workers

Over four lakh jobs were created in firms with 10 or


more workers in nine selected sectors in October-
December 2021, according to the Labour and
Employment Ministry’s third Quarterly Employment
Survey (QES)

Q4. As a part of ‘The Great Resignation’ during the


It is the first-ever national database of unorganised
pandemic, Mr. Prateek left his job and he is currently
workers including migrant workers, construction
looking for a new job from the past few months. The
workers, gig and platform workers, etc. Any worker who
type of Unemployment Mr.Prateek is experiencing is :

Telegram – Indian_Economy_by_Aman_Soni Page 71


is a home based-worker, self-employed worker or a 3. Additional excise duty (road and infrastructure
wage worker in the unorganised sector including a cess) and
worker in the organised sector who is not a member of 4. Agriculture and infrastructure development
ESIC or EPFO or not a Govt. employee is called an cess.
Unorganised Worker.
A. 1, 2 & 3 only
Who can register in eShram (NDUW) Portal? B. 1, 2 & 4 only
C. 2, 3 & 4 only
Any individual satisfying following condition can register D. 1, 2, 3 & 4 only
on the portal:
 An unorganised worker (UW). Ans - D
 Age should be between 16-59 years. At present, Centre levies the excise duty on crude oil
 Not a member of EPFO/ESIC or NPS (Govt. which is a specific tax i.e. charged on the number of
funded) units sold irrespective of the prices.

Q1. Which of the following components are included in The excise duty charged by the Centre, comprises of
the retail price of Petrol/ diesel? four elements —
1. Base price •Basic excise duty,
2. Centre excise duty •Special excise duty,
3. The State GST •Additional excise duty (road and infrastructure cess)
4. Agriculture and infrastructure development and
cess •Agriculture and infrastructure development cess.

A. 1, 2 & 3 only
B. 1, 2 & 4 only Q3. Choose the correct statements.
C. 2, 3 & 4 only
D. 1, 2, 3 & 4 only 1. India is the second largest producer of coal in
the world
Ans - B 2. The gross calorific value (GCV) of imported coal
Components of Fuel prices: is lower than that of domestic coal
 The base price, including the freight
 The Centre charges excise A. 1 only
duty on the base price B. 2 only
 The dealer’s commission C. Both 1 and 2
 The States levy value-added D. Neither 1 nor 2
tax (VAT) on the cost of
petrol and diesel, including Ans - A
excise duty and dealer’s The Power Ministry has asked the Central
commission. Electricity Authority (CEA) to create a
standard operating procedure (SOP) for
Five other petroleum products viz. blending of domestic and imported coal
crude oil, high speed diesel, motor used at thermal power plants (TPPs) as
spirit (petrol), natural gas, and the Coal supply at thermal plants is under
aviation turbine fuel lie outside the pressure due to high electricity demand.
GST.
The pressure on domestic supply for the key commodity
Q2. Which of the following are included in the Centre’s forced the government to increase the percentage of
Excise duty on Crude oil : imported coal in the domestic mix to 10 per cent from 4
1. Basic excise duty, per cent in December 2021.
2. Special excise duty,
Telegram – Indian_Economy_by_Aman_Soni Page 72
India imports coal from Indonesia, South Africa and to platform aggregators like Amazon and Walmart-
Australia. The gross calorific value (GCV) of imported owned Flipkart, who have thus far monopolised India’s
coal is in the range of 5700-6300 k Cal per kg while that e-commerce landscape.
of domestic coal ranges from 2900-4200 k Cal per kg.
ONDC is incorporated as a private non-profit (Section-8)
India has the fifth largest coal reserves in the world. company to establish the public digital infrastructure
India is the second largest producer of coal in the world, needed to expand access to India’s e-commerce
after China. ecosystem – pegged as a US$200 billion opportunity by
2027. Since ONDC will not be following a platform-
Q4. Which of the following act as an indicator of centric model, the Indian government hopes it will
growing external vulnerability that the Indian Economy democratize the country’s online market for all buyers
is facing : and sellers, irrespective of their size, so that millions of
1. Capital outflows small retailers and mom-and-pop (kirana) stores get an
2. Reducing Import Cover equal opportunity.
3. Imported Inflation
4. Currency depreciation Various public and private sector banks have already
acquired stakes in ONDC. Now in its pilot phase, ONDC
A. 1 & 3 only has been rolled out in five cities – Delhi NCR, Bengaluru,
B. 1 & 4 only Bhopal, Shillong, and Coimbatore. It will be expanded to
C. 2, 3 & 4 only 100 cities by October 2022.
D. 1, 2, 3 & 4 only

Ans - D Q1. In which of the following cases would RBI increase


Indicators of growing external vulnerability: its Repo Rate :
 Capital outflows 1. Global Supply chain disruptions leading to
 Reducing forex and import cover imported inflation
 Imported Inflation and related currency 2. Policy normalization by the Federal Bank of USA
depreciation causing capital outflows
 Rising current account deficit
A. 1 only
Q5. Choose the correct statements wrt Open Network B. 2 only
for Digital Commerce (ONDC) C. Both 1 and 2
D. Neither 1 nor 2
1. It was launched as a joint venture between
Amazon and Flipkart Ans – C
2. It will provide level playing field for e-commerce
operators and widen the digital market access The RBI’s rate setting panel on Wednesday announced
for small businesses and traders an ‘off-cycle’ increase in benchmark interest rates. Rise
in Repo Rate– by 40 basis points to 4.4% with
A. 1 only immediate effect.
B. 2 only
C. Both 1 and 2 Reasons :
D. Neither 1 nor 2  Russia’s invasion of Ukraine and the subsequent
western sanctions on Moscow led to supply
Ans - B chain disruption and cost push inflation.
India is keen to level the playing field for e-commerce  Policy normalization by the Federal Bank of USA
operators and widen the digital market access for causing capital outflows
millions of small businesses and traders in the country.  Heightening the risks of imported inflation due
To this end, the government has established the Open to new COVID infections
Network for Digital Commerce (ONDC) as an alternative
Telegram – Indian_Economy_by_Aman_Soni Page 73
Q2. Choose the INcorrect statements. NRI investments on a repatriation basis in Indian
companies are considered as FDIs, and are subject to
1. The GST compensation was to be calculated by regulations and caps.
assuming a 14% yoy growth over revenues in Investments by NRIs on a non-repatriation basis as
2015-16 from the State taxes subsumed in GST stipulated under Schedule IV of Foreign Exchange
2. GST compensation cess is imposed on tobacco Management (non-debt instruments) Rules 2019 are
products & certain passenger motor vehicles deemed to be domestic investment at par with the
investments made by residents.
A. 1 only
B. 2 only Q4. “World Press Freedom” Index is released by
C. Both 1 and 2
D. Neither 1 nor 2 A. Reporters Without Borders
B. Fraser Institute
Ans - D C. Doctors Without Borders
The GST Compensation Act, 2017 guaranteed states D. Varieties of Democracy (V-Dem) Institute
that they would be compensated for any revenue
shortfall below 14% growth (base year 2015-16) for the Ans - A
first five years ending 2022. India’s ranking in the 2022 World Press Freedom Index
has fallen to 150 out of 180 countries, according to the
The compensation was to be calculated by assuming a latest report released by the global media watchdog,
14% year-on-year growth over revenues in 2015-16 Reporters Without Borders (RSF). In last year’s report,
from the State taxes subsumed in GST, and remitted India was ranked 142.
from a compensation cess fund.
The top three positions for countries with the highest
A GST compensation fund is created from which the press freedom were taken by the Nordic trio of Norway
state would be paid the shortfall every two months by (a score of 92.65), Denmark (90.27) and Sweden
the Centre . This corpus is funded through a (88.84).
compensation cess that is levied on so-called ‘demerit’
goods. The items are pan masala, cigarettes and Q5. Choose the correct statements
tobacco products, aerated water, caffeinated
beverages, coal and certain passenger motor vehicles. 1. Standing Deposit Facility (SDF) is an additional
tool for absorbing liquidity without any
Q3. Choose the correct statements. collateral
2. SDF has replaced the fixed rate reverse repo
1. NRI investments that are repatriable are (FRRR) as the floor of the LAF corridor.
considered FDI
2. NRI investments that are non-repatriable A. 1 only
investments are considered domestic B. 2 only
investment. C. Both 1 and 2
D. Neither 1 nor 2
A. 1 only
B. 2 only Ans – C
C. Both 1 and 2 Standing Deposit Facility (SDF) –The SDF has its origins
D. Neither 1 nor 2 in a 2018 amendment to the RBI Act and is an additional
tool for absorbing liquidity without any collateral. It has
Ans - C replaced the fixed rate reverse repo (FRRR) as the floor
NRI investments that are repatriable are considered FDI of the LAF corridor.
while non-repatriable investments are considered
domestic investment. Thus, the LAF corridor will be symmetric around the
policy repo rate with the MSF rate as the ceiling and the

Telegram – Indian_Economy_by_Aman_Soni Page 74


SDF rate as the floor with immediate effect. Access to D. Neither 1 nor 2
SDF and MSF will be at the discretion of banks, unlike Ans - A
repo/reverse repo, OMO and CRR which are available at Domestic banks‘ share in the overall commercial credit
the discretion of the Reserve Bank. plunged to a low of 34% in FY21 from 56% in FY11 partly
due to the pandemic and more because companies
were moving away from banks for funds. The share of
non-banks in commercial credit has more than doubled
Q1. Choose the correct statements. to 44% while that of foreign banks rose to 22% in FY21
according to a report by BofA Global Research.
1. Quantitative easing reduces the money supply
in the Economy The credit intensity of NBFCs, measured by NBFC credit
2. Tapering is the gradual slowing of the pace of as a ratio of GDP, has been rising consistently and stood
the USA Fed Bank’s large scale asset purchases. at 13.7 at end March 2021. Industry remained the
largest recipient of credit extended by the NBFC sector,
A. 1 only followed by retail loans and services.
B. 2 only
C. Both 1 and 2 Q3. Choose the correct statements.
D. Neither 1 nor 2
1. Rising interest rate will lead to a raise in the
Ans - B bond yields
Quantitative Easing refers to an unconventional 2. The yield on bonds is generally used as the risk-
monetary policy in which a central bank purchases free rate when calculating the cost of capital
government securities or other securities from the
market in order to lower interest rates and increase the A. 1 only
money supply. B. 2 only
C. Both 1 and 2
Quantitative easing increases the money supply by D. Neither 1 nor 2
flooding financial institutions with capital in an effort to
promote increased lending and liquidity. Quantitative Ans - C
easing helps the economy by reducing the long-term Factors affecting the yield - Monetary policy of the RBI
interest rates, thus making business and mortgage (interest Rates), fiscal position of the government and
borrowing cheaper thus giving a signal that the US Fed its borrowing programme, global markets, economy,
wants to support the economy. and inflation.
A fall in interest rates makes bond prices rise, and bond
Tapering is the gradual slowing of the pace of the USA yields fall. Rising interest rates cause bond prices to fall,
Fed Bank’s large scale asset purchases. Once the and bond yields to rise. So, a rise in bond yields means
tapering is complete then the Fed may go for reduction interest rates in the monetary system have fallen, and
in the size of the balance sheet. The aim is to slowly the returns for investors have declined.
remove the monetary stimulus.
The yield on bonds is normally used as the risk-free rate
Q2. Choose the correct statements. when calculating the cost of capital. It implies that when
bond yields go up, the cost of capital goes up. When
1. The share of non-banks in commercial credit bond yields go up, it is a signal that corporates will have
has more than doubled in FY 21 to pay a higher interest cost on debt. The risk of
2. Retail loans remained the largest recipient of bankruptcy and default also increases as debt servicing
credit extended by the NBFC sector costs go higher and this typically makes mid-cap and
highly leveraged companies vulnerable.
A. 1 only
B. 2 only Q4. Choose the INcorrect statements.
C. Both 1 and 2

Telegram – Indian_Economy_by_Aman_Soni Page 75


1. Under On-tap licensing, applications for bank finance was availed of but has diverted the
license will be accepted only during a fixed time funds for other purposes.
frame.  Default in repayment obligations by the unit to
2. RBI can fix the maximum age limit/ tenure for the lender and has siphoned off the funds so
MD & CEO even in Private sector banks. that the funds have not been utilized for the
specific purpose for which finance was availed
A. 1 only of, nor are the funds available with the unit in
B. 2 only the form of other assets.
C. Both 1 and 2  Default in repayment obligations by the unit to
D. Neither 1 nor 2 the lender and has also disposed off or removed
the movable fixed assets or immovable
Ans - A property given by it for the purpose of securing
On-tap licensing means that the window for getting a a term loan without the knowledge of the
bank license from RBI is open throughout the year. bank/lender.
Earlier, RBI used to invite applications for giving bank
licenses, and prospective players submitted their
applications within a fixed time-frame as prescribed by Q1. Which of the following sectors are included under
RBI. This arrangement was not open all the time. the Priority sector lending :
1. Agriculture
The Reserve Bank of India (RBI) fixed the tenure of MD, 2. MSME
CEO and whole-time director (WTD) in a private sector 3. Renewable Energy
bank at 15 years and prescribed the maximum age of 70 4. Export Credit
years for such functionaries. These directives form part
of the instructions issued by the RBI with regard to the A. 1, 2 & 3 only
chair and meetings of the board, composition of certain B. 1, 2 & 4 only
committees of the board, age, tenure and remuneration C. 2, 3 & 4 only
of directors, and appointment of the WTDs D. 1, 2, 3 & 4 only

Ans - D
Q5. In which of the following cases, the borrower is Priority sector lending refers to those sectors or areas of
considered as a willful defaulter : the economy which may not get timely and adequate
1. When borrower has the capacity to honor the credit. The RBI requires Indian banks to allocate certain
loan but has defaulted. portion of their overall lending for sectors mentioned
2. Borrower has NOT utilized the loan amount for under PSL. These areas of focus for PSL include
the specific purpose for which it was availed Agriculture, Micro, Small and Medium Enterprises
(MSME), Export Credit, Education, Housing, Social
A. 1 only Infrastructure, Renewable Energy
B. 2 only
C. Both 1 and 2 PSL guidelines are applicable to all domestic scheduled
D. Neither 1 nor 2 commercial banks (excluding Regional Rural Banks and
Small Finance Banks) and foreign banks with 20
Ans – C branches and above.

Cases of Wilful default : Q2. Choose the INcorrect statements.


 Default in repayment obligations by the unit to
the lender even when it has the capacity to 1. A very high gross NPA ratio means the bank‘s
honour the said obligations. asset quality is in very poor shape
 Default in repayment obligations by the unit to 2. Net NPA is that portion of bad loans which has
the lender and has not utilized the finance from not been provided for in the books.
the lender for the specific purposes for which

Telegram – Indian_Economy_by_Aman_Soni Page 76


2. A put option gives the holder the right to sell
A. 1 only the underlying asset at a specified price on or
B. 2 only before expiration
C. Both 1 and 2
D. Neither 1 nor 2 A. 1 only
B. 2 only
Ans – D C. Both 1 and 2
Gross non-performing assets (NPAs): They indicate how D. Neither 1 nor 2
much of a bank‘s loans are in danger of not being
repaid. If interest is not received for 3 months, a loan Ans - B
turns into NPA. A very high gross NPA ratio means the Call options are financial contracts that give the option
bank‘s asset quality is in very poor shape buyer the right, but not the obligation, to buy a stock,
bond, commodity or other asset or instrument at a
Net NPAs. The net NPA is that portion of bad loans specified price within a specific time period. The stock,
which has not been provided for in the books.Net NPA bond, or commodity is called the underlying asset. A call
is a better indicator of the health of the bank. buyer profits when the underlying asset increases in
price.
Q3. Choose the correct statements. A call is an option contract giving the owner the right,
but not the obligation, to buy a specified amount of an
1. Provision coverage ratio is a ratio of banks underlying security at a specified price within a
profits set aside as a provision against bad specified time.
loans.
2. A high PCR is a disadvantage for the bank in A call option may be contrasted with a put, which gives
meeting the losses arising out of future NPA’s the holder the right to sell the underlying asset at a
specified price on or before expiration.
A. 1 only
B. 2 only Q5. Choose the correct statements.
C. Both 1 and 2
D. Neither 1 nor 2 1. Under Peer-to-Peer Lending (P2P) generally an
online platform matches lenders with
Ans - A borrowers in order to provide unsecured loans
Provisioning coverage ratio. Banks usually set aside a 2. In India, currently the P2P lending business is
portion of their profits as a provision against bad loans. totally unregulated
A high PCR ratio (ideally above 70%) means most asset
quality issues have been taken care of and the bank is A. 1 only
not vulnerable. B. 2 only
C. Both 1 and 2
A Provisioning Coverage Ratio or PCR is the percentage D. Neither 1 nor 2
of funds that a bank sets aside for losses due to bad
debts. A high PCR can be beneficial to banks to buffer Ans - A
themselves against losses if the NPAs start increasing Peer-to-Peer Lending (P2P) lending is a form of
faster. crowdfunding; an online platform that matches lenders
with borrowers in order to provide unsecured loans.
Provision Coverage Ratio = ( Total provisions / Gross
NPAs ) * 100 The borrower can either be an individual or a business
requiring a loan. A fee is paid to the platform by both
Q4. Choose the correct statements. the lender and the borrower. P2P lending is not a new
feature. In 2017, the Reserve Bank of India had brought
1. Call option is an obligation on the buyer to buy this service under its regulatory purview. In P2P lending,
a stock users sitting on idle money provide loans to potential

Telegram – Indian_Economy_by_Aman_Soni Page 77


borrowers identified by the service provider. These
lenders then receive payments from the borrowers on a 1. IMPS is a real-time instant inter-bank funds
set basis — either one time, or in equated monthly transfer system managed by Banks Board
instalments. Some of the major companies operating in Bureau
this space include RupeeCircle, Finzy, IndiaMoneyMart, 2. National Payments Corporation of India (NPCI)
etc. is wholly owned by the RBI

A. 1 only
Q1. Choose the correct statements. B. 2 only
C. Both 1 and 2
1. The Banks Board Bureau was setup based on D. Neither 1 nor 2
recommendation of P.J. Nayak Committee
2. The BBB is a public authority as defined in the Ans - D
Right to Information Act, 2005 Immediate Mobile Payment Services(IMPS) is a real-
time instant inter-bank funds transfer system managed
A. 1 only by National payment corporation of India. IMPS is
B. 2 only available 24/7 throughout the year including bank
C. Both 1 and 2 holidays
D. Neither 1 nor 2
National Payments Corporation of India (NPCI), an
Ans - C umbrella organisation for operating retail payments and
Banks Board Bureau (BBB) is an autonomous body of settlement systems in India, is an initiative of Reserve
the Government of India tasked to improve the Bank of India (RBI) and Indian Banks‘ Association (IBA)
governance of Public Sector Banks, recommend under the provisions of the Payment and Settlement
selection of chiefs of government owned banks and Systems Act, 2007. It is a ―Not for Profit‖ Company
financial institutions and to help banks in developing under the provisions of Section 25 of Companies Act
strategies and capital raising plans. 1956 (now Section 8 of Companies Act 2013), with an
The Ministry of Finance takes the final decision on the intention to provide infrastructure to the entire Banking
appointments in consultation with the Prime Minister‘s system in India for physical as well as electronic
Office. It had replaced Appointments Board of payment and settlement systems.
Government.
Q3. Choose the correct statements.
In February 2016, the NDA government approved the
proposal for setting up BBB and it started functioning 1. A Neo bank is a kind of digital bank without any
from April 2016. The BBB works as step towards branches.
governance reforms in Public Sector Banks (PSBs) as 2. Neo banks have to acquire their own banking
recommended by P.J. Nayak Committee. The BBB was license from the RBI
the part of Indradhanush Plan of government, aimed at
revamping the Public Sector Banks.  Banks Board A. 1 only
Bureau comprises the Chairman, three ex-officio B. 2 only
membersi.e Secretary, Department of Public C. Both 1 and 2
Enterprises, Secretary of the Department of Financial D. Neither 1 nor 2
Services and Deputy Governor of the Reserve Bank of
India, and five expert members, two of which are from Ans - A
the private sector. The Banks Board Bureau is a public A neo bank is a kind of digital bank without any
authority as defined in the Right to Information Act, branches. Rather than being physically present at a
2005. specific location, neo banking is entirely online. It‘s a
wide umbrella of financial service providers who
beseech today‘s tech-savvy customers. Neo banks can
Q2. Choose the correct statements. be called fintech firms that provide digital and mobile-

Telegram – Indian_Economy_by_Aman_Soni Page 78


first financial solutions payments and money transfers, operate the account after the expiry of 10 years, even
money lending, and more. Neo banks don‘t have a bank after such amount has been transferred to DEAF.
license of their own but count on bank partners to
provide bank licensed services.
Q5. Choose the correct statements.
And, since there isn‘t a physical location and that
they‘re completely online, the customer fees are 1. Oil bonds are issued by government to oil
slashed by a significant amount. Because Neo banks are marketing companies (OMCs) to defer the
customercentric, they provide personalized services to payment of money related to regulating
their customers that are fired up via technology. Data- domestic fuel prices
driven decisions drive the decision-making process of a 2. These bonds qualify as statutory liquidity ratio
neo bank. Since their platforms are also very (SLR) securities
modernized, it becomes easier for them to collect and
analyze data and understand how their customers A. 1 only
behave in the neo banking ecosystem. Based on these B. 2 only
observations, they create cohorts of customers based C. Both 1 and 2
on their actions rather than merely sticking to one or D. Neither 1 nor 2
two data points.
Ans - A
Q4. Choose the INcorrect statements. Oil bonds are issued by the government to compensate
oil marketing companies (OMCs), fertilizer companies
1. Public sector banks have a higher percentage of and the Food Corporation of India (FCI) for losses borne
unclaimed deposits than private banks by them in the process of regulating prices in the
2. Unclaimed deposits cannot be claimed by the domestic market. It was introduced in 2005 to defer the
legal heirs after a period of 5 years payment of money to the oil marketing companies.
They are akin to government securities. These usually
A. 1 only have a long maturity period extending over 15-20 years.
B. 2 only Interest payments will be due at fixed intervals during
C. Both 1 and 2 the tenure of the bond.
D. Neither 1 nor 2
These debts are not accounted in the fiscal deficit
Ans – B number of the issuing year. Unlike cash subsidies, there
is no direct cash flow. Moreover, oil bonds do not
Unclaimed deposits are defined as those deposits which qualify as statutory liquidity ratio (SLR) securities,
are lying in accounts not operated for a period of 10 or making them less liquid when compared to other
more years. Public sector banks have a higher government securities. Oil bonds can be traded for
percentage of unclaimed deposits. Section 26 of the liquid cash by sale in the secondary market to insurance
Banking Regulation Act, 1949 requires banks to submit companies, banks, and other financial institutions.
to RBI information about these accounts within 30 days
after each calendar year ends.
Q1. Choose the correct statements.
Unclaimed deposits can be claimed by their legal 1. In the public interest, RBI may direct for special
owners after satisfying certain conditions prescribed by audit of the banking company‘s accounts
RBI. As per Section 26A of the amended Banking 2. RBI may prohibit banking companies from
Regulation Act, 1949, money lying in dormant bank entering into any particular transaction or class
accounts is transferred to the Depositor Education and of transactions.
Awareness Fund (DEAF) within a period of three months
from the expiry of the above said 10 years. The A. 1 only
depositor is, however, entitled to claim from the bank B. 2 only
her/his deposit or any other unclaimed amount or C. Both 1 and 2

Telegram – Indian_Economy_by_Aman_Soni Page 79


D. Neither 1 nor 2 for the trader, algo-trading renders markets more liquid
and trading more systematic by ruling out the impact of
Ans - C human emotions on trading activities.
Under Banking Regulation Act the RBI enjoys the
following powers: Algorithmic trading was introduced and allowed in India
 Section 30 – Power to order Special audit: In the in 2008 by the Securities and Exchange Board of India
public interest or in the interest of the banking (Sebi). Initially, it started with Direct Market Access
company or its depositors, the RBI may at any (DMA) and was restricted to institutional investors only,
time by order direct that a special audit of the but due to the cost advantage and better execution, the
banking company‘s accounts. trading community welcomed it with open arms.
 Section 35 – Inspection of Banking Companies:
Reserve Bank on its own or being directed so to
do by the Central Government, inspect any Q3. Choose the correct statements.
banking company and its books and accounts 1. Front-running is a form of market manipulation
and supply to the banking company a copy of its and insider trading
report on such inspection. 2. It involves purchasing a stock based on advance
 Section 35A – Power of the Reserve Bank to give non-public information regarding an expected
directions: In the public interest or in the large transaction that will affect the price of the
interest of Banking policy RBI has powers to share.
issue, modify or cancel as it deems fit, and the
banking companies or the banking company, A. 1 only
are bound to comply with such directions. B. 2 only
 Section 36 – Further powers and functions of C. Both 1 and 2
Reserve Bank: RBI may caution or prohibit D. Neither 1 nor 2
banking companies or any banking company in
particular against entering into any particular Ans - C
transaction or class of transactions. Front-running, which is illegal in India, involves
purchasing a stock based on advance non-public
Q2. Choose the correct statements. information regarding an expected large transaction
that will affect the price of the share. When mutual
1. Algorithmic trading uses a computer program funds make a big order, some fund managers buy the
that follows a defined set of instructions to same shares in their personal accounts before executing
place a trade. the MFs’ order. When MFs purchase in huge quantities,
2. SEBI has banned Algorithmic trading on the the price of the share is expected to go up.
stock exchanges in India
Sebi has categorised front running as a form of market
A. 1 only manipulation and insider trading because a person who
B. 2 only commits a front running activity expects security’s price
C. Both 1 and 2 movements based on the non-public information. Sebi
D. Neither 1 nor 2 has investigated and penalised several fund houses and
fund managers in the past for front-running.
Ans - A
Algorithmic trading (also called automated trading,
black-box trading, or algotrading) uses a computer Q4. Choose the correct statements.
program that follows a defined set of instructions (an
algorithm) to place a trade. The trade, in theory, can 1. Dovish Monetary Policy Stance favors high
generate profits at a speed and frequency that is interest rates.
impossible for a human trader. The defined sets of 2. The major aim of Accommodative Monetary
instructions are based on timing, price, quantity, or any Policy Stance is to increase spending
mathematical model. Apart from profit opportunities
Telegram – Indian_Economy_by_Aman_Soni Page 80
A. 1 only Ans - A
B. 2 only Retail Direct scheme is a one-stop solution to facilitate
C. Both 1 and 2 investment in Government Securities by Individual
D. Neither 1 nor 2 Investors. Under this scheme Individual Retail investors
can open Gilt Securities Account – “Retail Direct Gilt
Ans - B (RDG)” Account with the RBI. Non-Resident retail
Dovish Monetary Policy Stance investors eligible to invest in Government Securities
This monetary policy stance involves low interest rates. under Foreign Exchange Management Act, 1999 are
Low-Interest Rates would entice consumers to take eligible under the scheme.
credit (loans) from Banks and other sources. As the
demand increases (due to increase in money supply The investor can place non competitive bids in Primary
owing to the low rate), the prices of Goods & Services issuance of all Central Government securities (including
would rise/increase. And we all know that general rise Treasury Bills and Sovereign Gold bonds) as well as
in prices of Goods & Services is called Inflation. Inflation securities issued by various State Governments.
will cause to balance Economic Growth. (Inflation is not
always harmful to the Economy, it is needed to trigger Under this scheme, the individual can also access
economic growth). Economists believe that lower Secondary market through “NDS OM” - RBI’s trading
interest rates will lead to a hike in employment and an system. The investor will automatically receive any
increase in Economic Growth. This stance might also interest paid/maturity proceeds into his linked bank
lead to a possible weakening of the country‘s currency. account on due dates.

Accommodative Monetary Policy Stance Q1. Choose the correct statements.


This happens when the economic growth is slowing
down. The major aim is to increase spending. 1. Minimum Wages Act fixes a single uniform
Accommodative monetary policy is implemented to minimum wage rate across the country
allow the money supply to rise in line with national 2. The Code on Wages, 2020 mentions the
income and the demand for money. This is also known concept of a floor wage
as ―easy monetary policy‖. When the economy slows
down, the central bank (RBI) can implement an A. 1 only
Accommodative Monetary Policy to stimulate the B. 2 only
economy. It does this by running a succession of C. Both 1 and 2
decreases in the Interest rates, making the cost of D. Neither 1 nor 2
borrowing cheaper. Accommodative money policy is
triggered to encourage more spending from consumers Ans – B
and businesses by making money less expensive to
borrow through the lowering of short-term interest Minimum Wages Act in 1948
rates.
India introduced the Minimum Wages Act in 1948,
Q3. Choose the correct statements. giving both the Central government and State
government jurisdiction in fixing wages. The act is
1. Through the Retail Direct scheme, Individual legally non-binding, but statutory. Payment of wages
Investors can subscribe to govt securities below the minimum wage rate amounts to forced
2. Currently, the access is limited to the Securities labour. Wage boards are set up to review the industry’s
in Primary market only capacity to pay and fix minimum wages such that they
at least cover a family of four requirements of calories,
A. 1 only shelter, clothing, education, medical assistance, and
B. 2 only entertainment.
C. Both 1 and 2 Under the law, wage rates in scheduled employment
D. Neither 1 nor 2 differ across states, sectors, skills, regions and
occupations owing to differences in costs of living,

Telegram – Indian_Economy_by_Aman_Soni Page 81


regional industries’ capacity to pay, consumption 1. Foreign Currency Assets are the largest
patterns, etc. Hence, there is no single uniform component of the forex reserve.
minimum wage rate across the country and the 2. Foreign exchange reserves include bonds,
structure has become overly complex. treasury bills and other government securities.

Fixation of Floor Wage: A. 1 only


B. 2 only
The Code on Wages, 2020 mentions the concept of a C. Both 1 and 2
floor wage, which empowers the central government to D. Neither 1 nor 2
fix floor wages taking into account the minimum living
standards of workers. The floor wage is a baseline wage Ans – D
below which minimum wages cannot be fixed by state
governments. The Wage Code permits the fixation of Foreign exchange reserves are assets held on reserve by
different floor level wages for different geographical a central bank in foreign currencies, which can include
areas. Also, the wage rules do not outline the exact bonds, treasury bills and other government securities. It
criteria and methods for fixation of floor wage rates by needs to be noted that most foreign exchange reserves
the Central Government. are held in US dollars.

Q2. Choose the correct statements. India’s Forex Reserve include:

1. As per a World Bank Group report annual Foreign Currency Assets


remittances transferred to India are highest in  Gold reserves
the world  Special Drawing Rights
2. Remittances in India have been substantially  Reserve position with the International
higher than even Foreign Direct Investment Monetary Fund (IMF).
(FDI)  Foreign Currency Assets

A. 1 only Foreign Currency Assets are assets that are valued


B. 2 only based on a currency other than the country’s own
C. Both 1 and 2 currency. Foreign Currency Assets are the largest
D. Neither 1 nor 2 component of the forex reserve. It is expressed in dollar
terms.The Foreign Currency Assets include the effect of
Ans – C appreciation or depreciation of non-US units like the
euro, pound and yen held in the foreign exchange
As per a World Bank Group report (2021),annual reserves.
remittances transferred to India are estimated to be
$87 billion- highest in the world, followed by China ($53
billion), Mexico ($53 billion), the Philippines ($36 billion) Q4. The “Inter Creditor Agreement” often seen in news
and Egypt ($33 billion). is related to :
India’s Remittances contribute 3% in GDP and is lower
than that of countries such as Nepal (24.8%), Pakistan A. Reforms in the WTO
(12.6%), Sri Lanka (8.3%) and Bangladesh (6.5%), B. Agreement between debtors and creditors
Remittances in India have been substantially higher C. Agreement among banks to resolve bad loans
than even Foreign Direct Investment (FDI) and the flow D. India France defense cooperation Pact
of remittances is much less fluctuating than that of FDI.
Ans – C

Q3. Choose the INcorrect statements. Inter Creditor Agreement is an agreement among banks
that have dues from a borrower in stress. The pact
mandates the lead bank to formulate a resolution plan
Telegram – Indian_Economy_by_Aman_Soni Page 82
that will be executed in a time bound manner. It was themselves every five years. Registered NGOs can
framed under the aegis of Indian banks association, receive foreign contributions for five purposes — social,
following the recommendation of Sunil Mehta educational, religious, economic and cultural.
Committee on stressed assert resolution. Around 22
public sector banks (including India Post Payments
Bank), 19 private lenders and 32 foreign banks signed Q1. Choose the correct statements.
the inter-creditor agreement (ICA) to fast track the
resolution of stressed assets. 1. Sovereign Gold Bonds are issued by RBI on
behalf of the govt
Applicability - The ICA is applicable to all corporate 2. Redemption of bonds will be based on the
borrowers who have availed loans for an amount of 50 mutually predetermined fixed prices
crore or more under consortium lending / multiple
banking arrangement. The agreement is part of the A. 1 only
proposed Project Sashakt. Sashakt plan is approved by B. 2 only
the government to address the problem of resolving C. Both 1 and 2
bad loans. The objective is to use this ICA for faster D. Neither 1 nor 2
facilitation of resolution of stressed assets.
Ans - A
Sovereign Gold Bond Scheme - It is to be issued by
Q5. Choose the correct statements. Reserve Bank of India on behalf of the Government of
India. The Bonds will be denominated in multiples of
1. Registration under the Foreign Contribution gram(s) of gold with a basic unit of 1 gram. The tenor of
(Regulation) Act is done by the Home Ministry the Bond will be for a period of 8 years with exit option
2. NGOs working in the field of religious activities after 5th year to be exercised on the next interest
cannot receive funding under the Act payment dates. The Gold Bonds will be issued as
Government of India Stock under GS Act, 2006. The
A. 1 only investors will be compensated at a fixed rate of 2.50
B. 2 only percent per annum payable semi-annually on the
C. Both 1 and 2 nominal value.
D. Neither 1 nor 2
The investors gain from appreciation in gold prices as
Ans – A redemption of bonds will be based on the then
prevailing prices. If gold prices treble after eight years,
Foreign Contribution (Regulation) Act (FCRA) the investor will get the higher prices plus the 2.5%
registration is granted to associations for a period of interest. The investor does not lose in terms of the units
five years. Before expiry of registration, NGOs are of gold which he has paid for if gold prices fall. Although
supposed to apply for renewal of registration if they the tenure of the bond is 8 years, early
wish to continue receiving foreign funding. Even after encashment/redemption of the bond is allowed after
an application is made, the Home Ministry reserves the the fifth year, on coupon payment dates.
right to reject the application in case it finds the NGO
concerned violating FCRA. The registration is mandatory Q2. Choose the correct statements.
for associations and NGOs to receive foreign funds.
1. Real Estate Investment Trusts pool investor
money like mutual funds and use it to buy a
Foreign funding of persons in India is regulated under portfolio of real estate assets
FCRA Act and is implemented by the Ministry of Home 2. REITs can invest in all kinds of income-
Affairs. The Act ensures that the recipients of foreign generating properties — residences, offices,
contributions adhere to the stated purpose for which hotels, malls & warehouses
such contribution has been obtained. Under the Act,
organisations are required to register/renew A. 1 only

Telegram – Indian_Economy_by_Aman_Soni Page 83


B. 2 only operating on positive gross margins in the host country
C. Both 1 and 2 or any other country abroad.
D. Neither 1 nor 2
Q4. Choose the INcorrect statements.
Ans - C
REIT’s (Real Estate Investment Trust) are investment 1. IGST is charged when movement of goods and
vehicles that pool investor money like mutual funds and services take place from one state to another.
use it to buy a portfolio of real estate assets. They 2. IGST on interstate supplies are collected by the
manage these assets to generate a regular income and respective state govts
capital appreciation. In order to ensure that the REIT is
able to generate income, the portfolio of a REIT should A. 1 only
be invested in completed and rent-generating B. 2 only
properties. C. Both 1 and 2
D. Neither 1 nor 2
REITs can invest in all kinds of income-generating
properties — residences, offices, hotels, malls, Ans – B
warehouses etc, in India the listed REITs are focused
mainly on office space. The first listing of REIT in the IGST meaning Integrated Goods and Service Tax, is one
country (Embassy REIT), the listing and trading of REITs of the three categories under Goods and Service Tax
on the exchanges allow investors to buy or sell them at (CGST, IGST and SGST) with a concept of one tax one
any time. The structure of a REIT is similar to a mutual nation. IGST falls under Integrated Goods and Service
fund. REITs focus on paying out regular income from Tax Act 2016. IGST is charged when movement of goods
rent earned on properties. and services from one state to another. For example, if
goods are moved from Tamil Nadu to Kerala, IGST is
Q3. A company operates on negative gross margin sales levied on such goods. The revenue out of IGST is shared
(losses) in the host country, is funded by positive gross by state government and central government as per the
margins (profits) in their home country. The above rates fixed by the authorities.
explanation is related to :
The GST on supplies in the course of interState trade
A. Qualified Institutional Investors shall be levied and collected by the Government of India
B. Capital Dumping and such tax shall be apportioned between the Union
C. Foreign Direct Investment and the States according to the provisions of law on the
D. Currency Convertibility recommendations of the Goods and Services Tax
Council. Under the IGST scheme, 50% of the collections
Ans – B will go to the Centre (as the Central Goods and Services
Tax component) and the remaining 50% will be
Dumping refers to exporting a good at a lower price allocated to the States and Union Territories (as the
than the price charged for the good at home. Capital State Goods and Services Tax component). And, 42% of
Dumping is on similar lines. Capital dumping may the CGST will be devolved to the States and Union
include the following: Territories.
i.When a company operates on negative gross margin
sales (losses) in the host country, funded by positive Q5. The term “pre-packs” often seen in news is related
gross margins (profits)in their home country or other to :
abroad countries.
ii. Also, Capital dumping in the context of the Indian A. Agreement for debt resolution
startup ecosystem would include companies operating B. Social security scheme for small traders
on negative gross margin sales in the host country, C. Venture capital for new startups
funded by Venture Capital money. Companies falling in D. Production linked Incentive Scheme
the second category may not be necessarily be
Ans – A

Telegram – Indian_Economy_by_Aman_Soni Page 84


are priced at ―reasonable levels (based on Nutrient
A pre-pack is an agreement for the resolution of the based Subsidy scheme).
debt of a distressed company through an agreement
between secured creditors and investors instead of a
public bidding process. A pre-pack envisages the Q2. Choose the correct statements.
resolution of the debt of a distressed company through
a direct agreement between secured creditors and the 1. Diammonium phosphate (DAP) fertilizer has
existing owners or outside investors, instead of a public potassium whereas the NPK fertilizer contains
bidding process. no potassium
2. The major share of DAP demand in India is met
The pre-pack mechanism is effective in arriving at a through imports
quick resolution for distressed companies, and that the
regime should be rolled out to all corporations over A. 1 only
time as legal issues are settled through case law. This B. 2 only
system is a mechanism for insolvency resolution in the C. Both 1 and 2
United Kingdom and Europe over the past decade. D. Neither 1 nor 2
Under the pre-pack system, financial creditors will agree
to terms with the promoters or a potential investor, and Ans - B
seek approval of the resolution plan from the National The key difference between DAP and NPK fertilizer is
Company Law Tribunal (NCLT). that the DAP fertilizer has no potassium whereas the
NPK fertilizer contains potassium as well. The term DAP
Q1. Choose the correct statements. refers to diammonium phosphate, and it is a phosphate
1. The MRP of Urea is fixed by the Central Govt fertilizer; the world's most common phosphorus
2. The govt currently does not provide any subsidy fertilizer.
for non-urea fertilizers
Rock phosphates or phosphorites are sedimentary
A. 1 only phosphatic deposits comprising fine-grained mixture of
B. 2 only various calcium phosphates, most important being
C. Both 1 and 2 hydroxylapatite, carbonateapatite, fluorapatite and
D. Neither 1 nor 2 their solid solutions.
About 80% phosphate production in the world is
Ans - A derived from phosphate rocks (phosphorite). Rock
The subsidy goes to fertiliser companies, although its Phosphate is the key raw material for DAP and NPK
ultimate beneficiary is the farmer who pays MRPs less fertilisers and India is 90% dependent on imports.
than the market-determined rates. Manufacturers of Players in the production of indigenous DAP notably
fertilizers (urea) receive 100% of subsidy after fertiliser include IFFCO, Gujarat State Fertilizer Corporation, Zuari
is delivered to the farmer, and the latter‘s identity viz. Industries, Paradeep Phosphates, etc. However,
Aadhaar is captured on the point of sale (PoS) machine indigenous production contributes to roughly 40% of
at the dealer‘s shop. Therefore, the subsidy continues the total annual demand for the product. For the rest,
to be routed through manufacturers even though the Indian consumers rely on imports.
sale of fertilizer is being verified using Aadhar
ecosystem Q3. Choose the correct statements.

The manufacturers sell urea at the maximum retail price 1. India is the largest fish producing country in the
(MRP) controlled by the Centre, which is kept at a low world.
level. They also get subsidy reimbursement on 2. The marine sector fish production is more than
unitspecific basis under the new pricing scheme (NPS). the country's fish production in the inland
The MRPs of non-urea fertilisers are decontrolled or sector
fixed by the companies. The Centre, however, pays a
flat per-tonne subsidy on these nutrients to ensure they A. 1 only

Telegram – Indian_Economy_by_Aman_Soni Page 85


B. 2 only cess and surcharge are not part of the divisible pool,
C. Both 1 and 2 from which devolution of Central taxes takes place to
D. Neither 1 nor 2 the States.

Ans - D Q5. The species “HeveaBrasiliensis” recently seen in


India is the 3rd largest fish producing and 2nd largest news is related to :
aquaculture nation in the world after China. The Blue
Revolution in India demonstrated importance of A. A type of Rubber Tree
Fisheries and Aquaculture sector. B. New Species of Spider discovered in the
western Ghats
In the recent past, Indian fisheries has witnessed a C. Frog discovered in the Andaman & Nicobar
paradigm shift from marine dominated fisheries to D. Snake Discovered in the Amazon Rain Forest
inland fisheries, with the latter emerging as a major Ans - A
contributor of fish production from 36% in the mid- Intercropping of medicinal plants in rubber plantations
1980 to 70% in the recent past. Within inland fisheries, has gained momentum with leading ayurvedic medicine
a shift from capture to culture-based fisheries has manufacturing companies coming forward to take up
paved the way for sustained blue economy. the project by supplying necessary planting materials.
Rubber is made from the latex of a tree called
Q4. Choose the correct statements. HeveaBrasiliensis.

1. Agriculture Infrastructure and Development The Rubber Board is a statutory body constituted by the
Cess (AIDC) is currently levied on imported urea, Government of India, under the Rubber Act 1947, for
, imported palm oil etc the overall development of the rubber industry in the
2. The money collected through cess and country. The Ministry of Commerce & Industry,
surcharge are not part of the divisible pool for Government of India, established the Rubber Board in
devolution of funds 1947 to strengthen the development of the rubber
industry by offering financial assistance, consolatory
A. 1 only and regulatory services. Head Office is in Kottayam,
B. 2 only Kerala. Foreign Direct Investment (FDI): 100% FDI in
C. Both 1 and 2 plantations of rubber, coffee, tea, cardamom, palm oil
D. Neither 1 nor 2 tree and olive oil tree.

Ans - C
Agriculture Infrastructure and Development Cess (AIDC)
was introduced in the Budget 2021.The purpose of the Q1. Choose the correct statements.
new AIDC is to raise funds to finance spending on
developing agriculture infrastructure. Due to low 1. Food Corporation of India (FCI) sells food grains
private investment in agriculture, the Centre now seeks to states at the Central Issue Price
to raise a dedicated fund to meet these expenses. The 2. Under the Decentralized Procurement Scheme
new cess will be levied on 29 products, prominent (DCP), The Central govt directly procures food
among which are gold, silver, imported apple, imported grains from the farmers
alcohol (excluding beer), imported pulses, imported
palm oil, imported urea, and petrol/diesel including A. 1 only
branded ones B. 2 only
C. Both 1 and 2
Articles 270 and 271 of the Constitution, gives power to D. Neither 1 nor 2
the Centre to collect cess and deposits it in the
Consolidated Fund of India. However, the money is then Ans - A
supposed to be transferred to a segregated fund to be Functions of FCI
used for specific purpose. The money collected through  To procure foodgrains

Telegram – Indian_Economy_by_Aman_Soni Page 86


 To Maintain operational stock and buffer stock The index has become a critical and timely monthly
for food security indicator of the state of international food markets,
 Allocation of grains to state gauging the change in food commodity prices over time
 Selling grains to state at Central Issue Price‘ in nominal and real terms.
 Distributing and transporting grains to state
Q4. Which of the following have been INcorrectly
Under the Decentralized Procurement Scheme (DCP), matched :
introduced in 1997-98, food grains are procured and
distributed by the State Governments themselves. The 1. K shaped Recovery - Different parts of the
designated States procure, store and issue food grains economy recover at different rates
under Targeted Public Distribution System (TPDS) and 2. V shaped - Economy after falling, struggles
other welfare schemes of the Government. around a low growth rate for some time, before
rising gradually.
Q2. Which of the following GI Tags and respective states
have been correctly matched : A. 1 only
B. 2 only
1. Suvarnarekha mangoes – Odisha C. Both 1 and 2
2. Jalgaon‘s Banana – Maharashtra D. Neither 1 nor 2
3. Nendran Banana – Tamil Nadu
4. Zardalu mango – Bihar Ans – B

A. 1 & 3 only K-Shaped Recovery: A K-shaped recovery occurs when,


B. 1 & 4 only following a recession, different parts of the economy
C. 2 & 3 only recover at different rates, times, or magnitudes. This is
D. 2 & 4 only in contrast to an even, uniform recovery across sectors,
Ans – D industries, or groups of people.

Suvarnarekha mangoes – Andra Pradesh V-shaped recovery - It is the next-best scenario after Z-
Dahanu Gholvad sapota & Marathwada Kesar mango - shaped recovery in which the economy quickly recoups
Maharashtra lost ground and gets back to the normal growth trend-
Jalgaon‘s Banana – Maharashtra line. In this, incomes and jobs are not permanently lost,
Changalikodan Nendran Banana - Kerala and the economic growth recovers sharply and returns
Shahi Litchi & Bhagalpuri Zardalu mango – Bihar to the path it was following before the disruption.

Q3. The World Food Price Index is released by : U-shaped recovery: It is a scenario in which the
economy, after falling, struggles around a low growth
A. World Bank rate for some time, before rising gradually to usual
B. WTO levels. In this case several jobs are lost and people fall
C. FAO upon their savings. If this process is more-long drawn
D. UNEA than it throws up the elongated U shape

Ans – C Q5. Choose the correct statements.

The FAO’s World Food Price Index is a measure of the 1. Human Capital is the productive capacity
monthly change in international prices of a basket of generated by an individual due to better
food commodities. It consists of the average of five education and health
commodity group price indices (cereal, vegetable, dairy, 2. ‘Human Capital Index’ is released by the World
meat and sugar), weighted with the average export Bank
shares.
A. 1 only

Telegram – Indian_Economy_by_Aman_Soni Page 87


B. 2 only directly or through other related enterprises) by a
C. Both 1 and 2 foreign direct investor to an enterprise. FDI has three
D. Neither 1 nor 2 components, viz., equity capital, reinvested earnings
and intra-company loans.
Ans - C
‘Human Capital Index’ is released by the World Bank. Q2. Choose the correct statements.
The HCI is an attempt to measure investments and
improvements required in health, schooling, and 1. Anti-dumping duty creates a level-playing field
nutrition around the world. Human Capital is the for domestic producers vis-a-vis foreign
productive capacity generated by an individual due to producers and exporters
better education and health. Better productive capacity 2. The imposition of anti-dumping duty is NOT
results in higher earning potential for individuals and permissible under the WTO regime
also higher earnings for the nation. The HCI therefore
measures the average shortfall in human capital for an A. 1 only
individual worker in a country if they were exposed to B. 2 only
full (i.e., optimal) education and health. C. Both 1 and 2
D. Neither 1 nor 2
The HCI has three components:
 Survival (a probability estimate that a child will Ans - A
survive up to age 5); An anti-dumping duty is a protectionist tariff that a
 Schooling (derived from the expected years of domestic government imposes on foreign imports that
schooling and a harmonised test score from it believes are priced below fair market value. Dumping
India‘s National Achievement Survey, or NAS by is a process where a company exports a product at a
the NCERT); price lower than the price it normally charges in its own
 Health (a composite of the fraction of children home market. The duty is aimed at ensuring fair trading
under five that are not stunted and the fraction practices and creating a level-playing field for domestic
of 15-year olds expected to survive till age 60). producers vis-a-vis foreign producers and exporters.

The duty is imposed only after a thorough investigation


Q1. Which of the following is considered as FDI : by a quasi-judicial body, such as Directorate General of
1. Equity Capital Trade Remedies, in India. The imposition of anti-
2. Reinvested Earnings dumping duty is permissible under the World Trade
Organization (WTO) regime.
A. 1 only
B. 2 only Q3. Which of the following have been correctly
C. Both 1 and 2 matched :
D. Neither 1 nor 2
1. NK Singh committee – Review of FRBM act
Ans - C 2. Dave committee – Capital account convertibility
FDI is the process whereby residents of one country (the 3. S Tendulkar committee – Redefining poverty
home country) acquire ownership of assets for the line
purpose of controlling the production, distribution and 4. Tarapur committee – Pension scheme for
other activities of a firm in another country (the host unorganised sector
country).
It is different from Foreign Portfolio Investment where A. 1 & 2 only
the foreign entity merely buys stocks and bonds of a B. 1 & 3 only
company. C. 2 & 3 only
D. 1, 3 & 4 only
FPI does not provide the investor with control over the
business. Flows of FDI comprise capital provided (either Ans – B

Telegram – Indian_Economy_by_Aman_Soni Page 88


1. It is an attached office of the Ministry of
Tarapur committee – Capital account convertibility External Affairs
Dave committee – Pension scheme for unorganised 2. It has the powers to regulate, restrict or
sector prohibit exports and imports
S Tendulkar committee – Redefining poverty line and its
calculation formula
Shah committee – Reforms relating to NBFC
Kumaramangalam Birla committee – Corporate A. 1 only
governance B. 2 only
NK Singh committee – Review of FRBM act C. Both 1 and 2
D. Neither 1 nor 2
Q4. Choose the INcorrect statements.
Ans - B
1. A Duty Credit Scrip is like a credit certificate Directorate General of Foreign Trade (DGFT)
issued by the Director General of Foreign Trade Directorate General of Foreign Trade (DGFT)
(DGFT) organisation is an attached office of the Ministry of
2. Duty Credit Scrips can be used by an exporter to Commerce and Industry and is headed by Director
pay their tax liabilities like basic customs duty General of Foreign Trade. It is responsible for
etc formulating and implementing the Foreign Trade Policy
with the main objective of promoting India‘s exports.
Headquarters: New Delhi
A. 1 only
B. 2 only Functions:
C. Both 1 and 2  Licensing of imports and exports.
D. Neither 1 nor 2  Regulate, restrict or prohibit exports and
imports.
Ans - D  It plays an advisory role to the Government on
Duty Credit Scrips (DCS)is an export promotion benefit Policy measures pertaining to national and
offered by the Government of India under the Foreign international economic scenarios.
Trade Policy (FTP) 2015-20. As with other export
benefits, the aim is to incentivize exporters so that they
boost the inflow of foreign exchange to India. Q1. Which of the following are the possible benefits of
Rupee – Rouble Trade :
A DCS provides tax incentives on exports, which can be
used by exporters to set off their import duties. It is 1. Eliminate supply chain disruptions
issued under the Merchandise Exports from India 2. Eliminates dependence on the US$
Scheme (MEIS), Service Exports from India Scheme 3. Fluctuation of value of Rouble doesn’t affect
(SEIS), and the Export Capital Goods Scheme. Rupee
4. Bypass economic sanctions
DCS can be used by an exporter to pay their tax
liabilities. It can be used against tax liabilities arising out
of basic customs duty, additional customs duty, A. 1, 2 & 3 only
safeguard duty, transitional specific safeguard duty, and B. 1, 2 & 4 only
anti-dumping duty. DCS can be transferred to others, C. 2, 3 & 4 only
but it cannot be used to set off GST, compensation cess, D. 1, 2, 3 & 4 only
and education cess.
Ans – B
Q5. Choose the correct statements wrt Directorate
General of Foreign Trade (DGFT) Benefits -
Eliminates dependence on the US$ for bilateral trade.
Telegram – Indian_Economy_by_Aman_Soni Page 89
Bypass economic sanctions like the blocking the Russian
banks from the SWIFT payments network. 1. The GAAR provisions come under the Income
A battered Rouble in the wake of its war with Ukraine Tax Act
works in India‘s favor with imports costing less. 2. Under GAAR aggressive tax planning is allowed
Avoid any delay or default in payments. but tax evasion is not allowed
Eliminate supply chain disruptions

Issue A. 1 only
The fluctuation in the value of rouble could make it B. 2 only
difficult to implement C. Both 1 and 2
the rupee-rouble payment mechanism. D. Neither 1 nor 2
The valuation is actually a key issue for pegging the
rouble-rupee exchange rate. Ans - A
General Anti-Avoidance Rule (GAAR):
Q2. Choose the correct statements. It is an anti-tax avoidance law in India to curb tax
evasion and avoid tax leaks. It came into effect on 1st
1. Import intensity can be defined as the degree of April 2017. The GAAR provisions come under the
value addition of an imported item that Income Tax Act, 1961. GAAR is a tool for checking
subsequently gets exported. aggressive tax planning especially that transaction or
2. Gems and jewellery products have import business arrangement which is/are entered into with
intensity in the Indian context the objective of avoiding tax.

A. 1 only It is specifically aimed at cutting revenue losses that


B. 2 only happen to the government due to aggressive tax
C. Both 1 and 2 avoidance measures practiced by companies. With
D. Neither 1 nor 2 GAAR there is no difference between tax avoidance and
tax evasion. All transactions which have the implication
Ans - C of avoiding tax can come under the scanner of GAAR.
The depreciation in the value of the rupee may be good
for the country‘s export sectors, but there‘s a Q4. Choose the correct statements.
vulnerable sub-category that‘s adversely impacted:
labour-intensive export sectors such as gems and 1. Baltic Index is related to the cost of Crude oil
jewellery, pharmaceuticals and electronics that are barrel at the international markets
highly dependent on imports of inputs 2. It is created by the Baltic Exchange based in
London
Sectors such as gems & jewellery and electronics that
have a high import intensity — the value addition of A. 1 only
imported items as a proportion of the value of items B. 2 only
that are subsequently exported — are faced with higher C. Both 1 and 2
input costs and lower demand as they are forced to D. Neither 1 nor 2
pass on some of the increase in cost of imports.
Ans – B
Import intensity can be defined as the degree of value
addition of an imported item that subsequently gets Baltic Freight Index stands for shipping and trade index
exported. In the Indian context, gems and jewellery is a which has been created by the Baltic Exchange based in
typical example of such export product having high London. It is a measure of the cost of transporting
import intensity. various raw materials. The Baltic Exchange contacts the
shipping brokers directly for assessing price levels for a
Q3. Choose the correct statements wrt General Anti- specific route, product, time and speed.
Avoidance Rules (GAAR):

Telegram – Indian_Economy_by_Aman_Soni Page 90


Q5. The IMF’s Quota is decided based on a weighted National calamity
average of which of the following factors : Collapse of agriculture
Structural reforms
1. GDP Decline in real output growth of a quarter by at least
2. Openness three percentage points below the average of the
3. Economic variability previous four quarters.
4. International reserves
Q2. Choose the correct statements.
A. 1, 2 & 3 only
B. 1, 2 & 4 only 1. The Phillips Curve portrays macroeconomic
C. 2, 3 & 4 only policy as a trade-off between unemployment
D. 1, 2, 3 & 4 only and inflation.
2. Stagflation occurs when the money supply is
Ans – D contracting while supply of goods and inputs is
being constrained
The IMF uses a quota formula to help assess a
member's relative position. The current quota formula
is a weighted average of GDP (weight of 50 percent), A. 1 only
openness (30 percent), economic variability (15 B. 2 only
percent), and international reserves (5 percent). C. Both 1 and 2
D. Neither 1 nor 2

Ans - A
Q1. Choose the correct statements. A recession is a significant decline in economic activity
spread across the economy, lasting more than a few
1. NK Singh review committee advocated for a months, normally visible in real GDP, real income,
Debt to GDP ratio of 80% to be targeted with a employment, industrial production, and wholesale-
40% limit for the centre and 40% limit for the retail sales.
states. While there is also no standard definition for
2. The Centre can exceed the annual fiscal deficit depression, it is commonly defined as a more severe
target citing National security, war etc version of a recession.

A. 1 only In particular, the economic theory of the Phillips Curve,


B. 2 only which developed in the context of Keynesian
C. Both 1 and 2 economics, portrayed macroeconomic policy as a trade-
D. Neither 1 nor 2 off between unemployment and inflation.

Ans - B Stagflation was first recognized during the 1970s when


In May 2016, the government set up a committee under many developed economies experienced rapid inflation
NK Singh to review the FRBM Act. The Committee and high unemployment as a result of an oil shock. The
suggested using debt as the primary target for fiscal prevailing economic theory at the time could not easily
policy. explain how stagflation could occur. Generally,
The targets set by NK Singh: stagflation occurs when the money supply is expanding
Debt to GDP ratio: The review committee advocated for while supply of goods and inputs is being constrained.
a Debt to GDP ratio of 60% to be targeted with a 40%
limit for the centre and 20% limit for the states. Q3. Choose the correct statements.

Under Section 4(2) of the Act, the Centre can exceed 1. Marine Products Export Development Authority
the annual fiscal deficit target citing certain grounds: is an autonomous body under the Ministry of
National security, war Fisheries

Telegram – Indian_Economy_by_Aman_Soni Page 91


2. MPEDA is empowered to carry out inspection of rotational basis. The summit is an informal gathering
marine products, its raw material & fixing that lasts two days, in which leaders of member
standards for sea food countries discuss a wide range of global issues.
The G-7 does not have a formal constitution or a fixed
headquarters.
A. 1 only The decisions taken by leaders during annual summits
B. 2 only are non-binding.
C. Both 1 and 2
D. Neither 1 nor 2
Q5. Choose the correct statements.
Ans - B
Marine Products Export Development Authority- 1. A letter of credit (L/C) is a type of “documentary
Established in 1972, is an autonomous body under the credit” or a “non-fund based credit”.
Ministry of Commerce aimed at increasing export- 2. An ‘Irrevocable Letter of Credit’ means It can be
oriented production, specifying standards, processing modified or cancelled without the agreement of
and export marketing of all kinds of fisheries and its all the three parties.
products.
A. 1 only
MPEDA to regulate exports of marine products and take B. 2 only
all measures required for ensuring sustained, quality C. Both 1 and 2
seafood exports from the country. MPEDA is given the D. Neither 1 nor 2
authority to prescribe for itself any matters which the
future might require for protecting and augmenting the Ans - A
seafood exports from the country. It is also empowered A letter of credit (L/C) is a type of “documentary credit”
to carry out inspection of marine products, its raw or a “non-fund based credit”. It is a document issued by
material, fixing standards, specifications, and training as a bank or financial institution at the request of a buyer
well as take all necessary steps for marketing the whereby the bank or financial institution gives
seafood overseas. assurance of payment to a seller if the terms and
conditions specified in the document are fulfilled. This
means that a Letter of Credit is a promise made by the
Q4. Choose the correct statements. Bank to pay to the exporter / seller on behalf of the
importer / buyer. The seller receives the payment only
1. The G-7 does not have a formal constitution or when all the requirements specified in the L/C are met
a fixed headquarters including the documents, delivery dates, product
2. The decisions taken by leaders during annual specification, etc.
summits are non-binding
Irrevocable Letter of Credit-It cannot be modified or
cancelled without the agreement of all the three
A. 1 only parties.
B. 2 only Revocable Letter of Credit-It may be modified or even
C. Both 1 and 2 cancelled by the issuing bank at any time and without
D. Neither 1 nor 2 notice to the beneficiary. Such L/Cs are rare nowadays
as they give maximum flexibility to the buyers but
Ans – C involve risk to the seller.

The G-7 or ‘Group of Seven’ are Canada, France,


Germany, Italy, Japan, the United Kingdom, and the Q1. Which of the following is a pillar of NITI Aayog &
United States. the Institute of Competitiveness released Export
The G-7 nations meet at annual summits that are Preparedness Index (EPI)
presided over by leaders of member countries on a

Telegram – Indian_Economy_by_Aman_Soni Page 92


1. Policy
2. Business Ecosyste Common Market - A type of custom union where there
3. Export Ecosystem are common policies on product regulation, and free
4. Export Performance movement of goods and services, capital and labour.

A. 1, 2 & 3 only Economic Union -An economic union is a type of trade


B. 1, 2 & 4 only bloc which is composed of a common market with a
C. 2, 3 & 4 only customs union. The participant countries have both
D. 1, 2, 3 & 4 only common policies on product regulation, freedom of
movement of goods, services and the factors of
Ans - D production (capital and labour) and a common external
NITI Aayog in partnership with the Institute of trade policy
Competitiveness releases the Export Preparedness
Index (EPI). It is the first report to examine export
preparedness and performance of Indian states, EPI Q3. The words “Decacorn and Hectocorn” often seen in
intends to identify challenges and opportunities; news is related to :
enhance the effectiveness of government policies; and
encourage a facilitative regulatory framework. A. New supercomputers
B. Anti drone missiles
The structure of the EPI includes 4 pillars –Policy; C. Type of startups
Business Ecosystem; Export Ecosystem; Export D. New virus variants
Performance – and 11 sub-pillars –Export Promotion
Policy; Institutional Framework; Business Environment; Ans – C
Infrastructure; Transport Connectivity; Access to
Finance; Export Infrastructure; Trade Support; R&D Soonicorns are startup companies that have a good
Infrastructure; Export Diversification; and Growth possibility of becoming 'unicorns', with the chance of
Orientation. acquiring late-stage investment. In the world of
business, the term unicorn‘ denotes a private company,
Q2. Which among the following are correctly matched many times a start-up with a stock valuation of over $1
billion.
1. Common Market - A type of custom union
where there are common policies on product The term unicorn is of a recent origin in the business
regulation & free movement of goods and world and was coined by Aileen Lee in 2013. Such
services companies are called unicorn so as to equate them with
2. Economic Union -An economic union is a type of the mythical animal thereby denoting the rarity of such
trade bloc which is composed of a common successful businesses. Similarly, Decacorn and
market with a customs union. Hectocorn are other terms used to depict companies
valued over $10 billion and $100 billion respectively.

A. 1 only Q4. Choose the correct statements.


B. 2 only
C. Both 1 and 2 1. The provisions regarding compulsory licenses
D. Neither 1 nor 2 are given in the Indian Patents Act, 1970
2. Under the act, compulsory licenses may be
Ans - C granted only for exports, under exceptional
Customs Union - An agreement among countries to circumstances
have free trade among themselves and to adopt
common external barriers against any other country
interested in exporting to these countries. Examples – A. 1 only
Gulf Cooperation Council (GCC) B. 2 only

Telegram – Indian_Economy_by_Aman_Soni Page 93


C. Both 1 and 2  Cheque dishonour.
D. Neither 1 nor 2  Money laundering.
 Transactions defrauding creditors.
Ans – A
After hearing the application, a special court
The Patents Act, 1970 was amended three times in (designated under the PMLA, 2002) may declare an
1999, 2002, 2005 respectively to include the concept of individual as a fugitive economic offender. It may
‘compulsory license‘ and these are given in the sections confiscate properties which are proceeds of crime,
84-92 of the Indian Patents Act, 1970. A patent is an Benami properties and any other property, in India or
exclusive right granted for an invention, whether it is a abroad.
product or a process which gives a new technical
solution to a problem, and this patent is granted for a Upon confiscation, all rights and titles of the property
specific period to the inventor. will vest in the central government, free from
encumbrances (such as any charges on the property).
Compulsory licenses may also be granted under : The central government may appoint an administrator
 Section 92 A– For exports, under exceptional to manage and dispose of these properties.
circumstances.
 Section 92A– In case of national emergency,
extreme urgency of public noncommercial use
by notification of the Central Government
 Section 92 A (1) – To a country which has
insufficient or no manufacturing power in the
pharmaceutical sector to address public health

Q5. Choose the correct statements.

1. A special court (designated under the PMLA,


2002) has powers to declare an individual as a
fugitive economic offender
2. All rights and titles of the confiscated property
from the offender, will vest in the central
government, free from encumbrances

A. 1 only
B. 2 only
C. Both 1 and 2
D. Neither 1 nor 2

Ans - C
Fugitive Economic Offenders Act, 2018 seeks to
confiscate properties of economic offenders who have
left the country to avoid facing criminal prosecution or
refuse to return to the country to face prosecution.

Fugitive economic offender is a person against whom an


arrest warrant has been issued for committing an
offence listed in the Act and the value of the offence is
at least Rs. 100 crore.
Some of the offences listed in the act are:
 Counterfeiting government stamps or currency
Telegram – Indian_Economy_by_Aman_Soni Page 94

You might also like